Discrete Book

308
Undergraduate Notes in Mathematics Arkansas Tech University Department of Mathematics Introductory Notes in Discrete Mathematics Marcel B. Finan c All Rights Reserved 2015 Edition

description

Discrete

Transcript of Discrete Book

Page 1: Discrete Book

Undergraduate Notes in Mathematics

Arkansas Tech UniversityDepartment of Mathematics

Introductory Notes in DiscreteMathematics

Marcel B. Financ©All Rights Reserved

2015 Edition

Page 2: Discrete Book
Page 3: Discrete Book

Preface

This book is designed for a one semester course in discrete mathematicsfor sophomore or junior level students. The text covers the mathematicalconcepts that students will encounter in many disciplines such as computerscience, engineering, Business, and the sciences.Besides reading the book, students are strongly encouraged to do all the exer-cises. Mathematics is a discipline in which working the problems is essentialto the understanding of the material contained in this book.Students are encouraged first to do the problems without referring to thesolutions. Answers and Solutions to problems found at the end of this bookcan only be used when you are stuck. Exert a reasonable amount of effortstowards solving a problem before you look up the answer, and rework anyproblem you miss.Students are strongly encouraged to keep up with the exercises and the se-quel of concepts as they are going along, for mathematics builds on itself.A solution guide to the text is available through email: [email protected]

Marcel B. FinanRussellville, ArkansasAugust 2014

i

Page 4: Discrete Book

ii PREFACE

Page 5: Discrete Book

Contents

Preface i

Fundamentals of Mathematical Logic 31 Propositions and Related Concepts . . . . . . . . . . . . . . . . . 42 Basics of Digital Logic Design . . . . . . . . . . . . . . . . . . . . 153 Conditional and Biconditional Propositions . . . . . . . . . . . . 274 Related Propositions: Inference Logic . . . . . . . . . . . . . . . . 335 Predicates and Quantifiers . . . . . . . . . . . . . . . . . . . . . . 44

Fundamentals of Mathematical Proofs 536 Methods of Direct Proof . . . . . . . . . . . . . . . . . . . . . . . 547 More Methods of Proof . . . . . . . . . . . . . . . . . . . . . . . 628 Methods of Indirect Proofs: Contradiction and Contraposition . . 689 Method of Proof by Induction . . . . . . . . . . . . . . . . . . . . 73

Number Theory and Mathematical Proofs 8110 Divisibility. The Division Algorithm . . . . . . . . . . . . . . . . 8211 The Euclidean Algorithm . . . . . . . . . . . . . . . . . . . . . . 89

Fundamentals of Set Theory 9312 Basic Definitions . . . . . . . . . . . . . . . . . . . . . . . . . . . 9413 Properties of Sets . . . . . . . . . . . . . . . . . . . . . . . . . . 10314 Boolean Algebra . . . . . . . . . . . . . . . . . . . . . . . . . . . 111

Relations and Functions 11715 Equivalence Relations . . . . . . . . . . . . . . . . . . . . . . . . 11816 Partial Order Relations . . . . . . . . . . . . . . . . . . . . . . . 13117 Bijective and Inverse Functions . . . . . . . . . . . . . . . . . . . 137

1

Page 6: Discrete Book

2 CONTENTS

18 Recursion . . . . . . . . . . . . . . . . . . . . . . . . . . . . . . 143

Fundamentals of Counting and Probability 15719 The Fundamental Principle of Counting . . . . . . . . . . . . . . 15820 Permutations . . . . . . . . . . . . . . . . . . . . . . . . . . . . 16321 Combinations . . . . . . . . . . . . . . . . . . . . . . . . . . . . 16822 Basic Probability Terms and Rules . . . . . . . . . . . . . . . . . 17423 The Pigeonhole Principle . . . . . . . . . . . . . . . . . . . . . . 186

Basics of Graph Theory 18924 Graphs and the Degree of a Vertex . . . . . . . . . . . . . . . . 19025 Paths and Circuits . . . . . . . . . . . . . . . . . . . . . . . . . 20226 Trees . . . . . . . . . . . . . . . . . . . . . . . . . . . . . . . . . 212

Answer Key 225

Index 301

Page 7: Discrete Book

Fundamentals of MathematicalLogic

Logic is commonly known as the science of reasoning. This introductorychapter covers modern mathematical logic such as propositions and quanti-fiers.

3

Page 8: Discrete Book

4 FUNDAMENTALS OF MATHEMATICAL LOGIC

1 Propositions and Related Concepts

A proposition is any meaningful statement that is either true or false, butnot both. We will use lowercase letters, such as p, q, r, · · · , to representpropositions. We will also use the notation

p : 1 + 1 = 3

to define p to be the proposition 1+1 = 3. The truth value of a propositionis true, denoted by T, if it is a true statement and false, denoted by F, if itis a false statement. Statements that are not propositions include questionsand commands.

Example 1.1Which of the following are propositions? Give the truth value of the propo-sitions.(a) 2 + 3 = 7.(b) Julius Caesar was president of the United States.(c) What time is it?(d) Be quiet !

Solution.(a) A proposition with truth value (F).(b) A proposition with truth value (F).(c) Not a proposition since no truth value can be assigned to this statement.(d) Not a proposition

Example 1.2Which of the following are propositions? Give the truth value of the propo-sitions.(a) The difference of two primes.(b) 2 + 2 = 4.(c) Washington D.C. is the capital of New York.(d) How are you?

Solution.(a) Not a proposition.(b) A proposition with truth value (T).(c) A proposition with truth value (F).

Page 9: Discrete Book

1 PROPOSITIONS AND RELATED CONCEPTS 5

(d) Not a proposition

New propositions called compound propositions or propositional func-tions can be obtained from old ones by using symbolic connectives whichwe discuss next. The propositions that form a propositional function arecalled the propositional variables.Let p and q be propositions. The conjunction of p and q, denoted by p ∧ q(read “p wedge q”), is the proposition: p and q. This proposition is definedto be true only when both p and q are true and it is false otherwise.The disjunction of p and q, denoted by p∨ q (read “p vee q”), is the propo-sition: p or q. The “or” is used in an inclusive way. This proposition is falseonly when both p and q are false, otherwise it is true.

Example 1.3Let

p : 5 < 9

q : 9 < 7.

Construct the propositions p ∧ q and p ∨ q.

Solution.The conjunction of the propositions p and q is the proposition

p ∧ q : 5 < 9 and 9 < 7.

This proposition is false since the proposition 9 < 7 has a truth value F.The disjunction of the propositions p and q is the proposition

p ∨ q : 5 < 9 or 9 < 7

which is a true proposition

Example 1.4Consider the following propositions

p : It is Friday

q : It is raining.

Construct the propositions p ∧ q and p ∨ q.

Page 10: Discrete Book

6 FUNDAMENTALS OF MATHEMATICAL LOGIC

Solution.The conjunction of the propositions p and q is the proposition

p ∧ q : It is Friday and it is raining.

The disjunction of the propositions p and q is the proposition

p ∨ q : It is Friday or It is raining

A truth table displays the relationships between the truth values of propo-sitions. Next, we display the truth tables of p ∧ q and p ∨ q.

p q p ∧ qT T TT F FF T FF F F

p q p ∨ qT T TT F TF T TF F F

Let p and q be two propositions. The exclusive or (or exclusive disjunc-tion) of p and q, denoted p⊕ q, is the proposition that is true when exactlyone of p and q is true and is false otherwise. The truth table of the exclusive“or” is displayed below

p q p⊕ qT T FT F TF T TF F F

Example 1.5(a) Construct a truth table for (p⊕ q)⊕ r.(b) Construct a truth table for p⊕ p.

Solution.(a) The truth table is

Page 11: Discrete Book

1 PROPOSITIONS AND RELATED CONCEPTS 7

p q r p⊕ q (p⊕ q)⊕ rT T T F TT T F F FT F T T FT F F T TF T T T FF T F T TF F T F TF F F F F

(b) The truth table is

p p⊕ pT FF F

The final operation on a proposition p that we discuss is the negation of p.The negation of p, denoted ∼ p, is the proposition not p. The truth table of∼ p is displayed below

p ∼ pT FF T

Example 1.6Consider the following propositions:p: Today is Thursday.q: 2 + 1 = 3.r: There is no pollution in New Jersey.Construct the truth table of [∼ (p ∧ q)] ∨ r.Solution.

p q r p ∧ q ∼ (p ∧ q) [∼ (p ∧ q)] ∨ rT T T T F TT T F T F FT F T F T TT F F F T TF T T F T TF T F F T TF F T F T TF F F F T T

Page 12: Discrete Book

8 FUNDAMENTALS OF MATHEMATICAL LOGIC

Example 1.7Find the negation of the proposition p : −5 < x ≤ 0.

Solution.The negation of p is the proposition ∼ p : x > 0 or x ≤ −5

A compound proposition is called a tautology if it is always true, regardlessof the truth values of the propositional variables which comprise it.

Example 1.8(a) Construct the truth table of the proposition (p∧ q)∨ (∼ p∨ ∼ q). Deter-mine if this proposition is a tautology.(b) Show that p∨ ∼ p is a tautology.

Solution.(a) The truth table is

p q ∼ p ∼ q ∼ p∨ ∼ q p ∧ q (p ∧ q) ∨ (∼ p∨ ∼ q)T T F F F T TT F F T T F TF T T F T F TF F T T T F T

Thus, the given proposition is a tautology.(b) The truth table is

p ∼ p p∨ ∼ pT F TF T T

Again, this proposition is a tautology

Two propositions are equivalent if they have exactly the same truth valuesunder all circumstances. We write p ≡ q.

Example 1.9(a) Show that ∼ (p ∨ q) ≡∼ p∧ ∼ q.(b) Show that ∼ (p ∧ q) ≡∼ p∨ ∼ q.(c) Show that ∼ (∼ p) ≡ p.Parts (a) and (b) are known as DeMorgan’s laws.

Page 13: Discrete Book

1 PROPOSITIONS AND RELATED CONCEPTS 9

Solution.(a) The truth table is

p q ∼ p ∼ q p ∨ q ∼ (p ∨ q) ∼ p∧ ∼ qT T F F T F FT F F T T F FF T T F T F FF F T T F T T

Note that the columns of ∼ (p∨q) and ∼ p∧ ∼ q have the same truth values.(b) The truth table is

p q ∼ p ∼ q p ∧ q ∼ (p ∧ q) ∼ p∨ ∼ qT T F F T F FT F F T F T TF T T F F T TF F T T F T T

Note that the columns of ∼ (p∧q) and ∼ p∨ ∼ q have the same truth values.(c) The truth table is

p ∼ p ∼ (∼ p)T F TF T F

Example 1.10(a) Show that p ∧ q ≡ q ∧ p and p ∨ q ≡ q ∨ p.(b) Show that (p ∨ q) ∨ r ≡ p ∨ (q ∨ r) and (p ∧ q) ∧ r ≡ p ∧ (q ∧ r).(c) Show that (p∧ q)∨ r ≡ (p∨ r)∧ (q ∨ r) and (p∨ q)∧ r ≡ (p∧ r)∨ (q ∧ r).

Solution.(a) The truth table is

p q p ∧ q q ∧ pT T T TT F F FF T F FF F F F

Page 14: Discrete Book

10 FUNDAMENTALS OF MATHEMATICAL LOGIC

p q p ∨ q q ∨ pT T T TT F T TF T T TF F F F

(b) The truth table is

p q r p ∨ q q ∨ r (p ∨ q) ∨ r p ∨ (q ∨ r)T T T T T T TT T F T T T TT F T T T T TT F F T F T TF T T T T T TF T F T T T TF F T F T T TF F F F F F F

p q r p ∧ q q ∧ r (p ∧ q) ∧ r p ∧ (q ∧ r)T T T T T T TT T F T F F FT F T F F F FT F F F F F FF T T F T F FF T F F F F FF F T F F F FF F F F F F F

(c) The truth table is

p q r p ∧ q p ∨ r q ∨ r (p ∧ q) ∨ r (p ∨ r) ∧ (q ∨ r)T T T T T T T TT T F T T T T TT F T F T T T TT F F F T F F FF T T F T T T TF T F F F T F FF F T F T T T TF F F F F F F F

Page 15: Discrete Book

1 PROPOSITIONS AND RELATED CONCEPTS 11

p q r p ∨ q p ∧ r q ∧ r (p ∨ q) ∧ r (p ∧ r) ∨ (q ∧ r)T T T T T T T TT T F T F F F FT F T T T F T TT F F T F F F FF T T T F T T TF T F T F F F FF F T F F F F FF F F F F F F F

Example 1.11Show that ∼ (p ∧ q) 6≡∼ p∧ ∼ q

Solution.We will use truth tables to prove the claim.

p q ∼ p ∼ q p ∧ q ∼ (p ∧ q) ∼ p∧ ∼ qT T F F T F FT F F T F T 6= FF T T F F T 6= FF F T T F T T

A compound proposition that has the value F for all possible values of thepropositions in it is called a contradiction.

Example 1.12Show that the proposition p∧ ∼ p is a contradiction.

Solution.

p ∼ p p∧ ∼ pT F FF T F

Page 16: Discrete Book

12 FUNDAMENTALS OF MATHEMATICAL LOGIC

Review Problems

Problem 1.1Indicate which of the following sentences are propositions.(a) 1,024 is the smallest four-digit number that is a perfect square.(b) She is a mathematics major.(c) 128 = 26.(d) x = 26.

Problem 1.2Consider the propositions:p: Juan is a math major.q: Juan is a computer science major.

Use symbolic connectives to represent the proposition “Juan is a math majorbut not a computer science major.”

Problem 1.3In the following sentence is the word “or” used in its inclusive or exclusivesense? “A team wins the playoffs if it wins two games in a row or a total ofthree games.”

Problem 1.4Write the truth table for the proposition: (p ∨ (∼ p ∨ q))∧ ∼ (q∧ ∼ r).

Problem 1.5Let t be a tautology. Show that p ∨ t ≡ t.

Problem 1.6Let c be a contradiction. Show that p ∨ c ≡ p.

Problem 1.7Show that (r ∨ p) ∧ [(∼ r ∨ (p ∧ q)) ∧ (r ∨ q)] ≡ p ∧ q.

Problem 1.8Use De Morgan’s laws to write the negation for the proposition: “This com-puter program has a logical error in the first ten lines or it is being run withan incomplete data set.”

Page 17: Discrete Book

1 PROPOSITIONS AND RELATED CONCEPTS 13

Problem 1.9Use De Morgan’s laws to write the negation for the proposition: “The dollaris at an all-time high and the stock market is at a record low.”

Problem 1.10Assume x ∈ R.Use De Morgan’s laws to write the negation for the proposition:−5 <x ≤ 0.

Problem 1.11Show that the proposition s = (p ∧ q) ∨ (∼ p ∨ (p∧ ∼ q)) is a tautology.

Problem 1.12Show that the proposition s = (p∧ ∼ q) ∧ (∼ p ∨ q) is a contradiction.

Problem 1.13(a) Find simpler proposition forms that are logically equivalent to p⊕ p andp⊕ (p⊕ p).(b) Is (p⊕ q)⊕ r ≡ p⊕ (q ⊕ r)? Justify your answer.(c) Is (p⊕ q) ∧ r ≡ (p ∧ r)⊕ (q ∧ r)? Justify your answer.

Problem 1.14Show the following:(a) p ∧ t ≡ p, where t is a tautology.(b) p ∧ c ≡ c, where c is a contradiction.(c) ∼ t ≡ c and ∼ c ≡ t, where t is a tautology and c is a contradcition.(d) p ∨ p ≡ p and p ∧ p ≡ p.

Problem 1.15Which of the following statements are propositions?(a) The Earth is round.(b) Do you know how to swim?(c) Please leave the room.(d) x+ 3 = 5.(e) Canada is in Asia.

Problem 1.16Write the negation of the following propositions:(a) ∼ p ∧ q.(b) John is not at work or Peter is at the gym.

Page 18: Discrete Book

14 FUNDAMENTALS OF MATHEMATICAL LOGIC

Problem 1.17Construct the truth table of the compound proposition (p ∧ q) ∨ (∼ p).

Problem 1.18Show that p⊕ q ≡ (p ∨ q)∧ ∼ (p ∧ q).

Problem 1.19Show that p∨ ∼ (p ∧ q) is a tautology.

Problem 1.20Show that ∼ p ∧ (p ∧ q) is a contradiction.

Page 19: Discrete Book

2 BASICS OF DIGITAL LOGIC DESIGN 15

2 Basics of Digital Logic Design

In this section we discuss the logic of digital circuits which are considered tobe the basic components of most digital systems, such as electronic comput-ers, electronic phones, traffic light controls, etc.The purpose of digital systems is to manipulate discrete information whichare represented by physical quantities such as voltages and current. Thesmallest representation unit is one bit, short for binary digit. Since electronicswitches have two physical states, namely high voltage and low voltage weattribute the bit 1 to high voltage and the bit 0 for low voltage.A logic gate is the smallest processing unit in a digital system. It takes oneor few bits as input and generates one bit as an output.A circuit is composed of a number of logic gates connected by wires. Ittakes a group of bits as input and generates one or more bits as output.The six basic logic gates are the following:(1) NOT gate (also called inverter): Takes an input of 0 to an output of1 and an input of 1 to an output of 0. The corresponding logical symbol is∼ P.(2) AND gate: Takes two bits, P and Q, and outputs 1 if P and Q are 1 and0 otherwise. The logical symbol is P ∧ Q. In Boolean algebra notation, oneuses P ·Q.(3) OR gate: outputs 1 if either P or Q is 1 and 0 otherwise. The logicalsymbol is P ∨Q. The corresponding Boolean algebra notation is P +Q.(4) NAND gate: outputs a 0 if both P and Q are 1 and 1 otherwise. Thesymbol is ∼ (P ∧ Q). Also, denoted by P |Q, where | is called a Schefferstroke.(5) NOR gate: output a 0 if at least one of P or Q is 1 and 1 otherwise. Thesymbol is ∼ (P ∨Q) or P ↓ Q, where ↓ is a Pierce arrow.(6) XOR gate or the exclusive or: Outputs a 1 if exactly one of the inputs is1 and 0 otherwise. The symbol is P ⊕Q.

Example 2.1Construct the input/output tables of the gates discussed in this section.

Solution.Table for NOT-gate:

P ∼ P1 00 1

Page 20: Discrete Book

16 FUNDAMENTALS OF MATHEMATICAL LOGIC

Table for AND-gate:

P Q P ∧Q1 1 11 0 00 1 00 0 0

Table for OR-gate:

P Q P ∨Q1 1 11 0 10 1 10 0 0

Table for NAND-gate:

P Q ∼ (P ∧Q)1 1 01 0 10 1 10 0 1

Table for NOR-gate:

P Q ∼ (P ∨Q)1 1 01 0 00 1 00 0 1

Table for XOR-gate:

P Q P ⊕Q1 1 01 0 10 1 10 0 0

Graphical representations of the logic gates are shown in Figure 2.1.

Page 21: Discrete Book

2 BASICS OF DIGITAL LOGIC DESIGN 17

Figure 2.1

If you are given a set of input signals for a circuit, you can find its outputby tracing through the circuit gate by gate.

Example 2.2Give the output signal S for the following circuit, given that P = 0, Q = 1,and R = 0 :

Solution.The circuit is shown in Figure 2.2

Page 22: Discrete Book

18 FUNDAMENTALS OF MATHEMATICAL LOGIC

Figure 2.2

A variable with exactly two possible values is called a Boolean variable.A Boolean expression is an expression composed of Boolean variables andconnectives (which are the gates in this section).

Example 2.3Find the Boolean expression that corresponds to the circuit of Example 2.2.

Solution.The Boolean expression is (P ∨Q) ∧ (P ∨R)

Two digital logic circuits are equivalent if, and only if, their correspondingBoolean expressions are logically equivalent. Alternatively, the two Booleanexpressions have the same truth table.

Example 2.4Show that the following two circuits are equivalent:

Page 23: Discrete Book

2 BASICS OF DIGITAL LOGIC DESIGN 19

Solution.The Boolean expression corresponding to (a) is given by (P ∧ Q) ∨ Q andthat corresponding to (b) is given by (P ∨Q)∧Q. These two expressions arelogically equivalent:

(P ∧Q) ∨Q ≡ (P ∨Q) ∧ (Q ∨Q)

≡(P ∨Q) ∧Q

In the next example, we describe the process of converting a number frombase 10 to base 2 (binary) and vice versa.

Example 2.5(a) Write the number 1, 99810 in base 2.(b) Write the number 110012 in base 10.

Solution.(a) Let q denote the quotient of the division of a by b and r denote theremainder. We have

a b q r1,998 2 999 0999 2 499 1499 2 249 1249 2 124 1124 2 62 062 2 31 031 2 15 115 2 7 17 2 3 13 2 1 11 2 0 1.

Hence,1, 99810 = 111110011102.

(b) We have

110012 = 1× 24 + 1× 23 + 0× 22 + 0× 2 + 1 = 25

Page 24: Discrete Book

20 FUNDAMENTALS OF MATHEMATICAL LOGIC

Integers Binary RepresentationsSeveral methods have been used for expressing negative integers in the com-puter. The most obvious way is to convert the number to binary and stickon another bit to indicate sign, 0 for positive and 1 for negative. Supposethat integers are stored using this signed-magnitude technique in 8 bits sothat the leftmost bit holds the sign while the remaining bits represent themagnitude. Thus, +4110 = 00101001 and −4110 = 10101001.The above procedure has a gap. How one would represent the bit 0? Well,there are two ways for storing 0. One way is 00000000 which represents+0 and a second way 10000000 represents −0. A method for representingnumbers that avoid this problem is called the two’s complement. Con-sidering −4110 again, first, convert the absolute value to binary obtaining4110 = 00101001. Then take the complement of each bit obtaining 11010110.This is called the one’s complement of 41. To complete the procedure,increment by 1 the one’s complement to obtain −4110 = 11010111.Conversion of +4110 to two’s complement consists merely of expressing thenumber in binary, i.e., +4110 = 00101001.

Example 2.6(a) Represent the integer −610 using one’s complement.(b) Represent the integer −610 using two’s complement.

Solution.(a) We have 610 = 01102. The one’s complement is −610 = 1001.(b) The two’s complement is −610 = 1001 + 1 = 1010Now, an algorithm to find the decimal representation of a negative integerwith a given 8-bit two’s complement is the following:1. Find the two’s complement of the given two’s complement,2. write the decimal equivalent of the result.

Example 2.7(a) What is the decimal representation for the integer with 8-bit two’s com-plement 10101001?(b) What is the decimal representation for the integer with 8-bit two’s com-plement 00101111?

Solution.(a) The two’s complement of 10101001 is 01010111 = 8710. Thus, the number

Page 25: Discrete Book

2 BASICS OF DIGITAL LOGIC DESIGN 21

is −8710.(b) Since the integer is positive, 00101111 = 1011112 = 1 · 25 + 1 · 23 + 1 ·22 + 1 · 2 + 1 = 4710

Page 26: Discrete Book

22 FUNDAMENTALS OF MATHEMATICAL LOGIC

Review Problems

Problem 2.1Write the input/output table for the circuit of Example 2.2 where P,Q, andR are any inputs.

Problem 2.2Construct the circuit corresponding to the Boolean expression: (P ∧Q)∨ ∼R.

Problem 2.3For the following input/output table, construct (a) the corresponding Booleanexpression and (b) the corresponding circuit:

P Q R S1 1 1 01 1 0 11 0 1 01 0 0 00 1 1 00 1 0 00 0 1 00 0 0 0

Problem 2.4Consider the following circuit

Page 27: Discrete Book

2 BASICS OF DIGITAL LOGIC DESIGN 23

Complete the following table

P Q R S1 11 00 10 0

Problem 2.5(a) Convert 104310 to base 2.(b) Convert 011011012 to base 10.

Problem 2.6Express the numbers 104 and −104 in two’s complement representation with8 bits.

Problem 2.7Construct the input/output table of the circuit given below.

Problem 2.8The negation of the exclusive or is the exclusive nor (abbreviated by XNOR)whose gate is shown below.

Page 28: Discrete Book

24 FUNDAMENTALS OF MATHEMATICAL LOGIC

Contruct the input/output table of this gate.

Problem 2.9Show that A⊕ A and A⊕ (∼ A) are constants.

Problem 2.10Construct a circuit whose Boolean expression is Q = (A ∧ B) ∨ [(B ∧ C) ∧(B ∨ C)].

Problem 2.11Find the Boolean expression that corresponds to the circuit.

Problem 2.12Find the Boolean expression S that corresponds to the input output table,where P,Q, and R are the Boolean variables.

P Q R S1 1 1 11 1 0 01 0 1 11 0 0 00 1 1 00 1 0 00 0 1 00 0 0 0

Page 29: Discrete Book

2 BASICS OF DIGITAL LOGIC DESIGN 25

Problem 2.13Show that the following two circuits are equivalent:

Problem 2.14Show that the following two circuits are equivalent:

Problem 2.15Let A,B,C and D be the Boolean variables of the circuit below. Find theBoolean Expressions: X, Y, Z,W, and Q.

Page 30: Discrete Book

26 FUNDAMENTALS OF MATHEMATICAL LOGIC

Problem 2.16(a) Write the number 51310 in base 2.(b) Write the number 11102 in base 10.

Problem 2.17Write the two’s complement values of the numbers 7 and −2 in (a) 4-bitformat (b) 8-bit format (c) 16-bit format.

Problem 2.18Represent the integer −7210 as an 8-bit two’s complement.

Problem 2.19Convert 8110 to an 8-bit two’s complememt.

Problem 2.20What is the decimal representation for the integer with 8-bit two’s comple-ment 10010011?

Problem 2.21What is the decimal representation for the integer with 8-bit two’s comple-ment 01001000?

Page 31: Discrete Book

3 CONDITIONAL AND BICONDITIONAL PROPOSITIONS 27

3 Conditional and Biconditional Propositions

Let p and q be propositions. The conditional proposition p → q is theproposition that is false only when p is true and q is false; otherwise it is true.p is called the hypothesis and q is called the conclusion. The connective→ is called the conditional connective.

Example 3.1Construct the truth table of the implication p→ q.

Solution.The truth table is

p q p→ qT T TT F FF T TF F T

Example 3.2Show that p→ q ≡ (∼ p) ∨ q.

Solution.

p q ∼ p p→ q (∼ p) ∨ qT T F T TT F F F FF T T T TF F T T T

It follows from the previous example that the proposition p → q is alwaystrue if the hypothesis p is false, regardless of the truth value of q. We saythat p→ q is true by default or vacuously true.In terms of words the proposition p→ q also reads:(a) if p then q.(b) p implies q.(c) p is a sufficient condition for q.(d) q is a necessary condition for p.(e) p only if q.

Page 32: Discrete Book

28 FUNDAMENTALS OF MATHEMATICAL LOGIC

Remark 3.1In a purely logical sense, conditional sentences do not necessarily imply acause and effect between the components p and q, although in mathematicsand in general discourse they do. From a logical point of view the proposition“If 2 + 3 = 7 then I −1 > 0” is a true proposition although there is norelationship between the component parts.

In propositional functions that involve the connectives ∼,∧,∨, and → theorder of operations is that ∼ is performed first and → is performed last.

Example 3.3Show that ∼ (p→ q) ≡ p ∧ (∼ q).

Solution.We use De Morgan’s laws as follows.

∼ (p→ q) ≡ ∼ [(∼ p) ∨ q)≡[∼ (∼ p)] ∧ (∼ q)

≡p ∧ (∼ q)

The converse of p→ q is the proposition q → p. The opposite or inverseof p→ q is the proposition ∼ p→∼ q. The contrapositive of p→ q is theproposition ∼ q →∼ p.

Example 3.4Find the converse, opposite, and the contrapositive of the implication: “ If1 + 2 = 4 then President Lincoln is from Illinois.”

Solution.The converse: If President Lincoln is from Illinois then 1 + 2 = 4.The opposite: If 1 + 2 6= 4 then President Lincoln is not from Illinois.The contrapositive: If President Lincoln is not from Illinois then 1 + 2 6= 4

Example 3.5Show that p→ q ≡∼ q →∼ p.

Page 33: Discrete Book

3 CONDITIONAL AND BICONDITIONAL PROPOSITIONS 29

Solution.We use De Morgan’s laws as follows.

p→ q ≡(∼ p) ∨ q≡ ∼ [p ∧ (∼ q)]

≡ ∼ [(∼ q) ∧ p]≡[∼ (∼ q)] ∨ (∼ p)

≡q ∨ (∼ p)

≡ ∼ q →∼ p

Example 3.6Using truth tables show the following:(a) p→ q 6≡ q → p.(b) p→ q 6≡∼ p→∼ q.

Solution.(a) It suffices to show that (∼ p) ∨ q 6≡ (∼ q) ∨ p.

p q ∼ p ∼ q (∼ p) ∨ q (∼ q) ∨ pT T F F T TT F F T F 6= TF T T F T 6= FF F T T T T

(b) We will show that (∼ p) ∨ q 6≡ p ∨ (∼ q).

p q ∼ p ∼ q (∼ p) ∨ q p ∨ (∼ q)T T F F T TT F F T F 6= TF T T F T 6= FF F T T T T

The biconditional proposition of p and q, denoted by p ↔ q, is thepropositional function that is true when both p and q have the same truthvalues and false if p and q have opposite truth values. Also reads, “p if andonly if q” or “p is a necessary and sufficient condition for q.”

Example 3.7Construct the truth table for p↔ q.

Page 34: Discrete Book

30 FUNDAMENTALS OF MATHEMATICAL LOGIC

Solution.

p q p↔ qT T TT F FF T FF F T

Example 3.8Show that the biconditional proposition of p and q is logically equivalent tothe conjunction of the conditional propositions p→ q and q → p.

Solution.

p q p→ q q → p p↔ q (p→ q) ∧ (q → p)T T T T T TT F F T F FF T T F F FF F T T T T

The order of operations for the five logical connectives is as follows:1. ∼2. ∧,∨ in any order.3. →,↔ in any order.

Page 35: Discrete Book

3 CONDITIONAL AND BICONDITIONAL PROPOSITIONS 31

Review Problems

Problem 3.1Construct the truth table for the proposition: (∼ p) ∨ q → r.

Problem 3.2Construct the truth table for the proposition: (p→ r)↔ (q → r).

Problem 3.3Write negations for each of the following propositions.(a) If 2 + 2 = 4, then 2 is a prime number.(b) If 1 = 0 then

√2 is rational.

Problem 3.4Write the contrapositives for the propositions of Problem 3.3.

Problem 3.5Write the converses for the propositions of problem 3.3

Problem 3.6Write the inverses for the propositions of problem 3.3

Problem 3.7Show that p ∨ q ≡ (p→ q)→ q.

Problem 3.8Show that ∼ (p↔ q) ≡ (p∧ ∼ q) ∨ (∼ p ∧ q).

Problem 3.9Let p : 2 > 3 and q : 0 < 5. Find the truth value of p→ q and q → p.

Problem 3.10Assuming that p is true, q is false, and r is true, find the truth value of eachproposition.(a) p ∧ q → r.(b) p ∨ q →∼ r.(c) p ∨ (q → r).

Page 36: Discrete Book

32 FUNDAMENTALS OF MATHEMATICAL LOGIC

Problem 3.11Show using a chain of logical equivalences that (p→ r)∧(q → r) ≡ (p∨q)→r.

Problem 3.12Show using a chain of logical equivalences that p↔ q ≡ (p∧q)∨ (∼ p∧ ∼ q).

Problem 3.13(a) What are the truth values of p and q for which a conditional propositionand its inverse are both true?(b) What are the truth values of p and q for which a conditional propositionand its inverse are both false?

Problem 3.14Show that p↔ q ≡∼ (p⊕ q).

Problem 3.15Determine whether the following propostion is true or false: “If the moon ismade of milk then I am smarter than Einstein.”

Problem 3.16Construct the truth table of ∼ p ∧ (p→ q).

Problem 3.17Show that (p→ q) ∨ (q → q) is a tautology.

Problem 3.18Construct a truth table for (p→ q) ∧ (q → r).

Problem 3.19Find the converse, inverse, and contrapositive of“It is raining is a necessarycondition for me not going to town.

Problem 3.20Show that (p∧ ∼ q) ∨ q ↔ p ∨ q is a tautology.

Page 37: Discrete Book

4 RELATED PROPOSITIONS: INFERENCE LOGIC 33

4 Related Propositions: Inference Logic

The main concern of logic is how the truth of some propositions is connectedwith the truth of another. Thus, we will usually consider a group of relatedpropositions.An argument is a set of two or more propositions related to each other insuch a way that all but one of them, the premises, are supposed to providesupport for the remaining one, the conclusion.The transition from premises to conclusion is the inference upon which theargument relies.

Example 4.1Show that the propositions “The star is made of milk, and strawberries arered. My dog has fleas.” do not form an argument.

Solution.Indeed, the truth or falsity of each of the propositions has no bearing on thatof the others

Example 4.2Show that the propositions: “Mark is a lawyer. So Mark went to law schoolsince all lawyers have gone to law school” form an argument.

Solution.This is an argument. The truth of the conclusion, “Mark went to law school,”is inferred or deduced from its premises, “Mark is a lawyer” and “all lawyershave gone to law school.”

The above argument can be represented as follows: Letp : Mark is a lawyer.q : All lawyers have gone to law school.r : Mark went to law school.Then

p ∧ q

.̇. r

Page 38: Discrete Book

34 FUNDAMENTALS OF MATHEMATICAL LOGIC

The symbol .̇. is to indicate the inferrenced conclusion.

Now, suppose that the premises of an argument are all true. Then theconclusion may be either true or false. When the conclusion is true then theargument is said to be valid. When the conclusion is false then the argumentis said to be invalid.To test an argument for validity one proceeds as follows:(1) Identify the premises and the conclusion of the argument.(2) Construct a truth table including the premises and the conclusion.(3) Find rows in which all premises are true.(4) In each row of Step (3), if the conclusion is true then the argument isvalid; otherwise the argument is invalid.

Example 4.3Show that the argument

p→ q

q → p

.̇. p ∨ q

is invalid

Solution.We construct the truth table as follows.

p q p→ q q → p p ∨ qT T T T TT F F T TF T T F TF F T T F

From the last row we see that the premises are true but the conclusion isfalse. The argument is then invalid

Page 39: Discrete Book

4 RELATED PROPOSITIONS: INFERENCE LOGIC 35

Next, we discuss some basic rules of inference.

Example 4.4 (Modus Ponens or the method of affirmative)Show that the argument

p→ q

p

.̇. q

is valid.

Solution.The truth table is as follows.

p q p→ qT T TT F FF T TF F T

The first row shows that the argument is valid

Example 4.5 (Modus Tollens or the method of denial)Show that the argument

p→ q

∼ q

.̇. ∼ p

is valid.

Solution.The truth table is as follows.

p q p→ q ∼ q ∼ pT T T F FT F F T FF T T F TF F T T T

The last row shows that the argument is valid

Page 40: Discrete Book

36 FUNDAMENTALS OF MATHEMATICAL LOGIC

Example 4.6 (Disjunctive Addition)Show that the argument

p

.̇. p ∨ q

is valid.

Solution.The truth table is as follows.

p q p ∨ qT T TT F TF T TF F F

The first and second rows show that the argument is valid

Example 4.7 (Conjunctive addition)Show that

p, q

.̇. p ∧ q

is valid

Solution.The truth table is as follows.

p q p ∧ qT T TT F FF T FF F F

The first row shows that the argument is valid

Page 41: Discrete Book

4 RELATED PROPOSITIONS: INFERENCE LOGIC 37

Example 4.8 (Conjunctive simplification)Show that the argument

p ∧ q

.̇. p

is valid.

Solution.The truth table is as follows.

p q p ∧ qT T TT F FF T FF F F

The first row shows that the argument is valid

Example 4.9 (Disjunctive syllogism)Show that the argument

p ∨ q∼ q

.̇. p

is valid.

Solution.The truth table is as follows.

p q ∼ p ∼ q p ∨ qT T F F TT F F T TF T T F TF F T T F

The second row shows that the argument is valid

Page 42: Discrete Book

38 FUNDAMENTALS OF MATHEMATICAL LOGIC

Example 4.10 (Hypothetical syllogism)Show that the argument

p→ q

q → r

.̇. p→ r

is valid.

Solution.The truth table is as follows.

p q r p→ q q → r p→ rT T T T T TT T F T F FT F T F T TT F F F T FF T T T T TF T F T F TF F T T T TF F F T T T

The first , fifth, seventh, and eighth rows show that the argument is valid

Example 4.11 (Rule of contradiction)Show that if c is a contradiction then the following argument is valid for anyp.

∼ p→ c

.̇. p

Solution.Constructing the truth table we find

c p ∼ p→ cF T TF F F

The first row shows that the argument is valid

Page 43: Discrete Book

4 RELATED PROPOSITIONS: INFERENCE LOGIC 39

Review Problems

Problem 4.1Show that the propositions “Abraham Lincolm was the president of theUnited States, and blueberries are blue. My car is yellow” do not forman argument.

Problem 4.2Show that the propositions: “Steve is a physician. So Steve went to medicalschool since all doctors have gone to medical school” form an argument.Identify the premises and the conclusion.

Problem 4.3Show that the argument

∼ p ∨ q → r

∼ p ∨ q

.̇. r

is valid.

Problem 4.4Show that the argument

p→ q

q

.̇. p

is invalid.

Problem 4.5Show that the argument

p→ q

∼ p

.̇. ∼ q

is invalid.

Page 44: Discrete Book

40 FUNDAMENTALS OF MATHEMATICAL LOGIC

Problem 4.6Show that the argument

q

.̇. p ∨ q

is valid.

Problem 4.7Show that the argument

p ∧ q

.̇. q

is valid.

Problem 4.8Show that the argument

p ∨ q∼ p

.̇. q

is valid.

Problem 4.9Use modus ponens or modus tollens to fill in the blanks in the argumentbelow so as to produce valid inferences.

If√

2 is rational, then√

2 = ab

for some integers a and b.

It is not true that√

2 = ab

for some integers a and b.

.̇.

Page 45: Discrete Book

4 RELATED PROPOSITIONS: INFERENCE LOGIC 41

Problem 4.10Use modus ponens or modus tollens to fill in the blanks in the argumentbelow so as to produce valid inferences.

If logic is easy, then I am a monkey’s uncle.I am not a monkey’s uncle..̇.

Problem 4.11Use a truth table to determine whether the argument below is invalid.

p→ q

q → p

.̇. p ∧ q

Problem 4.12Use a truth table to determine whether the argument below is valid.

p

p→ q

∼ q ∨ r

.̇. r

Problem 4.13Use symbols to write the logical form of the given argument and then use atruth table to test the argument for validity.If Tom is not on team A, then Hua is on team B.If Hua is not on team B, then Tom is on team A..̇. Tom is not on team A or Hua is not on team B.

Problem 4.14Use symbols to write the logical form of the given argument and then use atruth table to test the argument for validity.If Jules solved this problem correctly, then Jules obtained the answer 2.Jules obtained the answer 2..̇. Jules solved this problem correctly.

Page 46: Discrete Book

42 FUNDAMENTALS OF MATHEMATICAL LOGIC

Problem 4.15Use symbols to write the logical form of the given argument and then use atruth table to test the argument for validity.If this number is larger than 2, then its square is larger than 4.This number is not larger than 2..̇. The square of this number is not larger than 4.

Problem 4.16Use the valid argument forms of this section to deduce the conclusion fromthe premises.

∼ p ∨ q → r

s∨ ∼ q

∼ t

p→ t

∼ p ∧ r →∼ s

.̇. ∼ q

Problem 4.17Use the valid argument forms of this section to deduce the conclusion fromthe premises.

∼ p→ r∧ ∼ s

t→ s

u→∼ p

∼ w

u ∨ w

.̇. ∼ t ∨ w

Problem 4.18Use the valid argument forms of this section to deduce the conclusion from

Page 47: Discrete Book

4 RELATED PROPOSITIONS: INFERENCE LOGIC 43

the premises.

∼ (p ∨ q)→ r

∼ p

∼ r

.̇. q

Problem 4.19Use the valid argument forms of this section to deduce the conclusion fromthe premises.

p ∧ qp→∼ (q ∧ r)s→ r

.̇. ∼ s

Problem 4.20Show that

p→ q → r

∼ (q → r)

.̇. ∼ p

Page 48: Discrete Book

44 FUNDAMENTALS OF MATHEMATICAL LOGIC

5 Predicates and Quantifiers

Statements such as “x > 3” or “x2 + 4 ≥ 4” are often found in mathematicalassertions and in computer programs. These statements are not propositionswhen the variables are not specified. However, one can produce propositionsfrom such statements.A predicate is an expression involving one or more variables defined on somedomain, called the domain of discourse. Substitution of a particular valuefor the variable(s) produces a proposition which is either true or false. Forinstance, P (n) : n is prime is a predicate on the set of natural numbers1 N.Observe that P (1) is false, P (2) is true. In the expression P (x), x is called afree variable. As x varies the truth value of P (x) varies as well. The set oftrue values of a predicate P (x) is called the truth set and will be denotedby TP .

Example 5.1Let Q(x, y) : x = y + 3 with domain Z+. What are the truth values of thepropositions Q(1, 2) and Q(3, 0)?

Solution.By substitution in the expression of Q we find: Q(1, 2) is false since 1 = x 6=y + 3 = 5. On the contrary, Q(3, 0) is true since x = 3 = 0 + 3 = y + 3

If P (x) and Q(x) are two predicates with a common domain D then the nota-tion P (x)⇒ Q(x) means that every element in the truth set of P (x) is also anelement in the truth set of Q(x). Same logical manipulations that were usedwith propositions can be used with predicates. For example, P (x) ⇒ Q(x)is the same as ∼ P (x) ∨Q(x).

Example 5.2Consider the two predicates P (x) : x is a factor of 4 and Q(x) : x is a factorof 8. Show that P (x)⇒ Q(x).

Solution.Finding the truth set of each predicate we have: TP = {1, 2, 4} and TQ ={1, 2, 4, 8}. Since every number appearing in TP also appears in TQ we have

1We define the set of natural numbers to be the set N = {1, 2, 3, · · · }. We will denotethe set of all non-negative integers or the set of whole numbers by W = Z+ = {0, 1, 2, · · · }.

Page 49: Discrete Book

5 PREDICATES AND QUANTIFIERS 45

P (x)⇒ Q(x)

If two predicates P (x) and Q(x) with a common domain D are such thatTP = TQ then we use the notation P (x)⇔ Q(x).

Example 5.3Let D = R. Consider the two predicates P (x) : −2 ≤ x ≤ 2 and Q(x) : |x| ≤2. Show that P (x)⇔ Q(x).

Solution.Indeed, if x in TP then the distance from x to the origin is at most 2. That is,|x| ≤ 2 and hence x belongs to TQ. Now, if x is an element in TQ then |x| ≤ 2,i.e. (x−2)(x+2) ≤ 0. Solving this inequality we find that −2 ≤ x ≤ 2. Thatis, x ∈ TP

QuantifiersA quantifier turns a predicate into a proposition without assigning spe-cific values for the variable. There are primarily two quantifiers: Universalquantifier and existential quantifier. The universal quantification of apredicate P (x) is the proposition ∀x ∈ D,P (x) is true, where the symbol ∀is the universal quantifier. For example, if k is a non-negative integer, thenthe predicate P (k) : 2k is even is true for all k ∈ Z+. Using the universalquantifier ∀, we can write,

∀k ∈ Z+, (2k is even).

The proposition ∀x ∈ D,P (x) is false if P (x) is false for at least one valueof x. In this case, x is called a counterexample.

Example 5.4Show that the proposition [∀x ∈ R, x > 1

x] is false.

Solution.A counterexample is x = 1

2. Clearly, 1

2< 2 = 1

12

Example 5.5Write in the form ∀x ∈ D,P (x) the proposition : “Every real number iseither positive, negative or 0.”

Page 50: Discrete Book

46 FUNDAMENTALS OF MATHEMATICAL LOGIC

Solution.∀x ∈ R, x > 0, x < 0, or x = 0

The existential quantification of the predicate P (x) is the proposition∃x ∈ D,P (x) that is true if there is at least one value of x ∈ D whereP (x) is true; otherwise it is false. The symbol ∃ is called the existentialquantifier.

Example 5.6Let P (x) denote the statement “x > 3.” What is the truth value of theproposition ∃x ∈ R, P (x).

Solution.Since 4 ∈ R and 4 > 3, the given proposition is true

The proposition ∀x ∈ D,P (x) =⇒ Q(x) is called the universal condi-tional proposition. For example, the proposition ∀x ∈ R, if x > 2 thenx2 > 4 is a universal conditional proposition.

Example 5.7Rewrite the proposition “If a real number is an integer then it is a rationalnumber” as a universal conditional proposition.

Solution.∀x ∈ R, if x is an integer then x is a rational number

Example 5.8(a) What is the negation of the proposition ∀x ∈ D,P (x)?(b) What is the negation of the proposition ∃x ∈ D,P (x)?(c) What is the negation of the proposition ∀x ∈ D,P (x) =⇒ Q(x)?

Solution.(a) ∃x ∈ D,∼ P (x). That is, there is an x ∈ D where P (x) is false.(b) ∀x ∈ D,∼ P (x). That is, P (x) is false for all x ∈ D.(c) There is an x ∈ D such that ∼ (∼ P (x) ∨Q(x)) = P (x)∧ ∼ Q(x). Thatis, P (x) is true and Q(x) is false

Example 5.9Write the negation of each of the following propositions:

Page 51: Discrete Book

5 PREDICATES AND QUANTIFIERS 47

(a) ∀x ∈ R, x > 3 =⇒ x2 > 9.(b) Every polynomial function is continuous.(c) There exists a triangle with the property that the sum of the interiorangles is different from 180◦.

Solution.(a) ∃x ∈ R, x > 3 and x2 ≤ 9.(b) There exists a polynomial that is not continuous everywhere.(c) For any triangle, the sum of the interior angles is equal to 180◦

Nested QuantifiersNext, we discuss predicates that contain multiple quantifiers. A typical ex-ample is the definition of a limit. We say that L = limx→a f(x) if and only if∀ε > 0,∃ a positive number δ such that if |x− a| ≤ δ then |f(x)− L| < ε.

Example 5.10(a) Let P (x, y) denote the statement “x + y = y + x.” What is the truthvalue of the proposition (∀x ∈ R)(∀y ∈ R), P (x, y)?(b) Let Q(x, y) denote the statement “x + y = 0.” What is the truth valueof the proposition (∃y ∈ R)(∀x ∈ R), Q(x, y)?

Solution.(a) The given proposition is always true since addition of real numbers iscommutative.(b) The proposition is false. For otherwise, one can choose x 6= −y to obtain0 6= x+ y = 0 which is impossible

Example 5.11Find the negation of the following propositions:(a) ∀x∃y, P (x, y).(b) ∃x∀y, P (x, y).

Solution.(a) ∃x∀y,∼ P (x, y).(b) ∀x∃y,∼ P (x, y)

Example 5.12The symbol ∃! stands for the phrase “there exists a unique”. Which of thefollowing statements are true and which are false.(a) ∃!x ∈ R,∀y ∈ R, xy = y.(b) ∃! integer x such that 1

xis an integer.

Page 52: Discrete Book

48 FUNDAMENTALS OF MATHEMATICAL LOGIC

Solution.(a) True. Let x = 1.(b) False since 1 and −1 are both integers with integer reciprocals

Page 53: Discrete Book

5 PREDICATES AND QUANTIFIERS 49

Review Problems

Problem 5.1By finding a counterexample, show that the proposition: “For all positiveintegers n and m,m.n ≥ m+ n” is false.

Problem 5.2Consider the statement

∃x ∈ R such that x2 = 2.

Which of the following are equivalent ways of expressing this statement?(a) The square of each real number is 2.(b) Some real numbers have square 2.(c) The real number x has square 2.(d) If x is a real number, then x2 = 2.(e) There is at least one real number whose square is 2.

Problem 5.3Rewrite the following propositions informally in at least two different wayswithout using the symbols ∃ and ∀ :(a) ∀ squares x, x is a rectangle.(b) ∃ a set A such that A has 16 subsets.

Problem 5.4Rewrite each of the following statements in the form “∃ x such that ”:(a) Some exercises have answers.(b) Some real numbers are rational numbers.

Problem 5.5Rewrite each of the following statements in the form “∀ , if then .”:(a) All COBOL programs have at least 20 lines.(b) Any valid argument with true premises has a true conclusion.(c) The sum of any two even integers is even.(d) The product of any two odd integers is odd.

Problem 5.6Which of the following is a negation for “Every polynomial function is con-tinuous”?

Page 54: Discrete Book

50 FUNDAMENTALS OF MATHEMATICAL LOGIC

(a) No polynomial function is continuous.(b) Some polynomial functions are continuous.(c) Every polynomial function fails to be continuous.(d) There is a non-continuous polynomial function.

Problem 5.7Determine whether the proposed negation is correct. If it is not, write acorrect negation.Proposition : For all integers n, if n2 is even then n is even.Proposed negation : For all integers n, if n2 is even then n is not even.

Problem 5.8Let D = {−48,−14,−8, 0, 1, 3, 16, 23, 26, 32, 36}. Determine which of the fol-lowing propositions are true and which are false. Provide counterexamplesfor those propositions that are false.(a) ∀x ∈ D, if x is odd then x > 0.(b) ∀x ∈ D, if x is less than 0 then x is even.(c) ∀x ∈ D, if x is even then x ≤ 0.(d) ∀x ∈ D, if the ones digit of x is 2, then the tens digit is 3 or 4.(e) ∀x ∈ D, if the ones digit of x is 6, then the tens digit is 1 or 2

Problem 5.9Write the negation of the proposition :∀x ∈ R, if x(x+ 1) > 0 then x > 0 orx < −1.

Problem 5.10Write the negation of the proposition : If an integer is divisible by 2, then itis even.

Problem 5.11Given the following true propostion: “∀ real numbers x, ∃ an integer n suchthat n > x.” For each x given below, find an n to make the predicate n > xtrue.(a) x = 15.83 (b) x = 108 (c) x = 101010 .

Problem 5.12Given the proposition: ∀x ∈ R,∃ a real number y such that x+ y = 0.(a) Rewrite this proposition in English without the use of the quantifiers.(b) Find the negation of the given proposition.

Page 55: Discrete Book

5 PREDICATES AND QUANTIFIERS 51

Problem 5.13Given the proposition: ∃x ∈ R,∀y ∈ R, x+ y = 0.(a) Rewrite this proposition in English without the use of the quantifiers.(b) Find the negation of the given proposition.

Problem 5.14Consider the proposition “Somebody is older than everybody.” Rewrite thisproposition in the form “∃ a person x such that ∀ .”

Problem 5.15Given the proposition: “There exists a program that gives the correct answerto every question that is posed to it.”(a) Rewrite this proposition using quantifiers and variables.(b) Find a negation for the given proposition.

Problem 5.16Given the proposition: ∀x ∈ R,∃y ∈ R such that x < y.(a) Write a proposition by interchanging the symbols ∀ and ∃.(b) State which is true: the given proposition, the one in part (a), neither,or both.

Problem 5.17Find the contrapositive, converse, and inverse of the proposition “∀x ∈ R, ifx(x+ 1) > 0 then x > 0 or x < −1.”

Problem 5.18Find the truth set of the predicate P (x) : x + 2 = 2x where the domain ofdiscourse is the set of real numbers.

Problem 5.19Let P (x) be the predicate x + 2 = 2x, where the domain of discourse is theset {1, 2, 3}. Which of the following statements are true?(i) ∀x, P (x) (ii) ∃x, P (x).

Problem 5.20Let P (x, y) be the predicate x+ y = 10 where x and y are any real numbers.Which of the following statements are true?(a) (∀x)(∃y), P (x, y).(b) (∃y)(∀x), P (x, y).

Page 56: Discrete Book

52 FUNDAMENTALS OF MATHEMATICAL LOGIC

Page 57: Discrete Book

Fundamentals of MathematicalProofs

In this chapter we discuss some common methods of proof and the standardterminology that accompanies them.

53

Page 58: Discrete Book

54 FUNDAMENTALS OF MATHEMATICAL PROOFS

6 Methods of Direct Proof

A mathematical system consists of axioms, definitions, and undefinedterms. An axiom is a statement that is assumed to be true. A definitionis used to create new concepts in terms of existing ones. A theorem is aproposition that has been proved to be true. A lemma is a theorem thatis usually not interesting in its own right but is useful in proving anothertheorem. A corollary is a theorem that follows quickly from a theorem.

Example 6.1The Euclidean geometry furnishes an example of mathematical system:• points and lines are examples of undefined terms.• An example of a definition: Two angles are supplementary if the sum oftheir measures is 180◦.• An example of an axiom: Given two distinct points, there is exactly oneline that contains them.• An example of a theorem: If two sides of a triangle are equal, then theangles opposite them are equal.• An example of a corollary: If a triangle is equilateral, then it is equiangular.

An argument that establishes the truth of a theorem is called a proof. Logicis a tool for the analysis of proofs.

First we discuss methods for proving a theorem of the form “∃x such thatP (x).” This theorem guarantees the existence of at least one x for which thepredicate P (x) is true. The proof of such a theorem is constructive: thatis, the proof is either by finding a particular x that makes P (x) true or byexhibiting an algorithm for finding x.

Example 6.2Show that there exists a positive integer whose square can be written as thesum of the squares of two positive integers.

Solution.Indeed, one example is 52 = 32 + 42

Example 6.3Show that there exists an integer x such that x2 = 15, 129.

Page 59: Discrete Book

6 METHODS OF DIRECT PROOF 55

Solution.Applying the well-known algorithm of extracting the square root we findx = 123

In contrast to constructive existence proofs, a nonconstructive existenceproof uses known results to imply the existence of an x such that P (x) istrue without the need of knowing the actual value of x. We illustrate thismethod in the next example.

Example 6.4In Calculus, the Intermediate Value Theorem states that if a function f iscontinuous in a closed interval [a, b] and f(a) < c < f(b) then the equationf(x) = c has at least one solution in the open interval (a, b). Use the IVT toshow that the equation x5 + 2x3 + x − 5 = 0 has a solution in the interval(1, 2).

Solution.Let f(x) = x5 + 2x3 + x − 5. Note that f(1) = −1 and f(2) = 45 so thatf(1) < 0 < f(2) < 0 Since f is continuous in [1, 2], by the IntermediateValue Theorem, there is a number 1 < c < 2 such that f(c) = 0. That is,x5 + 2x3 + x− 5 = 0 has a solution in the interval (1, 2)

Next, we consider theorems of the form “∀x ∈ D, P (x).” If D is a finiteset, then one checks the truth value of P (x) for each x ∈ D. This method iscalled the method of exhaustion.

Example 6.5Show that for each positive integer 1 ≤ n ≤ 10, n2−n+11 is a prime number.

Solution.In this example,

D = {1, 2, 3, 4, 5, 6, 7, 8, 9, 10}

and P (n) = n2− n+ 11 is a prime number. Using the method of exhaustionwe see that

P (1) = 11 ; P (2) = 13 ; P (3) = 17 ; P (4) = 23P (5) = 31 ; P (6) = 41 ; P (7) = 53 ; P (8) = 67P (9) = 83 ; P (10) = 101

Page 60: Discrete Book

56 FUNDAMENTALS OF MATHEMATICAL PROOFS

The proposition “∀x ∈ D, P (x)” can be written in the form “∀x if x ∈ Dthen P (x).” Thus, we consider propositions of the form “∀x ∈ D if P (x) thenQ(x).” We call P (x) the hypothesis and Q(x) the conclusion.

By a direct method of proof we mean a method that consists of showingthat if P (x) is true for x ∈ D then Q(x) is also true.

The following shows the format of the direct proof of a theorem.

Theorem 6.1For all n,m ∈ Z, if m and n are even then so is m+ n.

Proof.Let m and n be two even integers. Then there exist positive integers k1 andk2 such that n = 2k1 and m = 2k2. We must show that m + n is even, thatis, an integer multiple of 2. Indeed,

m+ n = 2k1 + 2k2 = 2(k1 + k2)2k

where k = k1 + k2 ∈ Z. Thus, by the definition of even, m+ n is even

Example 6.6Prove the following theorem.

Theorem Every integer is a rational number. That is, for all n, if n ∈ Zthen n ∈ Q.

Solution.Proof. Let n be an arbitrary integer. Then n = n

1. By the definition of

rational numbers, n is rational

Example 6.7Prove the following theorem.

Theorem If a, b ∈ Q then a+ b ∈ Q.

Solution.Proof. Let a and b be two rational numbers. Then there exist integers

Page 61: Discrete Book

6 METHODS OF DIRECT PROOF 57

a1, a2, b1 6= 0, and b2 6= 0 such that a = a1b1

and b = a2b2. By the property of

addition of two fractions we have

a+ b =a1b1

+a2b2

=a1b2 + a2b1

b1b2.

By letting p = a1b2 + a2b1 ∈ Z and q = b1b2 ∈ Z∗ we get a+ b = pq. That is,

a+ b ∈ Q

Example 6.8Prove the following corollary.

Corollary The double of a rational number is rational.

Solution.Proof. Let a = b in the previous theorem we see that 2a = a+a = a+b ∈ Q

Common mistakes when writing a mathematical proofNext, we point out of some common mistakes that must be avoided in prov-ing theorems.

• Arguing from examples. The validity of a general statement can not beproved by just using a particular example.

• Using the same letters to mean two different things. For example, sup-pose that m and n are any two given even integers. Then by writing m = 2kand n = 2k this would imply that m = n which is inconsistent with thestatement that m and n are arbitrary.

• Jumping to a conclusion. Let us illustrate by an example. Suppose thatwe want to show that if the sum of two integers is even so is their difference.Consider the following proof: Suppose that m+ n is even. Then there is aninteger k such that m+ n = 2k. Then, m = 2k − n and so m− n is even.The problem with this proof is that the crucial step m− n = 2k − n− n =2(k − n) is missing. The author of the proof has jumped prematurely to aconclusion.

Page 62: Discrete Book

58 FUNDAMENTALS OF MATHEMATICAL PROOFS

• Begging the question. By that we mean that the author of a proof uses inhis argument a fact that he is supposed to prove.

Finally, to show that a proposition of the form ∀x ∈ D, if P (x) then Q(x)is false it suffices to find an element x ∈ D where P (x) is true but Q(x) isfalse. Such an x is called a counterexample.

Example 6.9Disprove the proposition ∀a, b ∈ R, if a < b then a2 < b2.

Solution.A counterexample is the following. Let a = −2 and b = −1. Then a < b buta2 > b2

Page 63: Discrete Book

6 METHODS OF DIRECT PROOF 59

Review ProblemsA real number r is called rational if there exist two integers a and b 6= 0such that r = a

b. A real number that is not rational is called irrational.

Problem 6.1Show that there exist integers m and n such that 3m+ 4n = 25.

Problem 6.2Show that there is a positive integer x such that x4 + 2x3 + x2− 2x− 2 = 0.Hint: Use the rational zero test.

Problem 6.3Show that there exist two prime numbers whose product is 143.

Problem 6.4Show that there exists a point in the Cartesian system that is not on the liney = 2x− 3.

Problem 6.5Using a constructive proof, show that the number r = 6.321521521... is arational number.

Problem 6.6Show that the equation x3 − 3x2 + 2x − 4 = 0 has at least one solution inthe interval (2, 3).

Problem 6.7Use a non-constructive proof to show that there exist irrational numbers a

and b such that ab is rational. Hint: Look at the number q =√

2√2. Consider

the cases q is rational or q is irrational.

Problem 6.8Use the method of exhaustion to show: ∀n ∈ N, if n is even and 4 ≤ n ≤ 21then n can be written as the sum of two prime numbers.

Problem 6.9Use the method of exhaustion to show: ∀n ∈ N, if n is prime and n < 7 then2n+ 1 is prime.

Page 64: Discrete Book

60 FUNDAMENTALS OF MATHEMATICAL PROOFS

Problem 6.10Prove the following theorem.

Theorem. The product of two rational numbers is a rational number.

Problem 6.11Use the previous problem to prove the following.

Corollary. The square of any rational number is rational.

Problem 6.12Use the method of constructive proof to show that if r and s are two realnumbers then there exists a real number x such that r < x < s.

Problem 6.13Disprove the following statement by finding a counterexample: ∀x, y, z ∈ R,if x > y then xz > yz.

Problem 6.14Disprove the following statement by finding a counterexample: ∀x ∈ R, ifx > 0 then 1

x+2= 1

x+ 1

2.

Problem 6.15Show that for any even integer n, we have (−1)n = 1.

Problem 6.16Show that the product of two odd integers is odd.

Problem 6.17Identify the error in the following proof:“For all positive integer n, the prod-uct (n− 1)n(n+ 2) is divisible by 3 since 3(4)(5) is divisible by 3.”

Problem 6.18Identify the error in the following proof:“ If n amd m are two different oddintegers then there exist integer k such that n = 2k + 1 and m = 2k + 1.”

Problem 6.19Identify the error in the following proof:“Suppose that m and n are integerssuch that n + m is even. We want to show that n −m is even. For n + mto be even, n and m must be even. Hence, n = 2k1 and m = 2k2 so thatn−m = 2(k1 − k2) which is even.”

Page 65: Discrete Book

6 METHODS OF DIRECT PROOF 61

Problem 6.20Identify the error in proving that if a product of two integers x and y isdivisible by 5 then x is divisible by 5 or y is divisible by 5:“ Since xy isdivisible by 5, there is an integer k such that xy = 5k. Hence, x = 5k1 forsome k1 or y = 5k2 for some k2. Thus, either x is divisible by 5 or y is divisibleby 5.”

Page 66: Discrete Book

62 FUNDAMENTALS OF MATHEMATICAL PROOFS

7 More Methods of Proof

A vacuous proof is a proof of an implication p → q in which it is shownthat p is false. Thus, if p is false then the implication is true regardless ofthe truth value of q.

Example 7.1Use the method of vacuous proof to show that if x ∈ ∅ then 2 is an oddnumber.

Solution.Since the proposition x ∈ ∅ is always false, the given proposition is vacuouslytrue

Example 7.2Show that if 4 is a prime number then −4 = 4.

Solution.The hypothesis is false, therefore the implication is vacuously true (eventhough the conclusion is also false)

A trivial proof of an implication p → q is one in which q is shown tobe true without any reference to p.

Example 7.3Use the method of trivial proof to show that if n is an even integer then n isdivisible by 1.

Solution.Since the proposition n is divisible by 1 is always true, the given implicationis trivially true. Notice that the hypothesis n is an even integer did not playa role in the proof

The method of proof by cases is a direct method of proving the condi-tional proposition p1 ∨ p2 ∨ · · · ∨ pn → q. The method consists of proving theconditional propositions p1 → q, p2 → q, · · · , pn → q.

Example 7.4Show that if n is a positive integer then n3 + n is even.

Page 67: Discrete Book

7 MORE METHODS OF PROOF 63

Solution.We use the method of proof by cases.

Case 1. Suppose that n is even. Then there is k ∈ N such that n = 2k.In this case, n3 + n = 8k3 + 2k = 2(4k3 + k) which is even.

Case 2. Suppose that n is odd. Then there is a k ∈ N such that n = 2k + 1.So, n3 + n = 2(4k3 + 6k2 + 4k + 1) which is even

Example 7.5Use the proof by cases to prove the triangle inequality: |x+ y| ≤ |x|+ |y|.

Solution.Case 1. x ≥ 0 and y ≥ 0. Then x+ y ≥ 0 and so |x+ y| = x+ y = |x|+ |y|.Case 2. x ≥ 0 and y < 0. Then x + y < x + 0 = |x| ≤ |x| + |y|. On theother hand, −(x + y) = −x + (−y) ≤ 0 + (−y) = |y| ≤ |x| + |y|. Thus,if |x + y| = x + y then |x + y| < |x| + |y| and if |x + y| = −(x + y) then|x+ y| ≤ |x|+ |y|.Case 3. The case x < 0 and y ≥ 0 is similar to case 2.Case 4. Suppose x < 0 and y < 0. Then x + y < 0 and therefore |x + y| =−(x+ y) = (−x) + (−y) = |x|+ |y|.So in all four cases |x+ y| ≤ |x|+ |y|

Now, given a real number x, the largest integer n such that n ≤ x < n + 1is called the floor of x and is denoted by bxc. The smallest integer n suchthat n− 1 < x ≤ n is called the ceiling of x and is denoted by dxe.

Example 7.6Compute bxc and dxe of the following values of x :(a) 37.999 (b) −57

2(c) −14.001

Solution.(a) b37.999c = 37, d37.999e = 38.(b) b−57

2c = −29, d−57

2e = −28.

(c) b−14.001c = −15, d−14.001e = −14

Example 7.7Use the proof by a counterexample to show that the proposition “∀x, y ∈R, bx+ yc = bxc+ byc” is false.

Page 68: Discrete Book

64 FUNDAMENTALS OF MATHEMATICAL PROOFS

Solution.Let x = y = 0.5. Then bx+ yc = 1 and bxc+ byc = 0

The following gives another example of the method of proof by cases.

Theorem 7.1For any integer n,

bn2c =

{n2, if n is even

n−12, if n is odd

Proof.Let n be any integer. Then we consider the following two cases.

Case 1. n is odd. In this case, there is an integer k such that n = 2k + 1.Hence,

bn2c = b2k + 1

2c = bk +

1

2c = k

since k ≤ k+ 12< k+ 1. Since n = 2k+ 1, solving this equation for k we find

k = n−12. It follows that

bn2c = k =

n− 1

2.

Case 2. Suppose n is even. Then there is an integer k such that n = 2k.Hence, bn

2c = bkc = k = n

2

Proof of Many Equivalent StatementsIn future math courses you will sometimes encounter a certain kind of theo-rem that is neither a conditional nor a biconditional statement. Instead, itasserts that a list of statements is “equivalent.”To say that statements p1, p2, · · · , pn are all equivalent means that eitherthey are all true or all false. To prove that they are equivalent, one assumesp1 to be true and proves that p2 is true, then assumes p2 to be true andproves that p3 is true, continuing in this fashion, assume that pn−1 is trueand prove that pn is true and finally, assume that pn is true and prove thatp1 is true. This is known as the proof by circular argument.

Example 7.8Show that all the following statements are equivalent:(i) ad = bc.

Page 69: Discrete Book

7 MORE METHODS OF PROOF 65

(ii) ab

= cd.

(iii) a+bb

= c+dd.

(iv) ac

= bd

where a, b, c, and d are non-zero numbers.

Solution.(i) =⇒ (ii): Suppose ad = bc. Divide both sides by bd to obtain a

b= c

d.

(ii) =⇒ (iii): Suppose ab

= cd

= m. Add 1 to both sides to obtain ab+1 = c

d+1.

Hence, a+bb

= c+dd.

(iii) =⇒ (iv): Suppose a+bb

= c+dd

= m Then ab

+ 1 = cd

+ 1. Subtract 1 fromboth sides to obtain a

b= c

d= m. Hence, a = bm and c = dm or ad = dbm

and bc = bdm. That is, ad = bc. Divide both sides by dc we obtain ac

= bd.

(iv) =⇒ (i): Suppose that ac

= bd

= m. Then a = cm and ad = dcm. Likewise,b = dm and bc = dcm. It follows that ad = bc

Page 70: Discrete Book

66 FUNDAMENTALS OF MATHEMATICAL PROOFS

Review Problems

Problem 7.1Using the method of vacuous proof, show that if n is a positive integer andn = −n then n2 = n.

Problem 7.2Show that for all x ∈ R, x2 + 1 < 0 implies x2 ≥ 4.

Problem 7.3Let x ∈ R. Show that if x2 − 2x+ 5 < 0 then x2 + 1 < 0.

Problem 7.4Use the method of trivial proof to show that if x ∈ R and x > 0 thenx2 + 1 > 0.

Problem 7.5Show that if 6 is a prime number then 62 = 36.

Problem 7.6Use the method of proof by cases to prove that for any integer n the productn(n+ 1) is even.

Problem 7.7Use the method of proof by cases to prove that the square of any integer hasthe form 4k or 4k + 1 for some integer k

Problem 7.8Use the method of proof by cases to to prove that for any integer n, n(n2 −1)(n+ 2) is divisible by 4.

Problem 7.9State a necessary and sufficient condition for the floor function of a realnumber to equal that number

Problem 7.10Prove that if n is an even integer then bn

2c = n

2.

Problem 7.11Show that the equality bx− yc = bxc − byc is not valid for all real numbersx and y.

Page 71: Discrete Book

7 MORE METHODS OF PROOF 67

Problem 7.12Show that the equality dx+ ye = dxe+ dye is not valid for all real numbersx and y.

Problem 7.13Prove that for all real numbers x and all integers m, dx+me = dxe+m.

Problem 7.14Show that if n is an odd integer then dn

2e = n+1

2.

Problem 7.15Prove that all of the following statements are equivalent:(i) x2 − 4 = 0.(ii) (x− 2)(x+ 2) = 0.(iii) x = 2 or x = −2.

Problem 7.16Let x and y be two real numbers. Show that all the following are equivalent:(a) x < y (b) x+y

2> x (c) x+y

2< y.

Problem 7.17Show that all the following are equivalent:(a) x2 − 5x+ 6 = 0.(b) (x− 2)(x− 3) = 0.(c) x− 2 = 0 or x− 3 = 0.(d) x = 2 or x = 3.

Page 72: Discrete Book

68 FUNDAMENTALS OF MATHEMATICAL PROOFS

8 Methods of Indirect Proofs: Contradiction

and Contraposition

Recall that in a direct proof one starts with the hypothesis of an implicationp → q and then proves that the conclusion is true. Any other method ofproof will be referred to as an indirect proof. In this section we study twomethods of indirect proofs, namely, the proof by contradiction and the proofby contrapositive.

Proof by contradictionWe want to show that q is true. Instead, we assume it is not and derive thatthe proposition p∧ ∼ p is true. But p ∧ p is a contradiction which is alwaysfalse (See Example 1.12). Hence, we conclude that q must be true.

Theorem 8.1If n2 is an even integer so is n.

Proof.Suppose the contrary. That is suppose that n is odd. Then there is aninteger k such that n = 2k + 1. In this case, n2 = 2(2k2 + 2k) + 1 is oddand this contradicts the assumption that n2 is even. Hence, n must be even

The method of proof by contradiction is not limited to just proving con-ditional propositions of the form p→ q, it can be used to prove any kind ofstatement whatsoever. We illustrate this point in the next example.

Theorem 8.2The number

√2 is irrational.

Proof.Suppose not. That is, suppose that

√2 is rational. Then there exist two

integers m and n with no common divisors such that√

2 = mn. Squaring

both sides of this equality we find that 2n2 = m2. Thus, m2 is even. ByTheorem 8.1, m is even. That is, 2 divides m. But then m = 2k for someinteger k. Taking the square we find that 2n2 = m2 = 4k2, that is n2 = 2k2.This says that n2 is even and by Theorem 8.1, n is even. We conclude that2 divides both m and n and this contradicts our assumption that m and nhave no common divisors. Hence,

√2 must be irrational

Page 73: Discrete Book

8 METHODS OF INDIRECT PROOFS: CONTRADICTION AND CONTRAPOSITION69

Example 8.1Use the proof by contradiction to show that there are no positive integers nand m such that n2 −m2 = 1.

Proof.Suppose the contrary. Then (n − m)(n + m) = 1 implies the system ofequation {

n−m = 1n+m = 1.

Solving this system we find n = 1 and m = 0, contradicting our assumptionthat n and m are positive integers

Proof by contrapositiveWe already know that p→ q ≡∼ q →∼ p. So to prove p→ q we sometimesinstead prove ∼ q →∼ p.

Theorem 8.3If n is an integer such that n2 is odd then n is also odd.

Proof.Suppose that n is an integer that is even. Then there exists an integer k suchthat n = 2k. But then n2 = 2(2k2) which is even

Remark 8.1How is the proof by contrapositive different from the proof by contrapositive?Let’s examine how the two methods work when trying to prove p → q.Using the method by contradiction, we assume that p and ∼ q and derive acontradiction. Using the method of contrapositive, we assume ∼ q and prove∼ p. Hence, the method of contrapositive has the advantage that your goalis clear: Prove ∼ p. In the method of contradiction, your goal is to prove acontradiction, but it is not always clear what the contradiction is going tobe at the start.

Example 8.2Prove by the method of contrapositive: If a and b are two integers such thata · b is even then at least one the two must be even.

Page 74: Discrete Book

70 FUNDAMENTALS OF MATHEMATICAL PROOFS

Solution.Suppose that a and b are both odd. Then there exist integers m and nsuch that a = 2m + 1 and b = 2n + 1. Thus, a · b = (2m + 1)(2n + 1) =2(2mn + m + n) + 1 = 2k + 1 where k = 2mn + m + n ∈ Z. Hence, a · b isodd

Example 8.3 (Perfect squares test)Prove by the method of contrapositive: If n is a positive integer such thatthe remainder of the division of n by 4 is either 2 or 3, then n is not a perfectsquare.

Solution.Suppose that n is a perfect square. Then n = k2 for some k ∈ Z.• If the remainder of the division of k by 4 is 0 then k = 4m1 and n =16m2

1 = 4(4m21) so that the remainder of the division of n by 4 is 0.

• If the remainder of the division of k by 4 is 1 then k = 4m2 + 1 andn = 4(4m2

2 + 2m2) + 1 so that the remainder of the division of n by 4 is 1.• If the remainder of the division of k by 4 is 2 then k = 4m3 + 2 andn = 4(4m2

3 + 4m3 + 1) + 1 so that the remainder of the division of n by 4 is1.• If the remainder of the division of k by 4 is 3 then k = 4m4 + 3 andn = 4(4m2

4 + 6m4 + 2) + 1 so that the remainder of the division of n by 4 is0

Page 75: Discrete Book

8 METHODS OF INDIRECT PROOFS: CONTRADICTION AND CONTRAPOSITION71

Review Problems

Problem 8.1Use the proof by contradiction to prove the proposition “There is no greatesteven integer.”

Problem 8.2Prove by contradiction that the difference of any rational number and anyirrational number is irrational.

Problem 8.3Use the proof by contraposition to show that if a product of two positive realnumbers is greater than 100, then at least one of the numbers is greater than10.

Problem 8.4Use the proof by contradiction to show that the product of any nonzerorational number and any irrational number is irrational.

Problem 8.5Show that if n is an integer and n2 is divisible by 3 then n is also divisibleby 3.

Problem 8.6Show that the number

√3 is irrational.

Problem 8.7Use the proof by contrapositive to show that if n and m are two integers forwhich n+m is even then either both n and m are even or both are odd.

Problem 8.8Use the proof by contrapositive to show that for any integer n, if 3n + 1 iseven then n is odd.

Problem 8.9Use the proof by contradiction to show that there are no positive integers nand m such that n2 −m2 = 2.

Problem 8.10Use the proof by contradiction to show that for any integers n and m, wehave n2 − 4m 6= 2.

Page 76: Discrete Book

72 FUNDAMENTALS OF MATHEMATICAL PROOFS

Problem 8.11Prove by contrapositive: If n is a positive integer such that the remainder ofthe division of n by 3 is 2 then n is not a perfect square.

Problem 8.12Prove by contrapositive: Suppose that a, b ∈ Z. If a · b is not divisible by 5then both a and b are not divisible by 5.

Page 77: Discrete Book

9 METHOD OF PROOF BY INDUCTION 73

9 Method of Proof by Induction

With the emphasis on structured programming has come the development ofan area called program verification, which means your program is correctas you are writing it.One technique essential to program verification is mathematical induc-tion, a method of proof that has been useful in every area of mathematicsas well.Consider an arbitrary loop in Pascal starting with the statement

FOR I := 1 TO N DO

If you want to verify that the loop does something regardless of the particularintegral value of N, you need mathematical induction.Also, sums of the form

n∑k=1

k =n(n+ 1)

2

are very useful in analysis of algorithms and a proof of this formula is math-ematical induction.Next we examine this method. We want to prove that a predicate P (n) istrue for any nonnegative integer n ≥ n0. The steps of mathematical induc-tion are as follows:(i) (Basis of induction) Show that P (n0) is true.(ii) (Induction hypothesis) Assume P (n) is true.(iii) (Induction step) Show that P (n+ 1) is true.

Example 9.1Use the technique of mathematical induction to show that

1 + 2 + 3 + · · ·+ n =n(n+ 1)

2, n ≥ 1.

Solution.Let P (n) : 1 + 2 + · · ·+ n = n(n+1)

2. Then

(i) (Basis of induction) P (1) : 1 = 1(1+1)2

. That is, P (1)is true.(ii) (Induction hypothesis) Assume P (n) is true. That is, P (n) : 1 + 2 + 3 +

· · ·+ n = n(n+1)2

.

Page 78: Discrete Book

74 FUNDAMENTALS OF MATHEMATICAL PROOFS

(iii) (Induction step) We must show that P (n+ 1) : 1 + 2 + 3 + · · ·+n+ 1 =(n+1)(n+2)

2. Indeed,

1 + 2 + · · ·+ n+ (n+ 1) =(1 + 2 + · · ·+ n) + n+ 1

=n(n+ 1)

2+ (n+ 1)

=(n+ 1)(n+ 2)

2

Example 9.2 (Geometric progression)

(a) Use induction to show P (n) :∑n

k=0 ark = a(1−rn+1)

1−r , n ≥ 0 where r 6= 1.

(b) Show that 1 + 12

+ · · ·+ 12n−1 ≤ 2, for all n ≥ 1.

Solution.(a) We use the method of proof by mathematical induction.(i) (Basis of induction) a = a1−r0+1

1−r =∑0

k=0 ark. That is, P (0)is true.

(ii) (Induction hypothesis) Assume P (n) is true. That is,∑n

k=0 ark =

a(1−rn+1)1−r .

(iii) (Induction step) We must show that P (n+1) is true. That is,∑n+1

k=0 ark =

a(1−rn+2)1−r . Indeed,

n+1∑k=0

ark =n∑

k=0

ark + arn+1

=a1− rn+1

1− r+ arn+11− r

1− r

=a1− rn+1 + rn+1 − rn+2

1− r

=a1− rn+2

1− r.

(b) By (a) we have

1 +1

2+

1

22+ · · ·+ 1

2n−1 =1− (1

2)n

1− 12

=2(1− (1

2)n)

=2− 1

2n−1

≤2

Page 79: Discrete Book

9 METHOD OF PROOF BY INDUCTION 75

Example 9.3 (Arithmetic progression)Use induction to show that P (n) :

∑nk=1(a+(k−1)r) = n

2[2a+(n−1)r], n ≥

1.

Solution.We use the method of proof by mathematical induction.(i) (Basis of induction) a = 1

2[2a + (1− 1)r] =

∑1k=1(a + (k − 1)r). That is,

P (1)is true.(ii) (Induction hypothesis) Assume P (n) is true. That is,

∑nk=1(a+(k−1)r) =

n2[2a+ (n− 1)r].

(iii) (Induction step) We must show that P (n+1) is true. That is,∑n+1

k=1(a+

(k − 1)r) = (n+1)2

[2a+ nr]. Indeed,

n+1∑k=1

(a+ (k − 1)r) =n∑

k=1

(a+ (k − 1)r) + a+ (n+ 1− 1)r

=n

2[2a+ (n− 1)r] + a+ nr

=2an+ n2r − nr + 2a+ 2nr

2

=2a(n+ 1) + n(n+ 1)r

2

=n+ 1

2[2a+ nr]

We next exhibit a theorem whose proof uses mathematical induction.

Theorem 9.1For all integers n ≥ 1, 22n − 1 is divisible by 3.

Proof.Let P (n) : 22n − 1 is divisible by 3. Then(i) (Basis of induction) P (1) is true since 3 is divisible by 3.(ii) (Induction hypothesis) Assume P (n) is true. That is, 22n − 1 is divisibleby 3.

Page 80: Discrete Book

76 FUNDAMENTALS OF MATHEMATICAL PROOFS

(iii) (Induction step) We must show that 22n+2 − 1 is divisible by 3. Indeed,

22n+2 − 1 =22n(4)− 1

=22n(3 + 1)− 1

=22n · 3 + (22n − 1)

=22n · 3 + P (n)

=3(22n + k)

where we used the induction hypothesis that 3|(22n−1). That is, 22n−1 = 3kfor some integer k. Since 22n +k is an integer, we conclude that 3|(22n+1−1

Example 9.4(a) Use induction to prove that n < 2n for all non-negative integers n.(b) Use induction to prove that 2n < n! for all non-negative integers n ≥ 4.

Solution.(a) Let P (n) : n < 2n We want to show that P (n) is valid for all n ≥ 0. Bythe method of mathematical induction we have(i) (Basis of induction) We have 0 < 20 = 1. Thus, P (0)is true.(ii) (Induction hypothesis) Assume P (n) is true. That is, n < 2n.(iii) (Induction step) We must show that P (n + 1) is also true. That is,n+ 1 < 2n+1. Indeed,

n+ 1 <2n + 1

≤2n + n

<2n + 2n

=2 · 2n = 2n+1

where we used the fact that n < 2n twice.(b) Let P (n) : 2n < n!. We want to show that P (n) is valid for all n ≥ 4. Bythe method of mathematical induction we have(i) (Basis of induction) 16 = 24 < 24 = 4!. That is, P (4)is true.(ii) (Induction hypothesis) Assume P (n) is true. That is, 2n < n!, n ≥ 4.(iii) (Induction step) We must show that P (n + 1) is true. That is, 2n+1 <(n+ 1)!. Indeed,

2n+1 =2n + 2n

<2n!

<(n+ 1)n! = (n+ 1)!

Page 81: Discrete Book

9 METHOD OF PROOF BY INDUCTION 77

where we have used the fact that if n+ 1 ≥ 5 > 2

Example 9.5 (Bernoulli’s inequality)Let h > −1. Use induction to show that

(1 + nh) ≤ (1 + h)n, n ≥ 0.

Solution.Let P (n) : (1 + nh) ≤ (1 + h)n. We want to show that P (n) is valid for allnonnegative integers.(i) (Basis of induction) 1 + 0h = 1 ≤ 1 = (1 + h)0. That is, P (0)is true.(ii) (Induction hypothesis) Assume P (n) is true. That is, (1+nh) ≤ (1+h)n.(iii) (Induction step) We must show that P (n+ 1) is true. That is, (1 + (n+1)h) ≤ (1 + h)n+1. Indeed,

(1 + nh) ≤(1 + h)n

(1 + h)(1 + nh) ≤(1 + h)n+1

1 + (n+ 1)h+ nh2 ≤(1 + h)n+1.

But nh2 ≥ 0 so that 1 + (n+ 1)h ≤ 1 + (n+ 1)h+ nh2 ≤ (1 + h)n+1

Example 9.6Define the following sequence of numbers: a1 = 2 and for n ≥ 2, an = 5an−1.Find a formula for an and then prove its validity by mathematical induction.

Solution.Listing the first few terms we find, a1 = 2, a2 = 5a1 = 5(2), a3 = 5a2 =52(2), a4 = 5a3 = 53(2). Thus, an = 2(5n−1). We will show that P (n) : an =2(5n−1) is valid for all n ≥ 1 by the method of mathematical induction.(i) (Basis of induction) a1 = 2 = 2(51−1). That is, P (1) is true.(ii) (Induction hypothesis) Assume P (n) is true. That is, an = 2(5n−1).(iii) (Induction step) We must show that an+1 = 2(5n). Indeed,

an+1 =5an

=5[2(5n−1)]

=2(5n)

Page 82: Discrete Book

78 FUNDAMENTALS OF MATHEMATICAL PROOFS

Review Problems

Problem 9.1Use the method of induction to show that

2 + 4 + 6 + · · ·+ 2n = n2 + n

for all integers n ≥ 1.

Problem 9.2Use mathematical induction to prove that

1 + 2 + 22 + · · ·+ 2n = 2n+1 − 1

for all integers n ≥ 0.

Problem 9.3Use mathematical induction to show that

12 + 22 + · · ·+ n2 =n(n+ 1)(2n+ 1)

6

for all integers n ≥ 1.

Problem 9.4Use mathematical induction to show that

13 + 23 + · · ·+ n3 =

(n(n+ 1)

2

)2

for all integers n ≥ 1.

Problem 9.5Use mathematical induction to show that

1

1 · 2+

1

2 · 3+ · · ·+ 1

n(n+ 1)=

n

n+ 1

for all integers n ≥ 1.

Page 83: Discrete Book

9 METHOD OF PROOF BY INDUCTION 79

Problem 9.6Use the formula

1 + 2 + · · ·+ n =n(n+ 1)

2to find the value of the sum

3 + 4 + · · ·+ 1, 000.

Problem 9.7Find the value of the geometric sum

1 +1

2+

1

22+ · · ·+ 1

2n.

Problem 9.8Let S(n) =

∑nk=1

k(k+1)!

. Evaluate S(1), S(2), S(3), S(4), and S(5). Make aconjecture about a formula for this sum for general n, and prove your con-jecture by mathematical induction.

Problem 9.9For each positive integer n let P (n) be the proposition 4n − 1 is divisible by3.(a) Write P (1). Is P (1) true?(b) Write P (k).(c) Write P (k + 1).(d) In a proof by mathematical induction that this divisibility property holdsfor all integers n ≥ 1, what must be shown in the induction step?

Problem 9.10For each positive integer n let P (n) be the proposition 23n− 1 is divisible by7. Prove this property by mathematical induction.

Problem 9.11Show that 2n < (n+ 2)! for all integers n ≥ 0.

Problem 9.12(a) Use mathematical induction to show that n3 > 2n + 1 for all integersn ≥ 2.(b) Use mathematical induction to show that n! > n2 for all integers n ≥ 4.

Problem 9.13A sequence a1, a2, · · · is defined recursively by a1 = 3 and an = 7an−1 forn ≥ 2. Show that an = 3 · 7n−1 for all integers n ≥ 1.

Page 84: Discrete Book

80 FUNDAMENTALS OF MATHEMATICAL PROOFS

Page 85: Discrete Book

Number Theory andMathematical Proofs

In this chapter, we look at the applications of mathematical proofs to numbertheory.

81

Page 86: Discrete Book

82 NUMBER THEORY AND MATHEMATICAL PROOFS

10 Divisibility. The Division Algorithm

In this section we study the divisibility of integers. Our main goal is toobtain the Division Algorithm. This is achieved by applying the well-ordering principle which we prove next.

Theorem 10.1 (The Well-Ordering Principle)If S is a nonempty subset of N then there is m ∈ S such that m ≤ x for allx ∈ S. That is, S has a smallest or least element.

Proof.We will use contraposition to prove the theorem. That is, by assuming thatS has no smallest element we will prove that S = ∅.We will prove that n 6∈ S for all n ∈ N. We do this by induction on n. SinceS has no smallest element, we have 1 6∈ S. Asuume that we have proved that1, 2, · · · , n 6∈ S. We will show that n+ 1 6∈ S. If n+ 1 ∈ S then n+ 1 wouldbe the smallest element of S since 1, 2, 3, · · · , n 6∈ S, and this contradicts theassumption that S has no smallest element. Thus, we must have n+ 1 6∈ S.Hence, by the principle of mathematical induction, n 6∈ S for all n ∈ N. Butthis leads to S = ∅. This establishes a proof of the theorem

Remark 10.1The above theorem is false if N is replaced by Z,Q, or R. For example, theset of even integers is nonempty subset of Z with no smallest element.

Example 10.1Prove that there is no positive integer between 0 and 1.

Solution.Suppose the contrary. Let n ∈ N such that 0 < n < 1. Define the setS = {a ∈ N : 0 < a < 1}. Since n ∈ S, S is non-empty. By Theorem 10.1,S has a smallest element b where b ∈ N and 0 < b < 1. Multiply this lastinequality by b to obtain 0 < b2 < b < 1. But then b2 ∈ S and b2 < b whichcontradicts the fact that b is the smallest element of S. By the method ofproof by contradiction, we conclude that there is no positive integer between0 and 1

With the Well-Ordering Principle we can establish the following theorem.

Page 87: Discrete Book

10 DIVISIBILITY. THE DIVISION ALGORITHM 83

Theorem 10.2 (Division Algorithm)If a and b are integers with b ≥ 1 then there exist unique integers q and rsuch that

a = bq + r, 0 ≤ r < b.

Proof.The proof consists of two parts: existence and uniqueness.

ExistenceConsider the sets

S = {a− bt : t ∈ Z}, S ′ = {x ∈ S : x ≥ 0}.

The set S ′ is nonempty. To see this, if a ≥ 0 then a− 0t ∈ S and a− 0t ≥ 0.That is, a ∈ S ′. If a < 0 then since a− ba ∈ S and a− ba = a(1− b) ≥ 0 sothat a− ba ∈ S ′.Now, if 0 ∈ S ′ then a− qb = 0 for some q ∈ Z and so r = 0 and in this casethe theorem holds. So, assume that 0 6∈ S ′. By Theorem 10.1, there exists asmallest element r ∈ S ′. That is,

a− qb = r, for some q ∈ Z.

Since r ∈ S ′, we have r ≥ 0. It remains to show that r < b. If we assume thecontrary, i.e., r ≥ b, then

a− b(q + 1) = a− bq − b = r − b ≥ 0

and this implies that a− b(q + 1) ∈ S ′. But b ≥ 1 > 0 so that

a− b(q + 1) = a− bq − b < a− bq = r

and this contradicts the definition of r as being the smallest element of S ′.Thus, for the given a and b we can find integers q and r such that

a = bq + r, 0 ≤ r < b.

UniquenessSuppose that

a = bq1 + r1, 0 ≤ r1 < b

anda = bq2 + r2, 0 ≤ r2 < b.

Page 88: Discrete Book

84 NUMBER THEORY AND MATHEMATICAL PROOFS

We must show that r1 = r2 and q1 = q2. Indeed, bq1 + r1 = bq2 + r2 impliesb(q1− q2) = r2− r1. This says that b|(r2− r1). But 0 ≤ r1 < b and 0 ≤ r2 < bso that −b < −r1 < r2 − r1 < r2 < b. That is, −b < r2 − r1 < b. Hence,−b < b(q2−q1) < b which by dividing through by b we find −1 < q2−q1 < 1.Now, q2 − q1 is an integer strictly between −1 and 1. Thus, q2 − q1 = 0 orq1 = q2. But then r2 − r1 = b(q1 − q2) = 0 and this implies r1 = r2

Example 10.2Find q and r when a = 11 and b = 4.

SolutionBy long division of numbers, we find q = 2 and r = 3

Next, we introduce the concept of divisibility and derive some of its proper-ties.An integer m is divisible by a nonzero integer n if and only if m = nq forsome integer q. We also say that n divides m, n is a divisor of m, m is amultiple of n, or n is a factor of m. We write n|m. If n does not divide mwe write n 6 | m. For example, 2| 8 and 4| 8. However, 4 6 | 6.

The following theorem discusses some of the properties of divisibility.

Theorem 10.3(a) If n|m then n|(−m).(b) If n|a and n|b then n|(a± b)(c) If n|m and m|p then n|p.(d) If n|m and m|n then either n = m or n = −m.

A positive integer n > 1 with only divisors 1 and n is called prime. Aninteger greater than 1 that is not prime is called composite. That is, acomposite number is a positive integer that has at least one positive divisorother than one or the number itself. For example, 2 is the first prime numberwhereas 4 is first composite number.

If a positive number p is composite then one can always write p as theproduct of primes, where the prime factors are written in increasing order.This result is known as the Fundamental Theorem of Arithmetic or theUnique Factorization Theorem.

Page 89: Discrete Book

10 DIVISIBILITY. THE DIVISION ALGORITHM 85

Theorem 10.4Every positive integer n > 1 is either a prime or can be written as a productof prime integers, and this product is unique except for the order of thefactors.

Example 10.3Write the prime factorization of 180.

Solution.The prime factorization is 180 = 22 × 32 × 5

The following important theorem shows that if a number is not divisibleby any prime less than to its square root then the number must be prime.

Theorem 10.5If n is a composite integer, then n has a prime divisor less than or equal to√n.

Proof.Since n is composite, there is a divisor a of n such that 1 < a < n. Writen = ab where b is a positive integer. Note that b is also a divisor of n. Ifa >√n and b >

√n then n = ab >

√n√n = n, a false conclusion. Thus,

either a ≤√n or b ≤

√n. Hence, n has a positive divisor which is less than

or equal to√n. If the divisor, say a, is a prime then we are done. If a is not

a prime then by the Fundamental Theorem of Arithmetic there is a primenumber p that divides a.That is, a = pk. But then n = pqb so that p dividesn and p < a ≤

√n. In either case, n has a prime divisor less than or equal

to√n

Example 10.4Use the previous theorem to show that the number 101 is prime.

Solution.The prime numbers less than or equal to

√101 are: 2, 3, 5, 7. Since none of

them divides 101, by the previous theorem, 101 is prime

Let a and b be two integers, not both zero. A positive integer d is calleda greatest common divisor of a and b if the following two conditions hold:(1) d|a and d|b.

Page 90: Discrete Book

86 NUMBER THEORY AND MATHEMATICAL PROOFS

(2) If c|a and c|b then c|d.We write d = (a, b) or d = gcd(a, b). If gcd(a, b) = 1 then we say that a andb are relatively prime. For example, the numbers 2 and 3 are relativelyprime.

Example 10.5Find the greatest common divisor of 42 and 56.

Solution.SinceD42 = {±1,±2,±3,±6,±7,±14,±21,±42} andD56 = {±1,±2,±4,±7,±8,±14,±28,±56},we have gcd(42, 56) = 14

The greatest common divisor is useful for writing fractions in lowest term.For example, −42

56= −3

4where we cancelled 14 = gcd(42, 56).

Remark 10.2In the next section we will establish the existence and uniqueness of thegreatest common divisor and provide an algorithm of how to find it.

Page 91: Discrete Book

10 DIVISIBILITY. THE DIVISION ALGORITHM 87

Review Problems

Problem 10.1Show that the set of all positive real numbers does not have a least element.

Problem 10.2Use the well-ordering principle to show that every positive integer n ≥ 8 canbe written as sums of 3’s and 5’s.

Problem 10.3Let a and b be positive integers such that a

b=√

2. Let S = {n ∈ N : nb

=√

2}.Show that S is the empty set. Hence, concluding that

√2 is irrational.

Problem 10.4Let n and m be integers such that n|m where n 6= 0. Show that n|(−m).

Problem 10.5Let n, a and b be integers such that n|a and n|b where n 6= 0. Show thatn|(a± b).

Problem 10.6Let n,m and p be integers such that n|m and m|p where n,m 6= 0. Showthat n|p.

Problem 10.7Let n and m be integers such that n|m and m|n where n,m 6= 0. Show thateither n = m or n = −m.

Problem 10.8Write the first 7 prime numbers.

Problem 10.9Show that 227 is a prime number.

Problem 10.10Write the prime factorization of 42 and 105.

Problem 10.11Find the greatest common divisor of 42 and 105.

Page 92: Discrete Book

88 NUMBER THEORY AND MATHEMATICAL PROOFS

Problem 10.12Are the numbers 27 and 35 relatively prime?

Problem 10.13We say that an integer n is even if and only if there exists an integer k suchthat n = 2k. An integer n is said to be odd if and only if there exists aninteger k such that n = 2k + 1.Let m and n be two integers.(a) Is 6m+ 8n an even integer?(b) Is 6m+ 4n2 + 3 odd?

Problem 10.14Let a, b, and c be integers such that a|b where a 6= 0. Show that a|(bc).

Problem 10.15Let m and n be positive integers with m > n. Is m2 − n2 composite?

Page 93: Discrete Book

11 THE EUCLIDEAN ALGORITHM 89

11 The Euclidean Algorithm

Recall the definition of the greatest common divisor from the previous sec-tion. Let a and b be two integers, not both zero. A positive integer d is calleda greatest common divisor of a and b if the following two conditions hold:(1) d|a and d|b.(2) If c|a and c|b then c|d.We write d = (a, b) or d = gcd(a, b).

We next discuss a systematic procedure for finding the greatest commomdivisor of two integers, known as the Euclid’s Algorithm. For that pur-pose, we need the following result.

Theorem 11.1If a, b, q, and r are integers such that a = bq + r then gcd(a, b) = gcd(b, r).

Proof.Let d1 = gcd(a, b) and d2 = gcd(b, r). We will show that d1 = d2. Since d2|bqand d2|r, we have d2|(bq+ r) (Problem 10.5). Hence, d2|a. Thus, by the defi-nition of gcd, we have d2|d1. Since d1|b, we have d1|bq (Problem 10.7). Sinced1|a, we have d1|(a − bq) (Problem 10.5). Hence, d1|r. From the definitionof d2, we have d1|d2. Since d1 and d2 are positive, using Problem 10.7, weconclude d1 = d2

The following theorem, establishes the existence and uniqueness of the great-est common divisor and provides an algorithm of how to find it.

Theorem 11.2 (The Euclidean Algorithm)If a and b are two integers with b > 0, then there exists a unique positiveinteger d such that the two conditions (1) and (2) above are satisfied.

Proof.Uniqueness: Let d1 and d2 be two positive integers that satisfy conditions(1) and (2). Then by (2) we can write d1|d2 and d2|d1. Since d1 and d2 areboth positive, Problem 10.7, implies that d1 = d2.

Existence: If a = 0 then d = b. So assume that a 6= 0. By the Divisionalgorithm there exist unique integers q1 and r1 such that

a = bq1 + r1, 0 ≤ r1 < b.

Page 94: Discrete Book

90 NUMBER THEORY AND MATHEMATICAL PROOFS

If r1 = 0 then d = b. So assume r1 > 0. Using the Division algorithm for asecond time to find unique integers q2 and r2 such that

b = r1q2 + r2 0 ≤ r2 < r1.

If r2 = 0 then r1 = gcd(b, r1) = gcd(a, b). We keep this process going andeventually we will find integers rn−2, rn−1 and rn such that

rn−2 = rn−1qn + rn, 0 ≤ rn < rn−1

andrn−1 = rnqn+1.

That is, rn is the last nonzero remainder in the process. By Theorem 11.1,we see that rn = gcd(a, b)

Example 11.1Find gcd(1776,1492).

Solution.Performing the arithmetic for the Euclidean algorithm we have

1776 = (1)(1492) + 284

1492 = (5)(284) + 72

284 = (3)(72) + 68

72 = (1)(68) + 4

68 = 4(17).

So gcd(1776, 1492) = 4

Page 95: Discrete Book

11 THE EUCLIDEAN ALGORITHM 91

Review Problems

Problem 11.1(i) Find gcd(120, 500).(ii) Show that 17 and 22 are relatively prime.

Problem 11.2If two integers a and b have the property that their difference a−b is divisibleby an integer n, i.e., a − b = nq for some integer q, we say that a and b arecongruent modulo n. Symbolically, we write a ≡ b(mod n). Show that ifa ≡ b(mod n) and c ≡ d(mod n) then a+ c ≡ (b+ d)(mod n).

Problem 11.3Show that if a ≡ b(mod n) and c ≡ d(mod n) then ac ≡ bd(mod n).

Problem 11.4Show that if a ≡ a(mod n) for all integer a.

Problem 11.5Show that if a ≡ b(mod n) then b ≡ a(mod n).

Problem 11.6Show that if a ≡ b(mod n) and b ≡ c(mod n) then a ≡ c(mod n).

Problem 11.7What are the solutions of the linear congruences 3x ≡ 4(mod 7)?

Problem 11.8Solve 2x+ 11 ≡ 7(mod 3)?

Problem 11.9Use the Euclidean algorithm to find gcd(414, 662).

Problem 11.10Use the Euclidean algorithm to find gcd(287, 91).

Problem 11.11Use the Euclidean algorithm to find gcd(24,−25).

Problem 11.12Use the Euclidean algorithm to find gcd(−6,−15).

Page 96: Discrete Book

92 NUMBER THEORY AND MATHEMATICAL PROOFS

Page 97: Discrete Book

Fundamentals of Set Theory

Set is the most basic term in mathematics and computer science. Hardlyany discussion in either subject can proceed without set or some synonymsuch as class or collection. In this chapter we introduce the concept of setsand its various operations and then study the properties of these operations.

93

Page 98: Discrete Book

94 FUNDAMENTALS OF SET THEORY

12 Basic Definitions

We first consider the following known as the barber puzzle: “The armycaptain orders his company barber to shave all members of the companyprovided they do not shave themselves. The barber is so busy at first thathis own beard begins to be unsightly. Just as he lathers up, the impossibilityof his position strikes him: If he shaves himself, he disobeys the captain’s or-der. If he does not shave himself, then by the captain’s order he is supposedto shave himself.”A situation like this is known as a paradox. To resolve the problem one hasto take the barber out of the company.Naive set theory defines a set to be any definable collection. Such a defi-nition leads to the following paradox:Russell’s Paradox. Consider the set A = {X : X is a set, X 6∈ X}.Since A is a set, saying that A ∈ A will imply that A 6∈ A by the definitionof A. Saying that A 6∈ A means that A ∈ A by the definition of A. Thusin either case the assumption that A is a set leads to an untenable paradox:A ∈ A and A 6∈ A.Such a paradox indicated the necessity of a formal axiomatization of set the-ory.We define a set A as a collection of well-defined objects (called elements ormembers of A) such that for any given object x either one (but not both)of the following holds:

• x belongs to A and we write x ∈ A.

• x does not belong to A, and in this case we write x 6∈ A.

With this definition, the A in Russell’s paradox is not a set.We denote sets by capital letters A,B,C, · · · and elements by lowercase let-ters a, b, c, · · · Sets consisting of sets will be denoted by script letters.There are different ways to represent a set. The first one is to list, withoutrepetition, the elements of the set. We say that the set is given in tabularform. Another way is to describe a property characterizing the members ofthe set. Such a representation is referred to as the descriptive form of aset. The third way is to write in symbolic form the common characteristicshared by all the elements of the set such as

A = {x ∈ Z|x2 − 4 = 0}.

Page 99: Discrete Book

12 BASIC DEFINITIONS 95

We refer to such a form as the set-builder form.We define the empty set, denoted by ∅, to be the set with no elements.

Example 12.1In what form are the following sets are defined?(a) A is the vowel of the English alphabet.(b) A = {1, 2, 3}.(c) A = {x ∈ R|x2 − 1 = 0}.

Solution.(a) Descriptive form.(b) Tabular form.(c) Set-builder form

Example 12.2List the elements of the following sets:(a) {x ∈ R|x2 = 1}.(b) {x ∈ Z|x2 − 3 = 0}.

Solution.(a) {−1, 1}.(b) ∅

Example 12.3Write the set-builder form of each of the following sets.(a) {a, e, i, o, u}.(b) {1, 3, 5, 7, 9}.

Solution.(a) {x is a letter of the English alphabet|x is a vowel}.(b) {n ∈ N|nis odd and less than 10}

Let A and B be two sets. We say that A is a subset of B, denoted byA ⊆ B, if and only if every element of A is also an element of B. Symboli-cally:

[A ⊆ B]⇔ [x ∈ A =⇒ x ∈ B].

If there exists an element of A which is not in B then we write A 6⊆ B.Now, for any set A, the proposition x ∈ ∅ =⇒ x ∈ A is vacuously true sincex ∈ ∅ is always false. Hence ∅ ⊆ A.

Page 100: Discrete Book

96 FUNDAMENTALS OF SET THEORY

Example 12.4Suppose that A = {2, 4, 6}, B = {2, 6}, and C = {4, 6}. Determine which ofthese sets are subsets of which other(s) of these sets.

Solution.B ⊆ A and C ⊆ A

If sets A and B are represented as regions in the plane, relationships be-tween A and B can be represented by pictures, called Venn diagrams.

Example 12.5Represent A ⊆ B using a Venn diagram.

Solution.

Two sets A and B are said to be equal if and only if A ⊆ B and B ⊆ A.We write A = B. Thus, to show that A = B it suffices to show the doubleinclusions mentioned in the definition. For non-equal sets we write A 6= B.

Example 12.6Determine whether each of the following pairs of sets are equal.(a) {1, 3, 5} and {5, 3, 1}.(b) {{1}} and {1, {1}}.

Solution.(a) {1, 3, 5} = {5, 3, 1}.(b) {{1}} 6= {1, {1}} since 1 6∈ {{1}}

Let A and B be two sets. We say that A is a proper subset of B, de-noted by A ⊂ B, if A ⊆ B and A 6= B. Thus, to show that A is a propersubset of B we must show that every element of A is an element of B andthere is an element of B which is not in A.

Page 101: Discrete Book

12 BASIC DEFINITIONS 97

Example 12.7Order the sets of numbers: Z,R,Q,N using ⊂

Solution.N ⊂ Z ⊂ Q ⊂ R

Example 12.8Determine whether each of the following statements is true or false.(a) x ∈ {x} (b) {x} ⊆ {x} (c) {x} ∈ {x}(d) {x} ∈ {{x}} (e) ∅ ⊆ {x} (f) ∅ ∈ {x}

Solution.(a) True (b) True (c) False (d) True (e) True (f) False

If U is a given set whose subsets are under consideration, then we call Ua universal set.Let U be a universal set and A,B be two subsets of U. The absolute com-plement of A is the set

Ac = {x ∈ U |x 6∈ A}.

The relative complement of A with respect to B is the set

B − A = {x ∈ U |x ∈ B and x 6∈ A}.

Example 12.9Let U = R. Consider the sets A = {x ∈ R|x < −1 or x > 1} and B = {x ∈R|x ≤ 0}. Find(a) Ac.(b) B − A.

Solution.(a) Ac = [−1, 1].(b) B − A = [−1, 0]

Let A and B be two sets. The union of A and B is the set

A ∪B = {x|x ∈ A or x ∈ B}.

Page 102: Discrete Book

98 FUNDAMENTALS OF SET THEORY

where the “or” is inclusive. This definition can be extended to more thantwo sets. More precisely, if A1, A2, · · · , are sets then

∪∞n=1An = {x|x ∈ Ai for some i}.

Let A and B be two sets. The intersection of A and B is the set

A ∩B = {x|x ∈ A and x ∈ B}.

If A ∩ B = ∅ we say that A and B are disjoint sets. Given the setsA1, A2, · · · , we define

∩∞n=1An = {x|x ∈ Ai for all i}.

Example 12.10Let A = {a, b, c}, B = {b, c, d}, and C = {b, c, e}.(a) Find A ∪ (B ∩ C), (A ∪ B) ∩ C, and (A ∪ B) ∩ (A ∪ C). Which of thesesets are equal?(b) Find A ∩ (B ∪ C), (A ∩ B) ∪ C, and (A ∩ B) ∪ (A ∩ C). Which of thesesets are equal?(c) Find A− (B − C) and (A−B)− C. Are these sets equal?

Solution.(a) A ∪ (B ∩ C) = A, (A ∪ B) ∩ C = {b, c}, (A ∪ B) ∩ (A ∪ C) = {b, c} =(A ∪B) ∩ C.(b) A ∩ (B ∪ C) = {b, c}, (A ∩ B) ∪ C = C, (A ∩ B) ∪ (A ∩ C) = {b, c} =A ∩ (B ∪ C).(c) A− (B − C) = A and (A−B)− C = {a} 6= A− (B − C)

Example 12.11For each n ≥ 1, let An = {x ∈ R : x < 1 + 1

n}. Show that

∩∞n=1An = {x ∈ R : x ≤ 1}.

Solution.The proof is by double inclusions method. Let y ∈ {x ∈ R : x ≤ 1}. Thenfor all positive integer n we have y ≤ 1 < 1 + 1

n. That is, y ∈ ∩∞n=1An. This

shows that {x ∈ R : x ≤ 1} ⊆ ∩∞n=1An.Conversely, let y ∈ ∩∞n=1An. Then y < 1 + 1

nfor all n ≥ 1. Now take the limit

of both sides as n→∞ to obtain y ≤ 1. That is, y ∈ {x ∈ R : x ≤ 1}. Thisshows that ∩∞n=1An ⊆ {x ∈ R : x ≤ 1}

Page 103: Discrete Book

12 BASIC DEFINITIONS 99

Example 12.12The symmetric difference of A and B, denoted by A∆B, is the set contain-ing those elements in either A or B but not both. Find A∆B if A = {1, 3, 5}and B = {1, 2, 3}.

Solution.A∆B = {2, 5}

The notation (a1, a2, · · · , an) is called an ordered n-tuples. We say thattwo n−tuples (a1, a2, · · · , an) and (b1, b2, · · · , bn) are equal if and only ifa1 = b1, a2 = b2, · · · , an = bn.Given n sets A1, A2, · · · , An the Cartesian product of these sets is the set

A1 × A2 × · · · × An = {(a1, a2, · · · , an)|a1 ∈ A1, a2 ∈ A2, · · · , an ∈ An}

Example 12.13Let A = {x, y}, B = {1, 2, 3}, and C = {a, b}. Find(a) A×B × C.(b) (A×B)× C.

Solution.(a)

A×B × C ={(x, 1, a), (x, 2, a), (x, 3, a), (y, 1, a), (y, 2, a),

(y, 3, a), (x, 1, b), (x, 2, b), (x, 3, b), (y, 1, b)

(y, 2, b), (y, 3, b)}

(b)

(A×B)× C ={((x, 1), a), ((x, 2), a), ((x, 3), a), ((y, 1), a), ((y, 2), a),

((y, 3), a), ((x, 1), b), ((x, 2), b), ((x, 3), b), ((y, 1), b)

((y, 2), b), ((y, 3), b)}

Page 104: Discrete Book

100 FUNDAMENTALS OF SET THEORY

Review Problems

Problem 12.1Write each the following sets in tabular form.(a) A = {x ∈ N |x > 0}.(b) A consists of the first five prime number.(c) A = {x ∈ Z|2x2 + x− 1 = 0}.

Problem 12.2Write each the following sets in set-builder notation.(a) A = {1, 2, 3, 4, 5}.(b) A = {4, 6, 8, 9, 10}.

Problem 12.3Which of the following sets are equal?(a) {a, b, c, d}(b) {d, e, a, c}(c) {d, b, a, c}(d) {a, a, d, e, c, e}

Problem 12.4Let A = {c, d, f, g}, B = {f, j}, and C = {d, g}. Answer each of the followingquestions. Give reasons for your answers.(a) Is B ⊆ A?(b) Is C ⊆ A?(c) Is C ⊆ C?(d) Is C a proper subset of A?

Problem 12.5(a) Is 3 ∈ {1, 2, 3}?(b) Is 1 ⊆ {1}?(c) Is {2} ∈ {1, 2}?(d) Is {3} ∈ {1, {2}, {3}}?(e) Is 1 ∈ {1}?(f) Is {2} ⊆ {1, {2}, {3}}?(g) Is {1} ⊆ {1, 2}?(h) Is 1 ∈ {{1}, 2}?(i) Is {1} ⊆ {1, {2}}?(j) Is {1} ⊆ {1}?

Page 105: Discrete Book

12 BASIC DEFINITIONS 101

Problem 12.6Let A = {b, c, d, f, g} and B = {a, b, c}. Find each of the following:(a) A ∪B.(b) A ∩B.(c) A−B.(d) B − A.

Problem 12.7Indicate which of the following relationships are true and which are false:(a) Z+ ⊆ Q.(b) R− ⊂ Q.(c) Q ⊂ Z.(d) Z+ ∪ Z− = Z.(e) Q ∩ R = Q.(f) Q ∪ Z = Z.(g) Z+ ∩ R = Z+

(h) Z ∪Q = Q.

Problem 12.8Let A = {x, y, z, w} and B = {a, b}. List the elements of each of the followingsets:(a) A×B(b) B × A(c) A× A(d) B ×B.

Problem 12.9(a) Find all possible subsets of the set A = {a, b, c}.(b) How many proper subsets are there?

Problem 12.10Subway prepared 60 4-inch sandwiches for a birthday party. Among thesesandwiches, 45 of them had tomatoes, 30 had both tomatoes and onions,and 5 had neither tomatoes nor onions. Using a Venn diagram, how manysandwiches did he make with(a) tomatoes or onions?(b) onions?(c) onions but not tomatoes?

Page 106: Discrete Book

102 FUNDAMENTALS OF SET THEORY

Problem 12.11A camp of international students has 110 students. Among these students,

75 speak english,52 speak spanish,50 speak french,33 speak english and spanish,30 speak english and french,22 speak spanish and french,13 speak all three languages.

How many students speak(a) english and spanish, but not french,(b) neither english, spanish, nor french,(c) french, but neither english nor spanish,(d) english, but not spanish,(e) only one of the three languages,(f) exactly two of the three languages.

Problem 12.12Let A be the set of the first five composite numbers and B be the set ofpositive integers less than or equal to 8. Find A−B and B − A.

Problem 12.13Let U be a universal set. Find U c and ∅c.

Problem 12.14Let A be the set of natural numbers less than 0 and B = {1, 3, 7}. Find A∪Band A ∩B.

Problem 12.15Let

A ={x ∈ N|4 ≤ x ≤ 8}B ={x ∈ N|x even andx ≤ 10}.

Find A ∪B and A ∩B.

Problem 12.16Using a Venn diagram, show that A∆B = (A− B) ∪ (B − A) = (A ∩ Bc) ∪(B ∩ Ac).

Page 107: Discrete Book

13 PROPERTIES OF SETS 103

13 Properties of Sets

In this section, we derive some properties of the set theory operations in-troduced in the previous section. The following example shows that theoperation ⊆ is reflexive and transitive, concepts that will be discussed in thenext chapter.

Example 13.1(a) Suppose that A,B,C are sets such that A ⊆ B and B ⊆ C. Show thatA ⊆ C.(b) Find two sets A and B such that A ∈ B and A ⊆ B.(c) Show that A ⊆ A.

Solution.(a) We need to show that every element of A is an element of C. Let x ∈ A.Since A ⊆ B, we have x ∈ B. But B ⊆ C so that x ∈ C.(b) A = {x} and B = {x, {x}}.(c) The proposition if x ∈ A then x ∈ A is always true. Thus, A ⊆ A

Theorem 13.1Let A and B be two sets. Then(a) A ∩B ⊆ A and A ∩B ⊆ B.(b) A ⊆ A ∪B and B ⊆ A ∪B.

Proof.(a) If x ∈ A ∩ B then x ∈ A and x ∈ B. This still imply that x ∈ A. Hence,A ∩B ⊆ A. A similar argument holds for A ∩B ⊆ B.(b) The proposition “if x ∈ A then x ∈ A ∪ B” is always true. Hence,A ⊆ A ∪B. A similar argument holds for B ⊆ A ∪B

Theorem 13.2Let A be a subset of a universal set U. Then(a) ∅c = U.(b) U c = ∅.(c) (Ac)c = A.(d) A ∪ Ac = U.(e) A ∩ Ac = ∅.

Page 108: Discrete Book

104 FUNDAMENTALS OF SET THEORY

Proof.(a) If x ∈ U then x ∈ U and x 6∈ ∅. Thus, U ⊆ ∅c. Conversely, suppose thatx ∈ ∅c. Then x ∈ U and x 6∈ ∅. This implies that x ∈ U. Hence, ∅c ⊆ U.(b) It is always true that ∅ ⊆ U c. Conversely, the proposition “x ∈ U andx 6∈ U implies x ∈ ∅” is vacuously true since the hypothesis is false. Thissays that U c ⊆ ∅.(c) Let x ∈ (Ac)c. Then x ∈ U and x 6∈ Ac. That is, x ∈ U and (x 6∈ U orx ∈ A). Since x ∈ U, we have x ∈ A. Hence, (Ac)c ⊆ A. Conversely, supposethat x ∈ A. Then x ∈ U and x ∈ A. That is, x ∈ U and x 6∈ Ac. Thus,x ∈ (Ac)c. This shows that A ⊆ (Ac)c.(d) It is clear that A ∪Ac ⊆ U. Conversely, suppose that x ∈ U. Then eitherx ∈ A or x 6∈ A. But this is the same as saying that x ∈ A ∪ Ac.(e) By definition ∅ ⊆ A∩Ac. Conversely, the conditional proposition “x ∈ Aand x 6∈ A implies x ∈ ∅” is vacuously true since the hypothesis is false. Thisshows that A ∩ Ac ⊆ ∅

Theorem 13.3If A and B are subsets of U then(a) A ∪ U = U.(b) A ∪ A = A.(c) A ∪ ∅ = A.(d) A ∪B = B ∪ A.(e) (A ∪B) ∪ C = A ∪ (B ∪ C).

Proof.(a) Clearly, A ∪ U ⊆ U. Conversely, let x ∈ U . Then definitely, x ∈ A ∪ U.That is, U ⊆ A ∪ U.(b) If x ∈ A then x ∈ A or x ∈ A. That is, x ∈ A ∪ A and consequentlyA ⊆ A ∪ A. Conversely, if x ∈ A ∪ A then x ∈ A. Hence, A ∪ A ⊆ A.(c) If x ∈ A ∪ ∅ then x ∈ A since x 6∈ ∅. Thus, A ∪ ∅ ⊆ A. Conversely, ifx ∈ A then x ∈ A or x ∈ ∅. Hence, A ⊆ A ∪ ∅.(d) If x ∈ A ∪ B then x ∈ A or x ∈ B. But this is the same thing as sayingx ∈ B or x ∈ A. That is, x ∈ B ∪ A. Now interchange the roles of A and Bto show that B ∪ A ⊆ A ∪B.(e) Let x ∈ (A∪B)∪C. Then x ∈ (A∪B) or x ∈ C. Thus, (x ∈ A or x ∈ B)or x ∈ C. This implies x ∈ A or (x ∈ B or x ∈ C). Hence, x ∈ A ∪ (B ∪ C).The converse is similar

Page 109: Discrete Book

13 PROPERTIES OF SETS 105

Theorem 13.4Let A and B be subsets of U . Then(a) A ∩ U = A.(b) A ∩ A = A.(c) A ∩ ∅ = ∅.(d) A ∩B = B ∩ A.(e) (A ∩B) ∩ C = A ∩ (B ∩ C).

Proof.(a) If x ∈ A ∩ U then x ∈ A. That is , A ∩ U ⊆ A. Conversely, let x ∈ A.Then definitely, x ∈ A and x ∈ U. That is, x ∈ A ∩ U. Hence, A ⊆ A ∩ U.(b) If x ∈ A then x ∈ A and x ∈ A. That is, A ⊆ A ∩ A. Conversely, ifx ∈ A ∩ A then x ∈ A. Hence, A ∩ A ⊆ A.(c) Clearly ∅ ⊆ A∩ ∅. Conversely, if x ∈ A∩ ∅ then x ∈ ∅. Hence, A∩ ∅ ⊆ ∅.(d) If x ∈ A∩B then x ∈ A and x ∈ B. But this is the same thing as sayingx ∈ B and x ∈ A. That is, x ∈ B ∩A. Now interchange the roles of A and Bto show that B ∩ A ⊆ A ∩B.(e) Let x ∈ (A ∩ B) ∩ C. Then x ∈ (A ∩ B) and x ∈ C. Thus, (x ∈ A andx ∈ B) and x ∈ C. This implies x ∈ A and (x ∈ B and x ∈ C). Hence,x ∈ A ∩ (B ∩ C). The converse is similar

Theorem 13.5If A,B, and C are subsets of U then(a) A ∩ (B ∪ C) = (A ∩B) ∪ (A ∩ C).(b) A ∪ (B ∩ C) = (A ∪B) ∩ (A ∪ C).

Proof.(a) Let x ∈ A ∩ (B ∪ C). Then x ∈ A and x ∈ B ∪ C. Thus, x ∈ A and(x ∈ B or x ∈ C). This implies that (x ∈ A and x ∈ B) or (x ∈ A andx ∈ C). Hence, x ∈ A ∩ B or x ∈ A ∩ C, i.e. x ∈ (A ∩ B) ∪ (A ∩ C). Theconverse is similar.(b) Let x ∈ A ∪ (B ∩ C). Then x ∈ A or x ∈ B ∩ C. Thus, x ∈ A or (x ∈ Band x ∈ C). This implies that (x ∈ A or x ∈ B) and (x ∈ A or x ∈ C).Hence, x ∈ A ∪ B and x ∈ A ∪ C, i.e. x ∈ (A ∪ B) ∩ (A ∪ C). The converseis similar

Theorem 13.6 (De Morgan’s Laws)Let A and B be subsets of U then(a) (A ∪B)c = Ac ∩Bc.(b) (A ∩B)c = Ac ∪Bc.

Page 110: Discrete Book

106 FUNDAMENTALS OF SET THEORY

Proof.(a) Let x ∈ (A ∪B)c. Then x ∈ U and x 6∈ A ∪B. Hence, x ∈ U and (x 6∈ Aand x 6∈ B). This implies that (x ∈ U and x 6∈ A) and (x ∈ U and x 6∈ B).It follows that x ∈ Ac ∩Bc. Now, go backward for the converse.(b) Let x ∈ (A∩B)c. Then x ∈ U and x 6∈ A∩B. Hence, x ∈ U and (x 6∈ Aor x 6∈ B). This implies that (x ∈ U and x 6∈ B) or (x ∈ U and x 6∈ A). Itfollows that x ∈ Ac ∪Bc. The converse is similar

Theorem 13.7Suppose that A ⊆ B. Then(a) A ∩B = A.(b) A ∪B = B.

Proof.(a) If x ∈ A ∩ B then by the definition of intersection of two sets we havex ∈ A. Hence, A ∩ B ⊆ A. Conversely, if x ∈ A then x ∈ B as well sinceA ⊆ B. Hence, x ∈ A ∩B. This shows that A ⊆ A ∩B.(b) If x ∈ A ∪ B then x ∈ A or x ∈ B. Since A ⊆ B we have x ∈ B.Hence, A ∪ B ⊆ B. Conversely, if x ∈ B then x ∈ A ∪ B. This shows thatB ⊆ A ∪B

Example 13.2Let A and B be arbitrary sets. Show that (A−B) ∩B = ∅.

Solution.Suppose not. That is, suppose (A−B)∩B 6= ∅. Then there is an element xthat belongs to both A−B and B. By the definition of A−B we have thatx 6∈ B. Thus, x ∈ B and x 6∈ B which is a contradiction

A collection of nonempty subsets {A1, A2, · · · , An} of A is said to be a par-tition of A if and only if

(i) A = ∪nk=1Ak.(ii) Ai ∩ Aj = ∅ for all i 6= j.

Example 13.3Let A = {1, 2, 3, 4, 5, 6}, A1 = {1, 2}, A2 = {3, 4}, A3 = {5, 6}. Show that{A1, A2, A3} is a partition of A.

Page 111: Discrete Book

13 PROPERTIES OF SETS 107

Solution.(i) A1 ∪ A2 ∪ A3 = A.(ii) A1 ∩ A2 = A1 ∩ A3 = A2 ∩ A3 = ∅

The number of elements of a set is called the cardinality of the set. Wewrite |A| to denote the cardinality of the set A. If A has a finite cardinalitywe say that A is a finite set. Otherwise, it is called infinite.

Example 13.4What is the cardinality of each of the following sets?(a) ∅.(b) {∅}.(c) {a, {a}, {a, {a}}}.

Solution.(a) |∅| = 0(b) |{∅}| = 1(c) |{a, {a}, {a, {a}}}| = 3

Let A be a set. The power set of A, denoted by P(A), is the empty settogether with all possible subsets of A.

Example 13.5Find the power set of A = {a, b, c}.

Solution.

P(A) = {∅, {a}, {b}, {c}, {a, b}, {a, c}, {b, c}, {a, b, c}}

Theorem 13.8If A ⊆ B then P(A) ⊆ P(B).

Proof.Let X ∈ P(A). Then X ⊆ A. Since A ⊆ B, we have X ⊆ B. Hence,X ∈ P(B)

Example 13.6(a) Use induction to show that if |A| = n then |P(A)| = 2n.(b) If P(A) has 256 elements, how many elements are there in A?

Page 112: Discrete Book

108 FUNDAMENTALS OF SET THEORY

Solution.(a) If n = 0 then A = ∅ and in this case P(A) = {∅}. Thus |P(A)| = 1.As induction hypothesis, suppose that if |A| = n then |P(A)| = 2n. LetB = A∪ {an+1}. Then P(B) consists of all subsets of A and all subsets of Awith the element an+1 added to them. Hence, |P(B)| = 2n+2n = 2·2n = 2n+1.(b) Since |P(A)| = 256 = 28, we have |A| = 8

Page 113: Discrete Book

13 PROPERTIES OF SETS 109

Review Problems

Problem 13.1Let A,B, and C be sets. Prove that if A ⊆ B then A ∩ C ⊆ B ∩ C.

Problem 13.2Find sets A,B, and C such that A ∩ C = B ∩ C but A 6= B.

Problem 13.3Find sets A,B, and C such that A ∩ C ⊆ B ∩ C and A ∪ C ⊆ B ∪ C butA 6= B.

Problem 13.4Let A and B be two sets. Prove that if A ⊆ B then Bc ⊆ Ac.

Problem 13.5Let A,B, and C be sets. Prove that if A ⊆ C and B ⊆ C then A ∪B ⊆ C.

Problem 13.6Let A,B, and C be sets. Show that A× (B ∪ C) = (A×B) ∪ (A× C).

Problem 13.7Let A,B, and C be sets. Show that A× (B ∩ C) = (A×B) ∩ (A× C).

Problem 13.8(a) Is the number 0 in ∅? Why?(b) Is ∅ = {∅}? Why?(c) Is ∅ ∈ {∅}? Why?

Problem 13.9Let A and B be two sets. Prove that (A−B) ∩ (A ∩B) = ∅.

Problem 13.10Let A and B be two sets. Show that if A ⊆ B then A ∩Bc = ∅.

Problem 13.11Let A,B and C be three sets. Prove that if A ⊆ B and B ∩ C = ∅ thenA ∩ C = ∅.

Page 114: Discrete Book

110 FUNDAMENTALS OF SET THEORY

Problem 13.12Find two sets A and B such that A ∩B = ∅ but A×B 6= ∅.

Problem 13.13Suppose that A = {1, 2} and B = {2, 3}. Find each of the following:(a) P(A ∩ B).(b) P(A).(c) P(A ∪ B).(d) P(A× B).

Problem 13.14(a) Find P(∅).(b) Find P(P(∅)).(c) Find P(P(P(∅))).

Problem 13.15Determine which of the following statements are true and which are false.Prove each statement that is true and give a counterexample for each state-ment that is false.(a) P(A ∪ B) = P(A) ∪ P(B).(b) P(A ∩ B) = P(A) ∩ P(B).(c) P(A) ∪ P(B) ⊆ P(A ∪ B).(d) P(A× B) = P(A)× P(B).

Page 115: Discrete Book

14 BOOLEAN ALGEBRA 111

14 Boolean Algebra

A Boolean algebra is a nonempty set S together with two operations ⊕and � that satisfy the following axioms:

• Closure: a⊕ b ∈ S and a� b ∈ S for all a, b ∈ S.• Cummutative law: a⊕ b = b⊕ a and a� b = b� a, ∀a, b ∈ S.• Associative law: a⊕ (b⊕ c) = (a⊕ b)⊕ c and a� (b� c) = (a� b)� c),∀a, b, c ∈ S.•Distributive law: a⊕(b�c) = (a⊕b)�(a⊕c) and a�(b⊕c) = a�b⊕a�c∀a, b, c ∈ S.• Identity element: There exist distinct elements 0 and 1 in S such thata⊕ 0 = a and a� 1 = a ∀a ∈ S.• Inverse element: For each a ∈ S there exists an element a such thata⊕ a = 1 and a� a = 0. We call a the complement or the negation of a.

We write (S,⊕,�).

Example 14.1Show that if S is a collection of propositions with finite propositional variablesthen (S,∨,∧) is a Boolean algebra.

Solution.Let p, q, r ∈ S. Then (1) p ∨ q ∈ S and p ∧ q ∈ S.(2) p ∨ q ≡ q ∨ p and p ∧ q ≡ q ∧ p.(3) p ∨ (q ∨ r) ≡ (p ∨ q) ∨ r and p ∧ (q ∧ r) ≡ (p ∧ q) ∧ r.(4) p ∨ (q ∧ r) ≡ (p ∨ q) ∧ (p ∨ r) and p ∧ (q ∨ r) ≡ (p ∧ q) ∨ (p ∧ r).(5) Let c be a contradiction and t a tautology then p ∨ c ≡ p and p ∧ t ≡ p.(6) If p ∈ S then ∼ p ∈ S is such that p∨ ∼ p ≡ t and p∧ ∼ p ≡ c

Example 14.2Show that for a given nonempty set S, (P(S),∪,∩) is a Boolean algebra.

Solution.Let A,B,C ∈ P(S). Then(1) A ∪B ∈ P(S) and A ∩B ∈ P(S).(2) A ∪B = B ∪ A and A ∩B = B ∩ A.(3) (A ∪B) ∪ C = A ∪ (B ∪ C) and (A ∩B) ∩ C = A ∩ (B ∩ C).(4) A∪ (B ∩C) = (A∪B)∩ (A∪C) and A∩ (B ∪C) = (A∩B)∪ (A∩C).

Page 116: Discrete Book

112 FUNDAMENTALS OF SET THEORY

(5) A ∪ ∅ = A and A ∩ S = A.(6) A ∪ Ac = S and A ∩ Ac = ∅

Example 14.3 (Idempotent law)Show that x⊕ x = x.

Solution.We have

x⊕ x =(x⊕ x)� 1 = (x⊕ x)� (x⊕ x)

=x⊕ (x� x)

=x⊕ 0 = x

Example 14.4 (Domination law)Show that x⊕ 1 = 1.

Solution.We have

x⊕ 1 =x⊕ (x⊕ x)

=(x⊕ x)⊕ x=x⊕ x = 1

Example 14.5 (Absorption law)Show that (x� y)⊕ x = x.

Solution.We have

(x� y)⊕ x =x� y ⊕ x� 1

=x� (y ⊕ 1)

=x� 1 = x

Example 14.6Show that if x⊕ y = 1 and x� y = 0 then y = x.

Page 117: Discrete Book

14 BOOLEAN ALGEBRA 113

Solution.We have

y =y � 1

=y � (x⊕ x)

=y � x⊕ y � x=0⊕ y � x=y � x.

Likewise, we have

x =x� 1

=x� (x⊕ y)

=x� x⊕ x� y=0⊕ x� y=x� y.

Hence, y = x

Example 14.7 (Double Complement)Show that x = x.

Solution.Since x⊕ x = 1 and x� x = 0, the previous example implies that x = x

Example 14.8Let u ∈ S such that x⊕ u = x for all x ∈ S. Show that u = 0.

Solution.Since x⊕ u = x for all x ∈ S, by choosing x = 0 we have 0 = 0⊕ u = u

Example 14.9 (DeMorgan’s law)Show that x� y = x⊕ y.

Solution.We have

x� y ⊕ (x⊕ y) =[x⊕ (x⊕ y)]� [y ⊕ (x⊕ y)]

=[(x⊕ x)⊕ y]� [x⊕ (y ⊕ y)]

=(1⊕ y)� (1⊕ x)

=1� 1 = 1

Page 118: Discrete Book

114 FUNDAMENTALS OF SET THEORY

and

(x� y)� (x⊕ y) =[(x� y)� x]⊕ [(x� y)� y]

=[(x� x)� y]⊕ [x� (y � y)]

=0� y ⊕ x� 0

=0.

Hence, by Example 14.6, we have x� y = x⊕ y

Page 119: Discrete Book

14 BOOLEAN ALGEBRA 115

Review Problems

Problem 14.1Show that x� x = x.

Problem 14.2Show that x� 0 = 0.

Problem 14.3Show that (x⊕ y)� x = x.

Problem 14.4Show that x⊕ (x� y) = x+ y.

Problem 14.5Let v ∈ S such that x� v = x for all x ∈ S. Show that v = 1.

Problem 14.6Show that x⊕ y = x� y.

Problem 14.7Show that 1 = 0 and 0 = 1.

Problem 14.8Show that x� (x⊕ y) = x� y.

Page 120: Discrete Book

116 FUNDAMENTALS OF SET THEORY

Page 121: Discrete Book

Relations and Functions

The reader is familiar with many relations which are used in mathematicsand computer science, i.e. “is a subset of”, “ is less than” and so on.One frequently wants to compare or contrast various members of a set, per-haps to arrange them in some appropriate order or to group together thosewith similar properties. The mathematical framework to describe this kindof organization of sets is the theory of relations.There are two kinds of relations which we discuss in this chapter: (i) equiv-alence relations, and (ii) order relations.

117

Page 122: Discrete Book

118 RELATIONS AND FUNCTIONS

15 Equivalence Relations

Let A be a given set. An ordered pair (a, b) of elements in A is definedto be the set {a, {a, b}}. The element a (resp. b) is called the first (resp.second) component.

Example 15.1(a) Show that if a 6= b then (a, b) 6= (b, a).(b) Show that (a, b) = (c, d) if and only if a = c and b = d.

Solution.(a) If a 6= b then {a, {a, b}} 6= {b, {a, b}}. That is, (a, b) 6= (b, a).(b) (a, b) = (c, d) if and only if {a, {a, b}} = {c, {c, d}} and this is equivalentto a = c and {a, b} = {c, d} by the definition of equality of sets. Thus, a = cand b = d

Example 15.2Find x and y such that (x+ y, 0) = (1, x− y).

Solution.We have the system {

x + y = 1

x− y = 0.

Solving by the method of elimination one finds x = 12

and y = 12

If A and B are sets, we let A × B denote the set of all ordered pairs (a, b)where a ∈ A and b ∈ B. We call A × B the Cartesian product of A andB.

Example 15.3(a) Show that if A is a set with m elements and B is a set of n elements thenA×B is a set of mn elements.(b) Show that if A×B = ∅ then A = ∅ or B = ∅.

Page 123: Discrete Book

15 EQUIVALENCE RELATIONS 119

Solution.(a) Suppose that A = {a1, a2, · · · , am} and B = {b1, b2, · · · , bn}. Then

A×B = {(a1, b1), (a1, b2), · · · , (a1, bn),

(a2, b1), (a2, b2), · · · , (a2, bn),

(a3, b1), (a3, b2), · · · , (a3, bn),

...

(am, b1), (am, b2), · · · , (am, bn)}

Thus, n(A×B) = m ·m = n(A) · n(B).(b) We use the proof by contrapositive. Suppose that A 6= ∅ and B 6= ∅.Then there is at least an a ∈ A and an element b ∈ B. That is, (a, b) ∈ A×Band this shows that A×B 6= ∅

Example 15.4Let A = {1, 2}, B = {1}. Show that A×B 6= B × A.

Solution.We have A×B = {(1, 1), (2, 1)} 6= {(1, 1), (1, 2)} = B × A

A binary relation R from a set A to a set B is a subset of A × B. If(a, b) ∈ R we write aRb and we say that a is related to b. If a is not relatedto b we write a 6Rb. In case A = B we call R a binary relation on A.The set

Dom(R) = {a ∈ A|(a, b) ∈ R for some b ∈ B}

is called the domain of R. The set

Range(R) = {b ∈ B|(a, b) ∈ R for some a ∈ A}

is called the range of R.

Example 15.5(a) Let A = {2, 3, 4} and B = {3, 4, 5, 6, 7}. Define the relation R by aRb ifand only if a divides b. Find, R,Dom(R),Range(R).(b) Let A = {1, 2, 3, 4}. Define the relation R by aRb if and only if a ≤ b.Find, R,Dom(R),Range(R).

Page 124: Discrete Book

120 RELATIONS AND FUNCTIONS

Solution.(a)R = {(2, 4), (2, 6), (3, 3), (3, 6), (4, 4)},Dom(R) = {2, 3, 4}, and Range(R) ={3, 4, 6}.(b)R = {(1, 1), (1, 2), (1, 3), (1, 4), (2, 2), (2, 3), (2, 4), (3, 3), (3, 4), (4, 4)},Dom(R) =A,Range(R) = A

A function is a special case of a relation. A function from A to B, de-noted by f : A → B, is a relation from A to B such that for every x ∈ Athere is a unique y ∈ B such that (x, y) ∈ f. The element y is called theimage of x and we write y = f(x). The set A is called the domain of f andthe set of all images of f is called the range of f.

Example 15.6(a) Show that the relation

f = {(1, a), (2, b), (3, a)}

defines a function from A = {1, 2, 3} to B = {a, b, c}. Find its range.(b) Show that the relation f = {(1, a), (2, b), (3, c), (1, b)} does not define afunction from A = {1, 2, 3} to B = {a, b, c}.

Solution.(a) Note that each element of A has exactly one image. Hence, f is a functionwith domain A and range Range(f) = {a, b}.(b) The relation f does not define a function since the element 1 has twoimages, namely a and b

An informative way to picture a relation on a set is to draw its digraph. Todraw a digraph of a relation on a set A, we first draw dots or vertices torepresent the elements of A. Next, if (a, b) ∈ R we draw an arrow (called adirected edge) from a to b. Finally, if (a, a) ∈ R then the directed edge issimply a loop.

Example 15.7Draw the directed graph of the relation in part (b) of Example 15.5.

Page 125: Discrete Book

15 EQUIVALENCE RELATIONS 121

Solution.

Next we discuss three ways of building new relations from given ones. Let Rbe a relation from a set A to a set B. The inverse of R is the relation R−1

from Range(R) to Dom(R) such that

R−1 = {(b, a) ∈ B × A : (a, b) ∈ R}.

Example 15.8Let R = {(1, y), (1, z), (3, y)} be a relation from A = {1, 2, 3} to B ={x, y, z}.(a) Find R−1.(b) Compare (R−1)−1 and R.

Solution.(a) R−1 = {(y, 1), (z, 1), (y, 3)}.(b) (R−1)−1 = R

Let R and S be two relations from a set A to a set B. Then we definethe relations R ∪ S and R ∩ S by

R ∪ S = {(a, b) ∈ A×B|(a, b) ∈ R or (a, b) ∈ S},

andR ∩ S = {(a, b) ∈ A×B|(a, b) ∈ R and (a, b) ∈ S}.

Example 15.9Given the following two relations from A = {1, 2, 4} to B = {2, 6, 8, 10} :

Page 126: Discrete Book

122 RELATIONS AND FUNCTIONS

aRb if and only if a|b.aSb if and only if b− 4 = a.

List the elements of R, S,R ∪ S, and R ∩ S.

Solution.We have

R ={(1, 2), (1, 6), (1, 8), (1, 10), (2, 2), (2, 6), (2, 8), (2, 10), (4, 8)}S ={(2, 6), (4, 8)}

R ∪ S =R

R ∩ S =S

Now, if we have a relation R from A to B and a relation S from B to Cwe can define the relation S ◦R, called the composition relation, to be therelation from A to C defined by

S ◦R = {(a, c)|(a, b) ∈ R and (b, c) ∈ S for some b ∈ B}.

Example 15.10Let

R ={(1, 2), (1, 6), (2, 4), (3, 4), (3, 6), (3, 8)}S ={(2, u), (4, s), (4, t), (6, t), (8, u)}

Find S ◦R.

Solution.

S ◦R = {(1, u), (1, t), (2, s), (2, t), (3, s), (3, t), (3, u)}

We next define four types of binary relations. A relation R on a set A iscalled reflexive if (a, a) ∈ R for all a ∈ A. In this case, the digraph of R hasa loop at each vertex.

Example 15.11(a) Show that the relation a ≤ b on the set A = {1, 2, 3, 4} is reflexive.(b) Show that the relation on R defined by aRb if and only if a < b is notreflexive.

Page 127: Discrete Book

15 EQUIVALENCE RELATIONS 123

Solution.(a) By Example 12.7, each vertex has a loop.(b) Indeed, for any real number a we have a− a = 0 and not a− a < 0

A relation R on A is called symmetric if whenever (a, b) ∈ R then wemust have (b, a) ∈ R. The digraph of a symmetric relation has the propertythat whenever there is a directed edge from a to b, there is also a directededge from b to a.

Example 15.12(a) Let A = {a, b, c, d} and R = {(a, a), (b, c), (c, b), (d, d)}. Show that R issymmetric.(b) Let R be the set of real numbers and R be the relation aRb if and onlyif a < b. Show that R is not symmetric.

Solution.(a) bRc and cRb so R is symmetric.(b) 2 < 4 but 4 6< 2

A relation R on a set A is called antisymmetric if whenever (a, b) ∈ Rand a 6= b then (b, a) 6∈ R. The digraph of an antisymmetric relation has theproperty that between any two vertices there is at most one directed edge.

Example 15.13(a) Let N be the set of nonnegative integers and R the relation aRb if andonly if a divides b. Show that R is antisymmetric.(b) Let A = {a, b, c, d} and R = {(a, a), (b, c), (c, b), (d, d)}. Show that R isnot antisymmetric.

Solution.(a) Suppose that a|b and b|a. We must show that a = b. Indeed, by the def-inition of division, there exist positive integers k1 and k2 such that b = k1aand a = k2b. This implies that a = k2k1a and hence k1k2 = 1. Since k1 andk2 are positive integers, we must have k1 = k2 = 1. Hence, a = b.(b) bRc and cRb with b 6= c

A relation R on a set A is called transitive if whenever (a, b) ∈ R and(b, c) ∈ R then (a, c) ∈ R. The digraph of a transitive relation has the prop-erty that whenever there are directed edges from a to b and from b to c thenthere is also a directed edge from a to c.

Page 128: Discrete Book

124 RELATIONS AND FUNCTIONS

Example 15.14(a) Let A = {a, b, c, d} and R = {(a, a), (b, c), (c, b), (d, d)}. Show that R isnot transitive.(b) Let Z be the set of integers and R the relation aRb if a divides b. Showthat R is transitive.

Solution.(a) (b, c) ∈ R and (c, b) ∈ R but (b, b) 6∈ R.(b) Suppose that a|b and b|c. Then there exist integers k1 and k2 such thatb = k1a and c = k2b. Thus, c = (k1k2)a which means that a|c

Now, let A1, A2, · · · , An be a partition of a set A. That is, the A′is aresubsets of A that satisfy(i) ∪ni=1Ai = A(ii) Ai ∩ Aj = ∅ for i 6= j.Define on A the binary relation x R y if and only if x and y belongs to thesame set Ai for some 1 ≤ i ≤ n.

Theorem 15.1The relation R defined above is reflexive, symmetric, and transitive.

Proof.• R is reflexive: If x ∈ A then by (i) x ∈ Ak for some 1 ≤ k ≤ n. Thus, xand x belong to Ak so that x R x.• R is symmetric: Let x, y ∈ A such that x R y. Then there is an index ksuch that x, y ∈ Ak. But then y, x ∈ Ak. That is, y R x.• R is transitive: Let x, y, z ∈ A such that x R y and y R z. Then there existindices i and j such that x, y ∈ Ai and y, z ∈ Aj. Since y ∈ Ai ∩ Aj, by (ii)we must have i = j. This implies that x, y, z ∈ Ai and in particular x, z ∈ Ai.Hence, x R z

A relation that is reflexive, symmetric, and transitive on a set A is calledan equivalence relation on A. For example, the relation “=” is an equiv-alence relation on R.

Example 15.15Let Z be the set of integers and n ∈ Z. Let R be the relation on Z definedby aRb if a− b is a multiple of n. We denote this relation by a ≡ b (mod n)read “a congruent to b modulo n.” Show that R is an equivalence relationon Z.

Page 129: Discrete Book

15 EQUIVALENCE RELATIONS 125

Solution.≡ is reflexive: For all a ∈ Z, a− a = 0 · n. That is, a ≡ a (mod n).≡ is symmetric: Let a, b ∈ Z such that a ≡ b (mod n). Then there is aninteger k such that a− b = kn. Multiply both sides of this equality by (−1)and letting k′ = −k we find that b− a = k′n. That is b ≡ a (mod n).≡ is transitive: Let a, b, c ∈ Z be such that a ≡ b (mod n) and b ≡ c (mod n).Then there exist integers k1 and k2 such that a − b = k1n and b − c = k2n.Adding these equalities together we find a − c = kn where k = k1 + k2 ∈ Zwhich shows that a ≡ c (mod n)

Theorem 15.2Let R be an equivalence relation on A. For each a ∈ A let

[a] = {x ∈ A|xRa}

A/R = {[a]|a ∈ A}.

Then the union of all the elements of A/R is equal to A and the intersectionof any two distinct members of A/R is the empty set. That is, the familyA/R forms a partition of A.

Proof.By the definition of [a] we have that [a] ⊆ A. Hence, ∪a∈A[a] ⊆ A. We nextshow that A ⊆ ∪a∈A[a]. Indeed, let a ∈ A. Since A is reflexive, a ∈ [a] andconsequently a ∈ ∪b∈A[b]. Hence, A ⊆ ∪b∈A[b]. It follows that A = ∪a∈A[a].This establishes (i).It remains to show that if [a] 6= [b] then [a]∩ [b] = ∅ for a, b ∈ A. Suppose thecontrary. That is, suppose [a]∩ [b] 6= ∅. Then there is an element c ∈ [a]∩ [b].This means that c ∈ [a] and c ∈ [b]. Hence, a R c and b R c. Since R is sym-metric and transitive, a R b. We will show that the conclusion a R b leads to[a] = [b]. The proof is by double inclusions. Let x ∈ [a]. Then x R a. Sincea R b and R is transitive, x R b which means that x ∈ [b]. Thus, [a] ⊆ [b].Now interchange the letters a and b to show that [b] ⊆ [a]. Hence, [a] = [b]which contradicts our assumption that [a] 6= [b]. This establishes (ii). Thus,A/R is a partition of A

The sets [a] defined in the previous exercise are called the equivalenceclasses of A given by the relation R. The element a in [a] is called a repre-sentative of the equivalence class [a].

Page 130: Discrete Book

126 RELATIONS AND FUNCTIONS

Example 15.16Let R be an equivalence relation on A. Show that if aRb then [a] = [b].

Solution.[a] ⊆ [b] : Let c ∈ [a]. Then cRa. But aRb so that cRb since R is transitive.Hence, c ∈ [b].[b] ⊆ [a] : Let c ∈ [b]. Then cRb. Since R is symmetric, bRa. Hence, cRa sinceR is transitive. Thus, c ∈ [a]

Example 15.17Find the equivalence classes of the the equivalence relation on Z defined bya ≡ b mod 4.

Solution.For any integer a ∈ Z, the congruence class of a is

[a] = {n ∈ Z|n− a = 4k for some k ∈ Z}.

Hence,

[0] ={0,±4,±8,±12, · · · }[1] ={· · · ,−11,−7,−3, 1, 5, 9, · · · }[2] ={· · · ,−10,−6,−2, 2, 6, 10, · · · }[3] ={· · · ,−9,−5,−1, 3, 7, 11, · · · }.

Note that {[0], [1], [2], [3]} is a partition of Z. Also, note that [0] = [±4] =[±8] = · · · ; [1] = [−11] = [−7] = · · · , etc

Page 131: Discrete Book

15 EQUIVALENCE RELATIONS 127

Review Problems

Problem 15.1Suppose that |A| = 3 and |A×B| = 24. What is |B|?

Problem 15.2Let A = {1, 2, 3}. Find A× A.

Problem 15.3Find x and y so that (3x− y, 2x+ 3y) = (7, 1).

Problem 15.4Find the domain and range of the relationR = {(5, 6), (−12, 4), (8, 6), (−6,−6), (5, 4)}.

Problem 15.5Let A = {1, 2, 3} and B = {a, b, c, d}.(a) Is the relation f = {(1, d), (2, d), (3, a)} a function from A to B? If so,find its range.(b) Is the relation f = {(1, a), (2, b), (2, c), (3, d)} a function from A to B? Ifso, find its range.

Problem 15.6Let R be the divisibility relation on the set A = {1, 2, 3, 4, 5, 6, 7, 8, 9}. Con-struct the digraph of R.

Problem 15.7Find the inverse relation of R = {(a, 1), (b, 5), (c, 2), (d, 1)}. Is the inverserelation a function?

Problem 15.8Let

A ={1, 2, 3, 4}B ={1, 2, 3}R ={(, 2), (1, 3), (1, 4), (2, 2), (3, 4), (4, 1), (4, 2)}S ={(1, 1), (1, 2), (1, 3), (2, 3)}.

Find (a) R ∪ S (b) R ∩ S (c) R− S and (d) S −R.

Page 132: Discrete Book

128 RELATIONS AND FUNCTIONS

Problem 15.9Let

A ={a, b, c}B ={1, 2}C ={a, b, g}R ={(a, 1), (a, 2), (b, 2), (c, 1)}S ={(1, a), (2, b), (2, g)}.

Find S ◦R.

Problem 15.10Let X = {a, b, c}. Recall that P(X) is the power set of X. Define a binaryrelation R on P(X) as follows:

A,B ∈ P(x), A R B ⇔ |A| = |B|.

(a) Is {a, b}R{b, c}?(b) Is {a}R{a, b}?(c) Is {c}R{b}?

Problem 15.11Let A = {4, 5, 6} and B = {5, 6, 7} and define the binary relations R, S, andT from A to B as follows:

(x, y) ∈ A×B, (x, y) ∈ R⇔ x ≥ y.

(x, y) ∈ A×B, x S y ⇔ 2|(x− y).

T = {(4, 7), (6, 5), (6, 7)}.

(a) Draw arrow diagrams for R, S, and T.(b) Indicate whether any of the relations S,R, or T are functions.

Problem 15.12Let A = {3, 4, 5} and B = {4, 5, 6} and define the binary relation R asfollows:

(x, y) ∈ A×B, (x, y) ∈ R⇔ x < y.

List the elements of the sets R and R−1.

Page 133: Discrete Book

15 EQUIVALENCE RELATIONS 129

Problem 15.13Let A = {2, 4} and B = {6, 8, 10} and define the binary relations R and Sfrom A to B as follows:

(x, y) ∈ A×B, (x, y) ∈ R⇔ x|y.

(x, y) ∈ A×B, x S y ⇔ y − 4 = x.

List the elements of A×B,R, S,R ∪ S, and R ∩ S.

Problem 15.14Consider the binary relation on R defined as follows:

x, y ∈ R, x R y ⇔ x ≥ y.

Is R reflexive? symmetric? transitive?

Problem 15.15Consider the binary relation on R defined as follows:

x, y ∈ R, x R y ⇔ xy ≥ 0.

Is R reflexive? symmetric? transitive?

Problem 15.16Let A 6= ∅ and P(A) be the power set of A. Consider the binary relation onP(A) defined as follows:

X, Y ∈ P(A), X R Y ⇔ X ⊆ Y.

Is R reflexive? symmetric? transitive?

Problem 15.17Let E be the binary relation on Z defined as follows:

a E b⇔ m ≡ n (mod 2).

Show that E is an equivalence relation on Z and find the different equivalenceclasses.

Page 134: Discrete Book

130 RELATIONS AND FUNCTIONS

Problem 15.18Let I be the binary relation on R defined as follows:

a I b⇔ a− b ∈ Z.

Show that I is an equivalence relation on R and find the different equivalenceclasses.

Problem 15.19Let A be the set all straight lines in the cartesian plane. Let || be the binaryrelation on A defined as follows:

l1||l2 ⇔ l1 is parallel to l2.

Show that || is an equivalence relation on A and find the different equivalenceclasses.

Problem 15.20Let A = N× N. Define the binary relation R on A as follows:

(a, b) R (c, d)⇔ a+ d = b+ c.

(a) Show that R is reflexive.(b) Show that R is symmetric.(c) Show that R is transitive.(d) List five elements in [(1, 1)].(e) List five elements in [(3, 1)].(f) List five elements in [(1, 2)].(g) Describe the distinct equivalence classes of R.

Problem 15.21Let R be a binary relation on a set A and suppose that R is symmetric andtransitive. Prove the following: If for every x ∈ A there is a y ∈ A such thatx R y then R is reflexive and hence an equivalence relation on A.

Page 135: Discrete Book

16 PARTIAL ORDER RELATIONS 131

16 Partial Order Relations

A relation ≤ on a set A is called a partial order if ≤ is reflexive, antisym-metric, and transitive. In this case we call (A,≤) a poset.

Example 16.1Show that the set Z of integers together with the relation of inequality ≤ isa poset.

Solution.≤ is reflexive: For all x ∈ Z we have x ≤ x since x = x.≤ is antisymmetric: By the trichotomy law of real numbers, for a given pairof numbers x and y only one of the following is true: x < y, x = y, or x > y.So if x ≤ y and y ≤ x then we must have x = y.≤ is transitive: If x ≤ y and y ≤ z then x ≤ z

Example 16.2Show that the relation a|b in N = {1, 2, 3, · · · } is a partial order relation.

Solution.Reflexivity: Since a = 1 · a, we have a|a.Antisymmetry: Suppose that a|b and b|a. Then there exist positive integersk1 and k2 such that b = k1a and a = k2b. Hence, a = k1k2a which impliesthat k1k2 = 1. Since k1, k2 ∈ N∗, we must have k1 = k2 = 1; that is, a = b.Transitivity: Suppose that a|b and b|c. Then there exist positive integers k1and k2 such that b = k1a and c = k2b. Thus, c = k1k2a which means thata|c

Example 16.3Let P(X) be the power set of X. Let R be the relation on P(X)defined by

A R B ⇔ A ⊆ B.

Show that P(X) is a poset.

Solution.⊆ is reflexive: For any set A ∈ P(X), A ⊆ A.⊆ is antisymmetric: By the definition of = of sets if A ⊆ B and B ⊆ A thenA = B, where A,B ∈ P(X).

Page 136: Discrete Book

132 RELATIONS AND FUNCTIONS

⊆ is transitive: We have seen in Example 11.1, that if A ⊆ B and B ⊆ Cthen A ⊆ C

Another simple pictorial representation of a partial order is the so calledHasse diagram. The Hasse diagram of a partial order on the set A is adrawing of the points of A and some of the arrows of the digraph of the or-der relation. We assume that the directed edges of the Hasse diagram pointupward. There are rules to determine which arrows are drawn and which areomitted, namely,• omit all arrows that can be inferred from transitivity• omit all loops• draw arrows without “heads”.

Example 16.4Let A = {1, 2, 3, 9, 18} and the “divides” relation on A. Draw the Hassediagram of this relation.

Solution.The directed graph of the given relation and the cooresponding Hasse dia-gram are shown in the figure below

Now, given the Hasse diagram of a partial order relation one can find thedigraph as follows:• reinsert the direction markers on the arrows making all arrows point up-ward• add loops at each vertex

Page 137: Discrete Book

16 PARTIAL ORDER RELATIONS 133

• for each sequence of arrows from one point to a second point and from thatsecond point to a third point, add an arrow from the first point to the third.

Example 16.5Let A = {1, 2, 3, 4} be a poset. Find the directed graph corresponding to thefollowing Hasse diagram on A.

Solution.

Next, if A is a poset then we say that a and b are comparable if eithera ≤ b or b ≤ a. If every pair of elements of A are comparable then we call ≤a total order.

Page 138: Discrete Book

134 RELATIONS AND FUNCTIONS

Example 16.6Consider the “divides” relation defined on the set A = {5, 15, 30}. Prove thatthis relation is a total order on A.

Solution.The fact that the “divides” relation is a partial order is easy to verify. Since5|15, 5|30, and 15|30, any pair of elements in A are comparable. Thus, the“divides” relation is a total order on A

Example 16.7Show that the “divides” relation on N is not a total order.

Solution.A counterexample of two noncomparable numbers are 2 and 3, since 2 doesnot divide 3 and 3 does not divide 2

Let (A,≤) be a poset. An element a ∈ A is called a least element ifand only if a ≤ b for all b ∈ A. Likewise, an element b ∈ A is called agreatest element of A if and only if a ≤ b for all a ∈ A.

Example 16.8(a) Find the least element of the poset (N,≤).(b) Find the least element and the greatest element of the poset (P(A),⊆)where A = {a, b}.

Solution.(a) 1 is the least element of the poset (N,≤).(b) The empty set is the least element of the given poset whereas A is thegreatest element

A partial order R on a set A is a well order if every non-empty subsetof A has a least element.

Example 16.9Show that (N,≤) is well-ordered.

Solution.This is true because every non-empty subset of natural numbers has a leastelement (See Theorem 8.1)

Page 139: Discrete Book

16 PARTIAL ORDER RELATIONS 135

Review Problems

Problem 16.1Define a relation R on Z as follows: for all m,n ∈ Z

m R n⇔ m+ n is even.

Is R a partial order? Prove or give a counterexample.

Problem 16.2Define a relation R on R as follows: for all m,n ∈ R

m R n⇔ m2 ≤ n2.

Is R a partial order? Prove or give a counterexample.

Problem 16.3Let S = {0, 1} and consider the partial order relation R defined on S × S asfollows: for all ordered pairs (a, b) and (c, d) in S × S

(a, b) R (c, d)⇔ a ≤ c and b ≤ d.

Draw the Hasse diagram for R.

Problem 16.4Consider the “divides” relation defined on the set A = {1, 2, 22, · · · , 2n},where n is a nonnegative integer.a. Prove that this relation is a total order on A.b. Draw the Hasse diagram for this relation when n = 3.

Problem 16.5Let R be a partial order on A. Show that R−1 is also a partial order on A.

Problem 16.6An order relation R is given by the following Hass-diagram. List all the ele-ments of R = {(x, y)|xRy}.

Page 140: Discrete Book

136 RELATIONS AND FUNCTIONS

.

Problem 16.7Let F be a collection of finite sets. On this set, we define the relation R by

A R B ⇔ |A| ≤ |B|.

Show that R is reflexive, transitive but is neither anti-symmetric nor sym-metric.

Problem 16.8Let A = N× N and define R on A by

(a, b)R(c, d)⇔ a ≤ c and b ≥ d.

Show that (A,R) is a poset.

Problem 16.9LetA = {a, b, c} and consider the poset (P(A),⊆). Let S = {{a}, {a, c}, {a, b}}.Find the least element and the greatest element of S.

Problem 16.10Show that the relation ≤ on Z is not well-ordered.

Problem 16.11Show that if A is well-ordered under ≤ then ≤ is a total order on A.

Problem 16.12Suppose that (A,≤) is a poset and S is a non-empty subset of A. If S has aleast element then this element is unique.

Page 141: Discrete Book

17 BIJECTIVE AND INVERSE FUNCTIONS 137

17 Bijective and Inverse Functions

Let f : A → B be a function. We say that f is injective or one-to-one ifand only if for all x, y ∈ A, if f(x) = f(y) then x = y. Using the concept ofcontrapositive, a function f is injective if and only if for all x, y ∈ A, if x 6= ythen f(x) 6= f(y). Taking the negation of this last conditional implicationwe see that f is not injective if and only if there exist two distinct elementsa and b of A such that f(a) = f(b) (Example 2.4(a)).

Example 17.1(a) Show that the identity function IA on a set A is injective.(b) Show that the function f : Z→ Z defined by f(n) = n2 is not injective.

Solution.(a) Let x, y ∈ A. If IA(x) = IA(y) then x = y by the definition of IA. Thisshows that IA is injective.(b) Since 12 = (−1)2 and 1 6= −1, f is not injective

Example 17.2 (Hash Functions)Let m > 1 be a positive integer . Show that the function h : Z→ Z definedby h(n) = n mod m is not injective.

Solution.Indeed, since m > 1, we have 2m+1 6= m+1 and h(m+1) = h(2m+1) = 1.So h is not injective

Example 17.3Show that if f : R→ R is increasing then f is one-to-one.

Solution.Suppose that x1 6= x2. Then without loss of generality we can assume thatx1 < x2. Since f is increasing, f(x1) < f(x2). That is, f(x1) 6= f(x2). Hence,f is one-to-one

Example 17.4Show that the composition of two injective functions is also injective.

Page 142: Discrete Book

138 RELATIONS AND FUNCTIONS

Solution.Let f : A→ B and g : B → C be two injective functions. We will show thatg ◦ f : A→ C is also injective. Indeed, suppose that (g ◦ f)(x1) = (g ◦ f)(x2)for x1, x2 ∈ A. Then g(f(x1)) = g(f(x2)). Since g is injective, f(x1) = f(x2).Now, since f is injective, x1 = x2. This completes the proof that g ◦ f isinjective

Now, for any function f : A → B we have Range(f) ⊆ B. If equality holdsthen we say that f is surjective or onto. It follows from this definition thata function f is surjective if and only if for each y ∈ B there is an x ∈ A suchthat f(x) = y. By taking the negation of this we see that f is not onto ifthere is a y ∈ B such that f(x) 6= y for all x ∈ A.

Example 17.5(a) Show that the function f : R→ R defined by f(x) = 3x− 5 is surjective.(b) Show that the function f : Z → Z defined by f(n) = 3n − 5 is notsurjective.

Solution.(a) Let y ∈ R. Is there an x ∈ R such that f(x) = y? That is, 3x − 5 = y.But solving for x we find x = y+5

3∈ R and f(x) = y. Thus, f is onto.

(b) Take m = 3. If f is onto then there should be an n ∈ Z such thatf(n) = 3. That is, 3n − 5 = 3. Solving for n we find n = 8

3which is not an

integer. Hence, f is not onto

Example 17.6 (Projection Functions)Let A and B be two nonempty sets. The functions prA : A×B → A definedby prA(a, b) = a and prB : A × B → B defined by prB(a, b) = b are calledprojection functions. Show that prA and prB are surjective functions.

Solution.We prove that prA is surjective. Indeed, let a ∈ A. Since B is not empty,there is a b ∈ B. But then (a, b) ∈ A × B and prA(a, b) = a. Hence, prA issurjective. The proof that prB is surjective is similar

Example 17.7Show that the composition of two surjective functions is also surjective.

Page 143: Discrete Book

17 BIJECTIVE AND INVERSE FUNCTIONS 139

Solution.Let f : A → B and g : B → C, where Range(f) ⊆ C, be two surjectivefunctions. We will show that g ◦ f : A → D is also surjective. Indeed, letz ∈ D. Since g is surjective, there is a y ∈ B such that g(y) = z. Since f issurjective, there is an x ∈ A such that f(x) = y. Thus, g(f(x)) = z. Thisshows that g ◦ f is surjective

Now, we say that a function f is bijective or one-to-one correspondenceif and only if f is both injective and surjective. A bijective function on a setA is called a permutation.

Example 17.8(a) Show that the function f : R→ R defined by f(x) = 3x− 5 is a bijectivefunction.(b) Show that the function f : R→ R defined by f(x) = x2 is not bijective.

Solution.(a) First we show that f is injective. Indeed, suppose that f(x1) = f(x2).Then 3x1 − 5 = 3x2 − 5 and this implies that x1 = x2. Hence, f is injective.f is surjective by Example 14.5 (a).(b) f is not injective since f(−1) = f(1) but −1 6= 1. Hence, f is notbijective

Example 17.9Show that the composition of two bijective functions is also bijective.

Solution.This follows from Example 17.4 and Example 17.7

Theorem 17.1Let f : X → Y be a bijective function. Then there is a function f−1 : Y → Xwith the following properties:

(a) f−1(y) = x if and only if f(x) = y.(b) f−1 ◦ f = IX and f ◦ f−1 = IY where IX denotes the identity functionon X.(c) f−1 is bijective.

Page 144: Discrete Book

140 RELATIONS AND FUNCTIONS

Proof.For each y ∈ Y there is a unique x ∈ X such that f(x) = y since f isbijective. Thus, we can define a function f−1 : Y → X by f−1(y) = x wheref(x) = y.(a) Follows from the definition of f−1.(b) Indeed, let x ∈ X such that f(x) = y. Then f−1(y) = x and (f−1◦f)(x) =f−1(f(x)) = f−1(y) = x = IX(x). Since x was arbitrary, f−1 ◦ f = IX . Theproof that f ◦ f−1 = IY is similar.(c) We show first that f−1 is injective. Indeed, suppose f−1(y1) = f−1(y2).Then f(f−1(y1)) = f(f−1(y2)); that is, (f ◦ f−1)(y1) = (f ◦ f−1)(y2). Byb. we have IY (y1) = IY (y2). From the definition of IY we obtain y1 = y2.Hence, f−1 is injective. We next show that f−1 is surjective. Indeed, lety ∈ Y . Since f is onto, there is a unique x ∈ X such that f(x) = y. Bythe definition of f−1, f−1(y) = x. Thus, for every element y ∈ Y there is anelement x ∈ X such that f−1(y) = x. This says that f−1 is surjective andcompletes a proof of the theorem

Example 17.10Show that f : R→ R defined by f(x) = 3x−5 is bijective and find a formulafor its inverse function.

Solution.We have already proved that f is bijective. We will just find the formulafor its inverse function f−1. Indeed, if y ∈ Y we want to find x ∈ X suchthat f−1(y) = x, or equivalently, f(x) = y. This implies that 3x− 5 = y andsolving for x we find x = y+5

3. Thus, f−1(y) = y+5

3

Finally, we say that two sets A and B have the same cardinality if thereis a one-to-one correspondence from A to B.

Page 145: Discrete Book

17 BIJECTIVE AND INVERSE FUNCTIONS 141

Review Problems

Problem 17.1(a) Define g : Z→ Z by g(n) = 3n− 2.(i) Is g one-to-one? Prove or give a counterexample.(ii) Is g onto? Prove or give a counterexample.(b) Define G : R → R by G(x) = 3x − 2. Is G onto? Prove or give acounterexample.

Problem 17.2Determine whether the function f : R→ R given by f(x) = x+1

xis one-to-one

or not.

Problem 17.3Determine whether the function f : R → R given by f(x) = x

x2+1is one-to-

one or not.

Problem 17.4Let f : R→ Z be the floor function f(x) = bxc.(a) Is f one-to-one? Prove or give a counterexample.(b) Is f onto? Prove or give a counterexample.

Problem 17.5If f : R→ R and g : R→ R are one-to-one functions, is f+g also one-to-one?Justify your answer.

Problem 17.6Define F : P{a, b, c} → N to be the number of elements of a subset ofP{a, b, c}.(a) Is F one-to-one? Prove or give a counterexample.(b) Is F onto? Prove or give a counterexample.

Problem 17.7If f : R → R and g : R → R are onto functions, is f + g also onto? Justifyyour answer.

Problem 17.8Show that the function F−1 : R → R given by F−1(y) = y−2

3is the inverse

of the function F (x) = 3x+ 2.

Page 146: Discrete Book

142 RELATIONS AND FUNCTIONS

Problem 17.9If f : X → Y and g : Y → Z are functions and g ◦ f : X → Z is one-to-one,must both f and g be one-to-one? Prove or give a counterexample.

Problem 17.10If f : X → Y and g : Y → Z are functions and g ◦ f : X → Z is onto, mustboth f and g be onto? Prove or give a counterexample.

Problem 17.11If f : X → Y and g : Y → Z are functions and g ◦ f : X → Z is one-to-one,must f be one-to-one? Prove or give a counterexample.

Problem 17.12If f : X → Y and g : Y → Z are functions and g ◦ f : X → Z is onto, mustg be onto? Prove or give a counterexample.

Problem 17.13Let f : W → X, g : X → Y and h : Y → Z be functions. Must h ◦ (g ◦ f) =(h ◦ g) ◦ f? Prove or give a counterexample.

Problem 17.14Let f : X → Y and g : Y → Z be two bijective functions. Show that (g◦f)−1

exists and (g ◦ f)−1 = f−1 ◦ g−1.

Page 147: Discrete Book

18 RECURSION 143

18 Recursion

A recurrence relation for a sequence a0, a1, · · · is a relation that definesan in terms of a0, a1, · · · , an−1. The formula relating an to earlier values inthe sequence is called the generating rule. The assignment of a value toone of the a’s is called an initial condition.

Example 18.1The Fibonacci sequence

1, 1, 2, 3, 5, · · ·is a sequence in which every number after the first two is the sum of thepreceding two numbers. Find the generating rule and the initial conditions.

Solution.The initial conditions are a0 = a1 = 1 and the generating rule is an =an−1 + an−2, n ≥ 2

A solution to a recurrence relation is an explicit formula for an in termsof n.The most basic method for finding the solution of a sequence defined recur-sively is by using iteration. The iteration method consists of starting withthe initial values of the sequence and then calculate successive terms of thesequence until a pattern is observed. At that point one guesses an explicitformula for the sequence and then uses mathematical induction to prove itsvalidity.

Example 18.2Find a solution for the recurrence relation

a0 =1

an =an−1 + 2, n ≥ 1.

Solution.Listing the first five terms of the sequence one finds

a0 =1

a1 =1 + 2

a2 =1 + 4

a3 =1 + 6

a4 =1 + 8

Page 148: Discrete Book

144 RELATIONS AND FUNCTIONS

Hence, a guess is an = 2n+ 1, n ≥ 0. It remains to show that this formula isvalid by using mathematical induction.Basis of induction: For n = 0, a0 = 1 = 2(0) + 1.Induction hypothesis: Suppose that an = 2n+ 1.Induction step: We must show that an+1 = 2(n+ 1) + 1. By the definition ofan+1 we have an+1 = an + 2 = 2n+ 1 + 2 = 2(n+ 1) + 1

Example 18.3Consider the arithmetic sequence

an = an−1 + d, n ≥ 1

where a0 is the initial value. Find an explicit formula for an.

Solution.Listing the first four terms of the sequence after a0 we find

a1 =a0 + d

a2 =a0 + 2d

a3 =a0 + 3d

a4 =a0 + 4d.

Hence, a guess is an = a0 + nd. Next, we prove the validity of this formulaby induction:Basis of induction: For n = 0, a0 = a0 + (0)d.Induction hypothesis: Suppose that an = a0 + nd.Induction step: We must show that an+1 = a0 + (n + 1)d. By the definitionof an+1 we have an+1 = an + d = a0 + nd+ d = a0 + (n+ 1)d

Example 18.4Consider the geometric sequence

an = ran−1, n ≥ 1

where a0 is the initial value. Find an explicit formula for an.

Solution.Listing the first four terms of the sequence after a0 we find

a1 =ra0

a2 =r2a0

a3 =r3a0

a4 =r4a0.

Page 149: Discrete Book

18 RECURSION 145

Hence, a guess is an = rna0. Next, we prove the validity of this formula byinduction.Basis of induction: For n = 0, a0 = r0a0.Induction hypothesis: Suppose that an = rna0.Induction step: We must show that an+1 = rn+1a0. By the definition of an+1

we have an+1 = ran = r(rna0) = rn+1a0

Example 18.5Find a solution to the recurrence relation

a0 =0

an =an−1 + (n− 1), n ≥ 1.

Solution.Writing the first five terms of the sequence we find

a0 =0

a1 =0

a2 =0 + 1

a3 =0 + 1 + 2

a4 =0 + 1 + 2 + 3.

We guess that

an = 0 + 1 + 2 + · · ·+ (n− 1) =n(n− 1)

2.

We next show that the formula is valid by using induction on n ≥ 0.Basis of induction: a0 = 0 = 0(0−1)

2.

Induction hypothesis: Suppose that an = n(n−1)2

.

Induction step: We must show that an+1 = n(n+1)2

. Indeed,

an+1 =an + n

=n(n− 1)

2+ n

=n(n+ 1)

2

Page 150: Discrete Book

146 RELATIONS AND FUNCTIONS

When iteration does not apply, other methods are available for finding ex-plicit formulas for special classes of recursively defined sequences. The methodexplained below works for sequences of the form

an = Aan−1 +Ban−2 (18.1)

where n is greater than or equal to some fixed nonnegative integer k and Aand B are real numbers with B 6= 0. Such an equation is called a second-order linear homogeneous recurrence relation with constant coeffi-cients.

Example 18.6Does the Fibonacci sequence satisfy a second-order linear homogeneous re-lation with constant coefficients?

Solution.Recall that the Fibonacci sequence is defined recursively by an = an−1 +an−2for n ≥ 2 and a0 = a1 = 1. Thus, an satisfies a second-order linear homoge-neous relation with A = B = 1

The following theorem gives a technique for finding solutions to (18.1).

Theorem 18.1Equation (18.1) is satisfied by the sequence 1, t, t2, · · · , tn, · · · where t 6= 0 ifand only if t is a solution to the characteristic equation

t2 − At−B = 0. (18.2)

Proof.(=⇒): Suppose that t is a nonzero real number such that the sequence1, t, t2, · · · satisfies (18.1). We will show that t satisfies the equation t2 −At−B = 0. Indeed, for n ≥ k we have

tn = Atn−1 +Btn−2.

Since t 6= 0 we can divide through by tn−2 and obtain t2 − At−B = 0.(⇐=) : Suppose that t is a nonzero real number such that t2 − At− B = 0.Multiply both sides of this equation by tn−2 to obtain

tn = Atn−1 +Btn−2.

This says that the sequence 1, t, t2, · · · satisfies (18.1)

Page 151: Discrete Book

18 RECURSION 147

Example 18.7Consider the recurrence relation

an = an−1 + 2an−2, n ≥ 2.

Find two sequences that satisfy the given generating rule and have the form1, t, t2, · · · .

Solution.According to the previous theorem t must satisfy the characteristic equation

t2 − t− 2 = 0.

Solving for t we find t = 2 or t = −1. So the two solutions to the givenrecurrence sequence are 1, 2, 22, · · · , 2n, · · · and 1,−1, · · · , (−1)n, · · ·

Are there other solutions than the ones provided by Theorem 22.1? Theanswer is yes according to the following theorem.

Theorem 18.2If sn and tn are solutions to (18.1) then for any real numbers C and D thesequence

an = Csn +Dtn, n ≥ 0

is also a solution.

Proof.Since sn and tn are solutions to (18.1), for n ≥ 2 we have

sn =Asn−1 +Bsn−2

tn =Atn−1 +Btn−2.

Therefore,

Aan−1 +Ban−2 =A(Csn−1 +Dtn−1) +B(Csn−2 +Dtn−2)

=C(Asn−1 +Bsn−2) +D(Atn−1 +Btn−2)

=Csn +Dtn = an

so that an satisfies (18.1)

Page 152: Discrete Book

148 RELATIONS AND FUNCTIONS

Example 18.8Find a solution to the recurrence relation

a0 =1, a1 = 8

an =an−1 + 2an−2, n ≥ 2.

Solution.By the previous theorem and Example 18.7, an = C2n +D(−1)n, n ≥ 2 is asolution to the recurrence relation

an = an−1 + 2an−2.

If an satisfies the system then we must have

a0 = C20 +D(−1)0

a1 = C21 +D(−1)1.

This yields the system {C +D = 12C −D = 8.

Solving this system to find C = 3 and D = −2. Hence, an = 3 ·2n−2(−1)n

Example 18.9Find an explicit formula for the Fibonacci sequence

a0 =a1 = 1

an =an−1 + an−2.

Solution.The roots of the characteristic equation

t2 − t− 1 = 0

are t = 1−√5

2and t = 1+

√5

2. Thus,

an = C(1 +√

5

2)n +D(

1−√

5

2)n

is a solution toan = an−1 + an−2.

Page 153: Discrete Book

18 RECURSION 149

Using the values of a0 and a1 we obtain the system{C +D = 1

C(1+√5

2) +D(1−

√5

2) = 1.

Solving this system to obtain

C =1 +√

5

2√

5and D = −1−

√5

2√

5.

Hence,

an =1√5

(1 +√

5

2)n+1 − 1√

5(1−√

5

2)n+1

Next, we discuss the case when the characteristic equation has a single root.

Theorem 18.3Let A and B be real numbers and suppose that the characteristic equation

t2 − At−B = 0

has a single root r. Then the sequences {1, r, r2, · · · } and {0, r, 2r2, 3r3, · · · , nrn, · · · }both satisfy the recurrence relation

an = Aan−1 +Ban−2.

Proof.Since r is a root to the characteristic equation, the sequence {1, r, r2, · · · } isa solution to the recurrence relation

an = Aan−1 +Ban−2.

Now, since r is the only solution to the characteristic equation we have

(t− r)2 = t2 − At−B.

This implies that A = 2r and B = −r2. Let sn = nrn, n ≥ 0. Then

Asn−1 +Bsn−2 =A(n− 1)rn−1 +B(n− 2)rn−2

=2r(n− 1)rn−1 − r2(n− 2)rn−2

=2(n− 1)rn − (n− 2)rn

=nrn = sn

So sn is a solution to an = Aan−1 +Ban−2.

Page 154: Discrete Book

150 RELATIONS AND FUNCTIONS

Example 18.10Find an explicit formula for

a0 =1, a1 = 3

an =4an−1 − 4an−2, n ≥ 2

Solution.Solving the characteristic equation

t2 − 4t+ 4 = 0

we find the single root r = 2. Thus,

an = C2n +Dn2n

is a solution to the equation an = 4an−1 − 4an−2. Since a0 = 1 and a1 = 3,we obtain the following system of equations:

C =1

2C + 2D =3

Solving this system to obtain C = 1 and D = 12. Hence, an = 2n + n

22n.

Example 18.11Let A1, A2, · · · , An be subsets of a set S.(a) Give a recursion definition for ∪ni=1Ai.(b) Give a recursion definition for ∩ni=1Ai.

Solution.(a) ∪1i=1Ai = A1 and ∪ni=1Ai = (∪n−1i=1 Ai) ∪ An, n ≥ 2.(b) ∩1

i=1Ai = A1 and ∩ni=1Ai = (∩n−1i=1 Ai) ∩ An, n ≥ 2.

Example 18.12Use mathematical induction to prove the following generalized De Morgan’slaw.

(∪ni=1Ai)c = ∩ni=1A

ci

Solution.Basis of induction: (∪1

i=1Ai)c = Ac

1 = ∩1i=1Aci .

Page 155: Discrete Book

18 RECURSION 151

Induction hypothesis: Suppose that (∪ni=1Ai)c = ∩ni=1A

ci .

Induction step: We must show that (∪n+1i=1 Ai)

c = ∩n+1i=1 A

ci . Indeed,

(∪n+1i=1 Ai)

c =((∪ni=1Ai) ∪ An+1)c

=(∪ni=1Ai)

c) ∩ Acn+1

=(∩ni=1A

ci) ∩ Ac

n+1

= ∩n+1i=1 A

ci

Example 18.13Let a1, a2, · · · , an be numbers.(a) Give a recursion definition for

∑ni=1 ai.

(b) Give a recursion definition for Πni=1ai.

Solution.(a)∑1

i=1 ai = a1 and∑n

i=1 ai = (∑n−1

i=1 ai) + an, n ≥ 2.(b) Π1

i=1ai = a1 and Πni=1ai = (Πn−1

i=1 ai) · an, n ≥ 2.

Example 18.14A function is said to be defined recursively or to be a recursive functionif its rule of definition refers to itself. Define the factorial function recursively.

Solution.We have

f(0) =1

f(n) =nf(n− 1), n ≥ 1

Example 18.15Let G : N→ Z be the relation given by

G(n) =

1, if n = 1

1 +G(n2), if n is even

G(3n− 1), if n > 1 is odd

Show that G is not a function.

Solution.Assume that G is a function so that G(5) exists. Listing the first five values

Page 156: Discrete Book

152 RELATIONS AND FUNCTIONS

of G we find

G(1) =1

G(2) =2

G(3) =G(8) = 1 +G(4) = 2 +G(2) = 4

G(4) =1 +G(2) = 3

G(5) =G(14) = 1 +G(7)

=1 +G(20)

=2 +G(10)

=3 +G(5)

But the last equality implies that 0 = 3 which is impossible. Hence, G doesnot define a function.

Page 157: Discrete Book

18 RECURSION 153

Review Problems

Problem 18.1Find the first four terms of the following recursively defined sequence:

v1 =1, v2 = 2

vn =vn−1 + vn−2 + 1, n ≥ 3.

Problem 18.2Prove each of the following for the Fibonacci sequence:(a) F 2

k − F 2k−1 = FkFk+1 − Fk+1Fk−1, k ≥ 1.

(b) F 2k+1 − F 2

k − F 2k−1 = 2FkFk−1, k ≥ 1.

(c) F 2k+1 − F 2

k = Fk−1Fk+2, k ≤ 1.(d) Fn+2Fn − F 2

n+1 = (−1)n for all n ≥ 0.

Problem 18.3Find limn→∞

Fn+1

Fnwhere F0, F1, F2, · · · is the Fibonacci sequence. (Assume

that the limit exists.)

Problem 18.4Define x0, x1, x2, · · · as follows:

xn =√

2 + xn−1, x0 = 0.

Find limn→∞ xn.

Problem 18.5Find a formula for each of the following sums:(a) 1 + 2 + · · ·+ (n− 1), n ≥ 2.(b) 3 + 2 + 4 + 6 + 8 + · · ·+ 2n, n ≥ 1.(c) 3 · 1 + 3 · 2 + 3 · 3 + · · · 3 · n, n ≥ 1.

Problem 18.6Find a formula for each of the following sums:(a) 1 + 2 + 22 + · · ·+ 2n−1, n ≥ 1.(b) 3n−1 + 3n−2 + · · ·+ 32 + 3 + 1, n ≥ 1.(c) 2n + 3 · 2n−2 + 3 · 2n−3 + · · ·+ 3 · 22 + 3 · 2 + 3, n ≥ 1.(d) 2n − 2n−1 + 2n−2 − 2n−3 + · · ·+ (−1)n−1 · 2 + (−1)n, n ≥ 1.

Page 158: Discrete Book

154 RELATIONS AND FUNCTIONS

Problem 18.7Use iteration to guess a formula for the following recursively defined sequenceand then use mathematical induction to prove the validity of your formula:c1 = 1, cn = 3cn−1 + 1, for all n ≥ 2.

Problem 18.8Use iteration to guess a formula for the following recursively defined sequenceand then use mathematical induction to prove the validity of your formula:w0 = 1, wn = 2n − wn−1, for all n ≥ 2.

Problem 18.9Determine whether the recursively defined sequence: a1 = 0 and an = 2an−1+n− 1 satisfies the recursive formula an = (n− 1)2, n ≥ 1.

Problem 18.10Which of the following are second-order homogeneous recurrence relationswith constant coefficients?(a) an = 2an−1 − 5an−2.(b) bn = nbn−1 + bn−2.(c) cn = 3cn−1 · c2n−2.(d) dn = 3dn−1 + dn−2.(e) rn = rn−1 − rn−2 − 2.(f) sn = 10sn−2.

Problem 18.11Let a0, a1, a2, · · · be the sequence defined by the recursive formula

an = C · 2n +D, n ≥ 0

where C and D are real numbers.(a) Find C and D so that a0 = 1 and a1 = 3. What is a2 in this case?(b) Find C and D so that a0 = 0 and a1 = 2. What is a2 in this case?

Problem 18.12

Let a0, a1, a2, · · · be the sequence defined by the recursive formula

an = C · 2n +D, n ≥ 0

where C and D are real numbers. Show that for any choice of C and D,

an = 3an−1 − 2an−2, n ≥ 2.

Page 159: Discrete Book

18 RECURSION 155

Problem 18.13Let a0, a1, a2, · · · be the sequence defined by the recursive formula

a0 =1, a1 = 2

an =2an−1 + 3an−2, n ≥ 2.

Find an explicit formula for the sequence.

Problem 18.14Let a0, a1, a2, · · · be the sequence defined by the recursive formula

a0 =1, a1 = 4

an =2an−1 − an−2, n ≥ 2.

Find an explicit formula for the sequence.

Problem 18.15The triangle inequality for absolute value states that for all real numbers aand b, |a+b| ≤ |a|+|b|. Use the recursive definition of summation, the triangleinequality, the definition of absolute value, and mathematical induction toprove that for all positive integers n, if a1, a2, · · · , an are real numbers then

|n∑

k=1

ak| ≤n∑

k=1

|ak|.

Problem 18.16Use the recursive definition of union and intersection to prove the followinggeneral distributive law: For all positive integers n, if A and B1, B2, · · · , Bn

are sets thenA ∩ (∪nk=1Bk) = ∪n

k=1(A ∩Bk).

Problem 18.17Use mathematical induction to prove the following generalized De Morgan’slaw.

(∩ni=1Ai)c = ∪ni=1A

ci

Problem 18.18Show that the relation F : N→ Z given by the rule

F (n) =

1 if n = 1.

F (n2) if n is even

1− F (5n− 9) if n is odd and n > 1

does not define a function.

Page 160: Discrete Book

156 RELATIONS AND FUNCTIONS

Page 161: Discrete Book

Fundamentals of Counting andProbability

The major goal of this chapter is to establish several (combinatorial) tech-niques for counting large finite sets without actually listing their elements.These techniques provide effective methods for counting the size of events,an important concept in probability theory.

157

Page 162: Discrete Book

158 FUNDAMENTALS OF COUNTING AND PROBABILITY

19 The Fundamental Principle of Counting

Sometimes one encounters the question of listing all the outcomes of a certainexperiment. One way for doing that is by constructing a so-called treediagram.

Example 19.1List all two-digit numbers that can be constructed from the digits 1,2, and3.

Solution.

The different numbers are {11, 12, 13, 21, 22, 23, 31, 32, 33}

Of course, trees are manageable as long as the number of outcomes is notlarge. If there are many stages to an experiment and several possibilitiesat each stage, the tree diagram associated with the experiment would be-come too large to be manageable. For such problems the counting of theoutcomes is simplified by means of algebraic formulas. The commonly usedformula is the Fundamental Principle of Counting, also known as themultiplication rule of counting, which states:

Theorem 19.1If a choice consists of k steps, of which the first can be made in n1 ways,for each of these the second can be made in n2 ways,· · · , and for each ofthese the kth can be made in nk ways, then the whole choice can be made inn1 · n2 · · · ·nk ways.

Page 163: Discrete Book

19 THE FUNDAMENTAL PRINCIPLE OF COUNTING 159

Proof.In set-theoretic term, we let Si denote the set of outcomes for the ith task,i = 1, 2, · · · , k. Note that n(Si) = ni. Then the set of outcomes for the entirejob is the Cartesian product S1 × S2 × · · · × Sk = {(s1, s2, · · · , sk) : si ∈Si, 1 ≤ i ≤ k}. Thus, we just need to show that

n(S1 × S2 × · · · × Sk) = n(S1) · n(S2) · · ·n(Sk).

The proof is by induction on k ≥ 2.

Basis of InductionThis is just Example 15.3(a).Induction HypothesisSuppose

n(S1 × S2 × · · · × Sk) = n(S1) · n(S2) · · ·n(Sk).

Induction ConclusionWe must show

n(S1 × S2 × · · · × Sk+1) = n(S1) · n(S2) · · ·n(Sk+1).

To see this, note that there is a one-to-one and onto correspondence be-tween the sets S1 × S2 × · · · × Sk+1 and (S1 × S2 × · · ·Sk) × Sk+1 given byf(s1, s2, · · · , sk, sk+1) = ((s1, s2, · · · , sk), sk+1). Thus, n(S1×S2×· · ·×Sk+1) =n((S1×S2×· · ·Sk)×Sk+1) = n(S1×S2×· · ·Sk)n(Sk+1) ( by Example 15.3(a)).Now, applying the induction hypothesis gives

n(S1 × S2 × · · ·Sk × Sk+1) = n(S1) · n(S2) · · ·n(Sk+1)

Example 19.2The following three factors were considered in the study of the effectivenenssof a certain cancer treatment:

(i) Medicine (A1, A2, A3, A4, A5)(ii) Dosage Level (Low, Medium, High)(iii) Dosage Frequency (1,2,3,4 times/day)

Find the number of ways that a cancer patient can be given the medecine?

Page 164: Discrete Book

160 FUNDAMENTALS OF COUNTING AND PROBABILITY

Solution.The choice here consists of three stages, that is, k = 3. The first stage, can bemade in n1 = 5 different ways, the second in n2 = 3 different ways, and thethird in n3 = 4 ways. Hence, the number of possible ways a cancer patientcan be given medecine is n1 · n2 · n3 = 5 · 3 · 4 = 60 different ways

Example 19.3How many license-plates with 3 letters followed by 3 digits exist?

Solution.A 6-step process: (1) Choose the first letter, (2) choose the second letter,(3) choose the third letter, (4) choose the first digit, (5) choose the seconddigit, and (6) choose the third digit. Every step can be done in a number ofways that does not depend on previous choices, and each license plate canbe specified in this manner. So there are 26 · 26 · 26 · 10 · 10 · 10 = 17, 576, 000ways

Example 19.4How many numbers in the range 1000 - 9999 have no repeated digits?

Solution.A 4-step process: (1) Choose first digit, (2) choose second digit, (3) choosethird digit, (4) choose fourth digit. Every step can be done in a numberof ways that does not depend on previous choices, and each number can bespecified in this manner. So there are 9 · 9 · 8 · 7 = 4, 536 ways

Example 19.5How many license-plates with 3 letters followed by 3 digits exist if exactlyone of the digits is 1?

Solution.In this case, we must pick a place for the 1 digit, and then the remainingdigit places must be populated from the digits {0, 2, · · · 9}. A 6-step process:(1) Choose the first letter, (2) choose the second letter, (3) choose the thirdletter, (4) choose which of three positions the 1 goes, (5) choose the firstof the other digits, and (6) choose the second of the other digits. Everystep can be done in a number of ways that does not depend on previouschoices, and each license plate can be specified in this manner. So there are26 · 26 · 26 · 3 · 9 · 9 = 4, 270, 968 ways

Page 165: Discrete Book

19 THE FUNDAMENTAL PRINCIPLE OF COUNTING 161

Review Problems

Problem 19.1If each of the 10 digits 0-9 is chosen at random, how many ways can youchoose the following numbers?(a) A two-digit code number, repeated digits permitted.(b) A three-digit identification card number, for which the first digit cannotbe a 0. Repeated digits permitted.(c) A four-digit bicycle lock number, where no digit can be used twice.(d) A five-digit zip code number, with the first digit not zero. Repeateddigits permitted.

Problem 19.2(a) If eight cars are entered in a race and three finishing places are considered,how many finishing orders can they finish? Assume no ties.(b) If the top three cars are Buick, Honda, and BMW, in how many possibleorders can they finish?

Problem 19.3You are taking 2 shirts(white and red) and 3 pairs of pants (black, blue, andgray) on a trip. How many different choices of outfits do you have?

Problem 19.4A Poker club has 10 members. A president and a vice-president are to beselected. In how many ways can this be done if everyone is eligible?

Problem 19.5In a medical study, patients are classified according to whether they haveregular (RHB) or irregular heartbeat (IHB) and also according to whethertheir blood pressure is low (L), normal (N), or high (H). Use a tree diagramto represent the various outcomes that can occur.

Problem 19.6If a travel agency offers special weekend trips to 12 different cities, by air,rail, bus, or sea; in how many different ways can such a trip be arranged?

Problem 19.7If twenty different types of wine are entered in wine-tasting competition, inhow many different ways can the judges award a first prize and a secondprize?

Page 166: Discrete Book

162 FUNDAMENTALS OF COUNTING AND PROBABILITY

Problem 19.8In how many ways can the 24 members of a faculty senate of a college choosea president, a vice-president, a secretary, and a treasurer?

Problem 19.9Find the number of ways in which four of ten new novels can be ranked first,second, third, and fourth according to their figure sales for the first threemonths.

Problem 19.10How many ways are there to seat 8 people, consisting of 4 couples, in a rowof seats (8 seats wide) if all couples are to get adjacent seats?

Page 167: Discrete Book

20 PERMUTATIONS 163

20 Permutations

Consider the following problem: In how many ways can 8 horses finish in arace (assuming there are no ties)? We can look at this problem as a decisionconsisting of 8 steps. The first step is the possibility of a horse to finish firstin the race, the second step is the possibility of a horse to finish second, · · · ,the 8th step is the possibility of a horse to finish 8th in the race. Thus, bythe Fundamental Principle of Counting there are

8 · 7 · 6 · 5 · 4 · 3 · 2 · 1 = 40, 320 ways.

This problem exhibits an example of an ordered arrangement, that is, theorder the objects are arranged is important. Such an ordered arrangement iscalled a permutation. Products such as 8 · 7 · 6 · 5 · 4 · 3 · 2 · 1 can be writtenin a shorthand notation called factorial. That is, 8 · 7 · 6 · 5 · 4 · 3 · 2 · 1 = 8!(read “8 factorial”). In general, we define n factorial by

n! = n(n− 1)(n− 2) · · · 3 · 2 · 1, n ≥ 1

where n is a whole number. By convention we define

0! = 1

Example 20.1Evaluate the following expressions: (a) 6! (b) 10!

7!.

Solution.(a) 6! = 6 · 5 · 4 · 3 · 2 · 1 = 720(b) 10!

7!= 10·9·8·7·6·5·4·3·2·1

7·6·5·4·3·2·1 = 10 · 9 · 8 = 720

Using factorials and the Fundamental Principle of Counting, we see thatthe number of permutations of n objects is n!.

Example 20.2There are 5! permutations of the 5 letters of the word “rehab.” In how manyof them is h the second letter?

Solution.Then there are 4 ways to fill the first spot. The second spot is filled by theletter h. There are 3 ways to fill the third, 2 to fill the fourth, and one wayto fill the fifth. There are 4! such permutations

Page 168: Discrete Book

164 FUNDAMENTALS OF COUNTING AND PROBABILITY

Example 20.3Five different books are on a shelf. In how many different ways could youarrange them?

Solution.The five books can be arranged in 5 · 4 · 3 · 2 · 1 = 5! = 120 ways

Counting PermutationsWe next consider the permutations of a set of objects taken from a largerset. Suppose we have n items. How many ordered arrangements of k itemscan we form from these n items? The number of permutations is denotedby nPk. The n refers to the number of different items and the k refers to thenumber of them appearing in each arrangement. A formula for nPk is givennext.

Theorem 20.1For any non-negative integer n and 0 ≤ k ≤ n we have

nPk =n!

(n− k)!.

Proof.We can treat a permutation as a decision with k steps. The first step can bemade in n different ways, the second in n − 1 different ways, ..., the kth inn − k + 1 different ways. Thus, by the Fundamental Principle of Countingthere are n(n − 1) · · · (n − k + 1) k−permutations of n objects. That is,

nPk = n(n− 1) · · · (n− k + 1) = n(n−1)···(n−k+1)(n−k)!(n−k)! = n!

(n−k)!

Example 20.4How many license plates are there that start with three letters followed by 4digits (no repetitions)?

Solution.The decision consists of two steps. The first is to select the letters and thiscan be done in 26P3 ways. The second step is to select the digits and thiscan be done in 10P4 ways. Thus, by the Fundamental Principle of Countingthere are 26P3 ·10 P4 = 78, 624, 000 license plates

Example 20.5How many five-digit zip codes can be made where all digits are different?The possible digits are the numbers 0 through 9.

Page 169: Discrete Book

20 PERMUTATIONS 165

Solution.The answer is 10P5 = 10!

(10−5)! = 30, 240 zip codes

Page 170: Discrete Book

166 FUNDAMENTALS OF COUNTING AND PROBABILITY

Review Problems

Problem 20.1Find m and n so that mPn = 9!

6!

Problem 20.2How many four-letter code words can be formed using a standard 26-letteralphabet(a) if repetition is allowed?(b) if repetition is not allowed?

Problem 20.3Certain automobile license plates consist of a sequence of three letters fol-lowed by three digits.(a) If letters can not be repeated but digits can, how many possible licenseplates are there?(b) If no letters and no digits are repeated, how many license plates arepossible?

Problem 20.4A permutation lock has 40 numbers on it.(a) How many different three-number permutation lock can be made if thenumbers can be repeated?(b) How many different permutation locks are there if the three numbers aredifferent?

Problem 20.5(a) 12 cabinet officials are to be seated in a row for a picture. How manydifferent seating arrangements are there?(b) Seven of the cabinet members are women and 5 are men. In how manydifferent ways can the 7 women be seated together on the left, and then the5 men together on the right?

Problem 20.6Using the digits 1, 3, 5, 7, and 9, with no repetitions of the digits, how many(a) one-digit numbers can be made?(b) two-digit numbers can be made?(c) three-digit numbers can be made?(d) four-digit numbers can be made?

Page 171: Discrete Book

20 PERMUTATIONS 167

Problem 20.7There are five members of the Math Club. In how many ways can thepositions of a president, a secretary, and a treasurer, be chosen?

Problem 20.8Find the number of ways of choosing three initials from the alphabet if noneof the letters can be repeated. Name initials such as MBF and BMF areconsidered different.

Page 172: Discrete Book

168 FUNDAMENTALS OF COUNTING AND PROBABILITY

21 Combinations

In a permutation the order of the set of objects or people is taken into ac-count. However, there are many problems in which we want to know thenumber of ways in which k objects can be selected from n distinct objects inarbitrary order. For example, when selecting a two-person committee from aclub of 10 members the order in the committee is irrelevant. That is choosingMr. A and Ms. B in a committee is the same as choosing Ms. B and Mr. A.A combination is defined as a possible selection of a certain number of objectstaken from a group without regard to order. More precisely, the number ofk−element subsets of an n−element set is called the number of combina-tions of n objects taken k at a time. It is denoted by nCk and is read“n choose k”. The formula for nCk is given next.

Theorem 21.1If nCk denotes the number of ways in which k objects can be selected froma set of n distinct objects then

nCk =nPk

k!=

n!

k!(n− k)!.

Proof.Since the number of groups of k elements out of n elements is nCk and eachgroup can be arranged in k! ways, we have nPk = k!nCk. It follows that

nCk =nPk

k!=

n!

k!(n− k)!

An alternative notation for nCk is

(nk

). We define nCk = 0 if k < 0 or

k > n.

Example 21.1A jury consisting of 2 women and 3 men is to be selected from a group of 5women and 7 men. In how many different ways can this be done? Supposethat either Steve or Harry must be selected but not both, then in how manyways this jury can be formed?

Solution.There are 5C2 ·7 C3 = 350 possible jury combinations consisting of 2 women

Page 173: Discrete Book

21 COMBINATIONS 169

and 3 men. Now, if we suppose that Steve and Harry can not serve togetherthen the number of jury groups that do not include the two men at the sametime is 5C25C22C1 = 200

The next theorem discusses some of the properties of combinations.

Theorem 21.2Suppose that n and k are whole numbers with 0 ≤ k ≤ n. Then(a) nC0 =n Cn = 1 and nC1 =n Cn−1 = n.(b) Symmetry property: nCk =n Cn−k.(c) Pascal’s identity: n+1Ck =n Ck−1 +n Ck.

Proof.(a) From the formula of nCk we have nC0 = n!

0!(n−0)! = 1 and nCn = n!n!(n−n)! =

1. Similarly, nC1 = n!1!(n−1)! = n and nCn−1 = n!

(n−1)! = n.

(b) Indeed, we have nCn−k = n!(n−k)!(n−n+k)!

= n!k!(n−k)! =n Ck.

(c) We have

nCk−1 +n Ck =n!

(k − 1)!(n− k + 1)!+

n!

k!(n− k)!

=n!k

k!(n− k + 1)!+n!(n− k + 1)

k!(n− k + 1)!

=n!

k!(n− k + 1)!(k + n− k + 1)

=(n+ 1)!

k!(n+ 1− k)!=n+1 Ck

Example 21.2The Russellville School District has six members. In how many ways(a) can all six members line up for a picture?(b) can they choose a president and a secretary?(c) can they choose three members to attend a state conference with noregard to order?

Solution.(a) 6P6 = 6! = 720 different ways(b) 6P2 = 30 ways(c) 6C3 = 20 different ways

Page 174: Discrete Book

170 FUNDAMENTALS OF COUNTING AND PROBABILITY

Pascal’s identity allows one to construct the so-called Pascal’s triangle (forn = 10) as shown in Figure 16.1.

Figure 16.1

As an application of combination we have the following theorem which pro-vides an expansion of (x+ y)n, where n is a non-negative integer.

Theorem 21.3 (Binomial Theorem)Let x and y be variables, and let n be a non-negative integer. Then

(x+ y)n =n∑

k=0

nCkxn−kyk

where nCk will be called the binomial coefficient.

Proof.The proof is by induction on n.

Basis of induction: For n = 0 we have

(x+ y)0 =0∑

k=0

0Ckx0−kyk = 1.

Induction hypothesis: Suppose that the theorem is true up to n. That is,

(x+ y)n =n∑

k=0

nCkxn−kyk

Page 175: Discrete Book

21 COMBINATIONS 171

Induction step: Let us show that it is still true for n+ 1. That is

(x+ y)n+1 =n+1∑k=0

n+1Ckxn−k+1yk.

Indeed, we have

(x+ y)n+1 =(x+ y)(x+ y)n = x(x+ y)n + y(x+ y)n

=xn∑

k=0

nCkxn−kyk + y

n∑k=0

nCkxn−kyk

=n∑

k=0

nCkxn−k+1yk +

n∑k=0

nCkxn−kyk+1

=[nC0xn+1 + nC1x

ny + nC2xn−1y2 + · · ·+ nCnxy

n]

+[nC0xny + nC1x

n−1y2 + · · ·+ nCn−1xyn + nCny

n+1]

=n+1C0xn+1 + [nC1 + nC0]x

ny + · · ·+[nCn + nCn−1]xy

n + n+1Cn+1yn+1

=n+1C0xn+1 + n+1C1x

ny + n+1C2xn−1y2 + · · ·

+n+1Cnxyn + n+1Cn+1y

n+1

=n+1∑k=0

n+1Ckxn−k+1yk.

Note that the coefficients in the expansion of (x + y)n are the entries of the(n+ 1)st row of Pascal’s triangle.

Example 21.3Expand (x+ y)6 using the Binomial Theorem.

Solution.By the Binomial Theorem and Pascal’s triangle we have

(x+ y)6 = x6 + 6x5y + 15x4y2 + 20x3y3 + 15x2y4 + 6xy5 + y6

Example 21.4How many subsets are there of a set with n elements?

Page 176: Discrete Book

172 FUNDAMENTALS OF COUNTING AND PROBABILITY

Solution.Since there are nCk subsets of k elements with 0 ≤ k ≤ n, the total numberof subsets of a set of n elements is

n∑k=0

nCk = (1 + 1)n = 2n

Page 177: Discrete Book

21 COMBINATIONS 173

Review Problems

Problem 21.1Find m and n so that mCn = 13

Problem 21.2A club with 42 members has to select three representatives for a regionalmeeting. How many possible choices are there?

Problem 21.3In a UN ceremony, 25 diplomats were introduced to each other. Supposethat the diplomats shook hands with each other exactly once. How manyhandshakes took place?

Problem 21.4There are five members of the math club. In how many ways can the two-person Social Committee be chosen?

Problem 21.5A medical research group plans to select 2 volunteers out of 8 for a drugexperiment. In how many ways can they choose the 2 volunteers?

Problem 21.6A consumer group has 30 members. In how many ways can the group choose3 members to attend a national meeting?

Problem 21.7Which is usually greater the number of combinations of a set of objects orthe number of permutations?

Problem 21.8Determine whether each problem requires a combination or a permutation:(a) There are 10 toppings available for your ice cream and you are allowed tochoose only three. How many possible 3-topping combinations can yo have?(b) Fifteen students participated in a spelling bee competition. The firstplace winner will receive $1,000, the second place $500, and the third place$250. In how many ways can the 3 winners be drawn?

Problem 21.9Use the binomial theorem and Pascal’s triangle to find the expansion of(a+ b)7.

Problem 21.10Find the 5th term in the expansion of (2a− 3b)7.

Page 178: Discrete Book

174 FUNDAMENTALS OF COUNTING AND PROBABILITY

22 Basic Probability Terms and Rules

Probability theory is one of the serious branches of mathematics with ap-plications to many sciences, namely the theory of statistics. This sectionintroduces the most basic ideas of probability.An experiment is any operation whose outcomes cannot be predicted withcertainty. The sample space S of an experiment is the set of all possibleoutcomes for the experiment. For example, if you roll a die one time thenthe experiment is the roll of the die. A sample space for this experiment isS = {1, 2, 3, 4, 5, 6} where each digit represents a face of the die.An event is any subset of a sample space. Thus, if S is the sample spacethen the collection of all possible events is the power set P(S).The Probability of an event E is the measure of occurrence of E. It is anumber between 0 and 1. If the event is impossible to occur then its proba-bility is 0. If the occurrence is certain then the probability is 1. The closerto 1 the probability is, the more likely the event is. The probability of oc-currence of an event E (called its success) will be denoted by P (E). Thus,0 ≤ P (E) ≤ 1. If an event has no outcomes, that is as a subset of S if E = ∅then P (∅) = 0. On the other hand, if E = S then P (S) = 1.

Example 22.1Which of the following numbers cannot be the probability of some event? (a)0.71 (b)−0.5 (c) 150% (d) 4

3.

Solution.(a) Yes. (b) No. Since the number is negative. (c) No since the number isgreater than 1. (d) No

Various probability concepts exist nowadays. The classical probabilityconcept applies only when all possible outcomes are equally likely, in whichcase we use the formula

P (E) =number of outcomes favorable to event

total number of outcomes=|E||S|

,

where |E| is the number of elements in E.

Example 22.2What is the probability of drawing an ace from a well-shuffled deck of 52playing cards?

Page 179: Discrete Book

22 BASIC PROBABILITY TERMS AND RULES 175

Solution.P (Ace) = 4

52= 1

13

Example 22.3What is the probability of rolling a 3 or a 4 with a fair die?

Solution.P (3 or 4) = 2

6= 1

3

A major shortcoming of the classical probability concept is its limited appli-cability, for there are many situations in which the various outcomes cannotall be regarded as equally likely. This would be the case, for instance, whenwe wonder whether a person will get a raise or when we want to predict theoutcome of an election. A widely used probability concept is the estimatedprobability which uses the relative frequency of an event and is given bythe formula:

P (E) = Relative frequency =f

n,

where f is the frequency of the event and n is the size of the sample space.

Example 22.4Records show (over a period of time) that 468 of 600 jets from Dallas toPhoenix arrived on time. Estimate the probability that any one jet fromDallas to Phoenix will arrive on time.

Solution.P (E) = f

n= 468

600= 39

50

We define the probability of nonoccurrence of an event E (called its fail-ure) by the formula

P (Ec) = 1− P (E).

Note thatP (E) + P (Ec) = 1.

Example 22.5The probability that a college student without a flu shot will get the flu is0.45. What is the probability that a college student without the flu shot willnot get the flu?

Page 180: Discrete Book

176 FUNDAMENTALS OF COUNTING AND PROBABILITY

Solution.The probability is 1− 0.45 = .55

Next, we discuss some of the rules of probability. The union of two eventsA and B is the event A ∪ B whose outcomes are either in A or in B. Theintersection of two events A and B is the event A ∩ B whose outcomesare outcomes of both events A and B. Two events A and B are said to bemutually exclusive if they have no outcomes in common. In this caseA ∩B = ∅.

Example 22.6If A and B are mutually exclusive then what is P (A ∩B)?

Solution.P (∅) = 0

Theorem 22.1For any events A and B the probability of A ∪ B is given by the additionrule

P (A ∪B) = P (A) + P (B)− P (A ∩B).

If A and B are mutually exclusive then by Example 32.6 the above formulareduces to

P (A ∪B) = P (A) + P (B).

Proof.By the Inclusion-Exclusion Principle we have

|A ∪B| = |A|+ |B| − |A ∩B|.

Thus,

P (A ∪B) =|A ∪B||S|

=|A||S|

+|B||S|− |A ∩B|

|S|=P (A) + P (B)− P (A ∩B)

Example 22.7For any event E of a sample space S show that P (E) =

∑x∈E P (x).

Page 181: Discrete Book

22 BASIC PROBABILITY TERMS AND RULES 177

Solution.This follows from the previous theorem

Example 22.8M&M plain candies come in a variety of colors. According to the manufac-turer, the color distribution is:(a) Orange: 15% (b) Green: 10% (c) Red: 20% (d) Yellow: 20% (e)Brown: 30% (f) Tan: 5%.Suppose you have a large bag of plain candies and you reach in and take onecandy at random. Find

1. P(orange candy Or tan candy). Are these outcomes mutually exclusive?

2. P(not brown candy).

Solution.1. P(orange candy Or tan candy) = .15 + .05 = .2 = 20%. The outcomesare mutually exclusive.2. P(not brown candy) = 1− .3 = .7 = 70%

Example 22.9If A is the event “drawing an ace” from a deck of cards and B is the event“drawing a spade”. Are A and B mutually exclusive? Find P (A ∪B).

Solution.The events are not mutually exclusive since there is an ace that is also aspade.

P (A ∪B) = P (A) + P (B)− P (A ∩B) =4

52+

13

52− 1

52= 31 %

Now, given two events A and B belonging to the same sample space S.The conditional probability P (A|B) denotes the probability that event Awill occur given that event B has occurred. It is given by the formula

P (A|B) =P (A ∩B)

P (B).

Page 182: Discrete Book

178 FUNDAMENTALS OF COUNTING AND PROBABILITY

Example 22.10Consider the experiment of tossing two dice. What is the probability thatthe sum of two dice equals six given that the first die is a four?

Solution.The possible outcomes of our experiment are

{(4, 1), (4, 2), (4, 3), (4, 4), (4, 5), (4, 6)}.

Thus, the probability that the sum is six given that the first die is four is 16.

Assuming that the experiment consists of tossing the two dice then by lettingB be the event that the first die is 4 and A be the event that the sum of thetwo dice is 6 then

P (A|B) =P (A ∩B)

P (B)=

136636

=1

6

If P (A|B) = P (A), we say that the two events A and B are independent.That is, the occurrence of A is independent whether or not B occurs. If twoevents are not independent, we say that they are dependent.

Example 22.11Show that A and B are independent if and only if

P (A ∩B) = P (A) · P (B).

Solution.Suppose that A and B are independent. Then P (A) = P (A|B) = P (A∩B)

P (B).

That is, P (A ∩ B) = P (A) · P (B). Conversely, if P (A ∩ B) = P (A) · P (B)

then P (A|B) = P (A∩B)P (B)

= P (A)

Example 22.12You roll two fair dice: a green one and a red one.(a) Are the outcomes on the dice independent?(b) Find P(5 on green die and 3 on red die).(c) Find P(3 on green die and 5 on red die).(d) Find P((5 on green die and 3 on red die) or (3 on green die and 5 on reddie)).

Page 183: Discrete Book

22 BASIC PROBABILITY TERMS AND RULES 179

Solution.(a) Yes.(b) P(5 on green die and 3 on red die) = 1

6· 16

= 136.

(c) P(3 on green die and 5 on red die) = 136.

(d) P((5 on green die and 3 on red die) or (3 on green die and 5 on red die))= 1

36+ 1

36= 1

18

Example 22.13Show that

P (B|A) =P (B) · P (A|B)

P (A).

Solution.This follows from the fact that P (A ∩ B) = P (B ∩ A) and the formula ofP (A|B) given above

Example 22.14Prove Bayes’ Theorem

P (A|B) =P (B|A)P (A)

P (B|A)P (A) + P (B|Ac)P (Ac).

Solution.Note first that {Ac ∩B,A ∩B} form a partition of B. Thus,

P (B) = P (A ∩B) + P (Ac ∩B).

Now by the previous example we have

P (A|B) =P (A) · P (B|A)

P (B)

=P (B|A)P (A)

P (B|A)P (A) + P (B|Ac)P (Ac)

Example 22.15Consider two urns. The first contains two white and seven black balls andthe second contains five white and six black balls. We flip a fair coin andthen draw a ball from the first urn or the second urn depending on whetherthe outcome was head or tail. What is the conditional probability that theoutcome of the toss was head given that a white ball was selected?

Page 184: Discrete Book

180 FUNDAMENTALS OF COUNTING AND PROBABILITY

Solution.Let W be the event that a white ball is drawn, and let H be the event thatthe coin comes up heads. The desired probability P (H|W ) may be calculatedas follows:

P (H|W ) =P (H ∩W )

P (W )

=P (W |H)P (H)

P (W )

=P (W |H)P (H)

P (W |H)P (H) + P (W |Hc)P (Hc)

=2912

2912

+ 511

12

=22

67

It frequently occurs that in performing an experiment we are mainly inter-ested in some functions of the outcome as opposed to the outcome itself. Forexample, in tossing dice we are interested in the sum of the dice and are notreally concerned about the actual outcome. These real-valued functions de-fined on the sample space are known as random variables. If the range isa finite subset of N then the random variable is called discrete. Otherwise,the random variable is said to be continuous. Discrete random variablesare usually the result of a count whereas a continuous random variable isusually the result of a measurement.A probability distribution is a correspondence that assigns probabilitiesto the values of a random variable. The graph of a probability distributionis called a histogram.

Example 22.16Let f denote the random variable that is defined as the sum of two fair dice.Find the probability distribution of f.

Solution.P (f = 2) = P ({(1, 1)}) = 1

36,

P (f = 3) = P ({(1, 2), (2, 1)}) = 236,

P (f = 4) = P ({(1, 3), (2, 2), (3, 1)}) = 336,

P (f = 5) = P ({(1, 4), (2, 3), (3, 2), (4, 1)}) = 436,

Page 185: Discrete Book

22 BASIC PROBABILITY TERMS AND RULES 181

P (f = 6) = P ({(1, 5, (5, 1), (2, 4), (4, 2), (3, 3)}) = 536,

P (f = 7) = P ({(1, 6), (6, 1), (2, 5), (5, 2), (4, 3), (3, 4)}) = 636,

P (f = 8) = P ({(2, 6), (6, 2), (3, 5), (5, 3), (4, 4)}) = 536,

P (f = 9) = P ({(3, 6), (6, 3), (4, 5), (5, 4)}) = 436,

P (f = 10) = P ({(4, 5), (5, 4), (5, 5)}) = 336,

P (f = 11) = P ({(5, 6), (6, 5)}) = 236,

P (f = 12) = P ({(6, 6)}) = 136

Example 22.17Construct the histogram of the random variable of Example 19.19.

Solution.

For a discrete random variable f we define the expected value ( or mean)of f by the formula

E(f) =∑x∈S

f(x)P (x).

In other words, E(f) is a weighted average of the possible values that f cantake on, each value being weighted by the probability that f assumes thatvalue.

Example 22.18Find E(f) where f is the outcome when we roll a fair die.

Page 186: Discrete Book

182 FUNDAMENTALS OF COUNTING AND PROBABILITY

Solution.Since P (1) = P (2) = · · · = P (6) = 1

6we find

E(f) = 1(1

6) + 2(

1

6) + · · ·+ 6(

1

6) =

7

2

Another quantity of interest is the variance of a random variable f , de-noted by V ar(f), which is defined by

V ar(f) = E[(f − E(f))2].

In other words, the variance measures the expected square of the deviationof f from its expected value. The standard deviation of a random variablef is the quantity defined to be the square root of the variance.

Example 22.19Show that if f and g are random variables then E(f + cg) = E(f) + cE(g)where c is a constant.

Solution.Indeed,

E(f + cg) =∑x∈S

(f + cg)(x)P (x)

=∑x∈S

f(x)P (x) + c∑x∈S

g(x)P (x)

=E(f) + cE(g)

Theorem 22.2

V ar(f) = E(f 2)− (E(f))2.

Proof.Indeed, using the previous example we have

V ar(f) =E(f 2 − 2E(f)f + (E(f))2)

=E(f 2)− 2E(f)E(f) + (E(f))2

=E(f 2)− (E(f))2

Page 187: Discrete Book

22 BASIC PROBABILITY TERMS AND RULES 183

Example 22.20Calculate V ar(f) when f represents the outcome when a fair die is rolled.

Solution.First note that

E(f 2) = (f(1))2P (1) + · · ·+ (f(6))2P (6) =91

6.

By the above theorem we have

V ar(f) = E(f 2)− (E(f))2 =91

6− (

7

2)2 =

35

12

Page 188: Discrete Book

184 FUNDAMENTALS OF COUNTING AND PROBABILITY

Review Problems

Problem 22.1What is the probability of drawing a red card from a well-shuffled deck of 52playing cards?

Problem 22.2If we roll a fair die, what are the probabilities of getting(a) a 1 or a 6;(b) an even number?

Problem 22.3A department store’s records show that 782 of 920 women who entered thestore on a Saturday afternoon made at least one purchase. Estimate theprobability that a woman who enters the store on a Saturday afternoon willmake at least one purchase.

Problem 22.4Which of the following are mutually exclusive? Explain your answers.(a) A driver getting a ticket for speeding and a ticket for going through a redlight.(b) Being foreign-born and being President of the United States.

Problem 22.5If A and B are the events that a consumer testing service will rate a givenstereo system very good or good, P (A) = 0.22, P (B) = 0.35. Find(a) P (Ac);(b) P (A ∪B);(c) P (A ∩B).

Problem 22.6If the probabilities are 0.20, 0.15, and 0.03 that a student will get a failinggrade in Statistics, in English, or in both, what is the probability that thestudent will get a failing grade in at least one of these subjects?

Problem 22.7If the probability that a research project will be well planned is 0.60 and theprobability that it will be well planned and well executed is 0.54, what is theprobability that a well planned research project will be well executed?

Page 189: Discrete Book

22 BASIC PROBABILITY TERMS AND RULES 185

Problem 22.8Given three events A, and B such that P (A) = 0.50, P (B) = 0.30, andP (A ∩B) = 0.15. Show that the events A and B are independent.

Problem 22.9There are 16 equally likely outcomes by flipping four coins. Let f repre-sent the number of heads. Find the probability distribution and graph thecorresponding histogram.

Page 190: Discrete Book

186 FUNDAMENTALS OF COUNTING AND PROBABILITY

23 The Pigeonhole Principle

The Pigeonhole principle asserts that if n pigeons fly into k holes withn > k then some of the pigeonholes contain at least two pigeons. Thereason this statement is true can be seen by arguing by contradiction. If theconclusion is false, each pigeonhole contains at most one pigeon and, in thiscase, we can account for at most k pigeons. Since there are more pigeonsthan holes, we have a contradiction.In problem solving, the “pigeons” are often numbers or objects, and the“pigeonholes” are properties that the numbers/objects might possess.

Example 23.1Ten persons have first names George, William, and Laura and last namesMoe, Carineo, and Barber. Show that at least two persons have the samefirst and last names.

Solution.The pigeons are the ten persons and a hole is an ordered pair (First Name,Last Name). Since there are at most nine holes, according to the pigeonholeprinciple there exist at least two persons with the same first and last name

A mathematical way to formulate the pigeonhole principle is given by thefollowing thoerem.

Theorem 23.1Let S be a finite set and {A1, A2, · · · , An} be a partition of S. There is an

index 1 ≤ i ≤ n such that |Ai| ≥ |S|n.

Proof.The proof is by contradiction. Suppose that |Ai| < |S|

nfor all 1 ≤ i ≤ n.

Since {A1, A2, · · · , An} is a partition of S we have

|S| = |A1|+ |A2|+ · · ·+ |An|

<|S|n

+|S|n

+ · · ·+ |S|n

= |S|

and this is a contradiction

Page 191: Discrete Book

23 THE PIGEONHOLE PRINCIPLE 187

Example 23.2Let S and T be two finite sets such that |S| > k|T | where k is a positiveinteger. Show that for any function f : S → T there is a t ∈ T such that theset At = {s ∈ S : f(s) = t} has more than k elements.

Solution.For each t ∈ T the set At is a subset of S. Moreover, if t1 and t2 are twodifferent elements of T and s ∈ At1 ∩ At2 then f(s) = t1 and f(s) = t2 andthis contradicts the definition of a function. Hence, At1 ∩ At2 = ∅. Finally,if s ∈ S then f(s) = t ∈ T so that s ∈ At. Hence, S = ∪t∈TAt. It followsthat {At}t∈T is a partition of S. By Theorem 23.1, there is a t ∈ T such

that|At| ≥ |S|n, where n ≤ |T |. Since |S| > k|T |, |S| > kn. Thus, |At| > k so

that At has at least k elements

As a consequence of the above example, we have

Example 23.3If S and T are finite sets such that |S| > |T | then any function f : S → T isnot one-to-one.

Solution.Let k = 1 in the previous problem. Then there is a set {s ∈ S : f(s) = t}with more than one element. Say, s1, s2 are such that f(s1) = f(s2) = t withs1 6= s2. But this says that f is not one-to-one

Page 192: Discrete Book

188 FUNDAMENTALS OF COUNTING AND PROBABILITY

Review Problems

Problem 23.1A family has 10 children. Show that at least two children were born on thesame day of the week.

Problem 23.2Show that if 11 people take an elevator in a 10-story building then at leasttwo people exist the elevator on the same floor.

Problem 23.3A class of 11 students wrote a short essay. George Perry made 9 errors, eachof the other students made less than that number. Prove that at least twostudents made equal number of errors.

Problem 23.4Let A = {1, 2, 3, 4, 5, 6, 7, 8}. Prove that if five integers are selected from A,then at least one pair of integers have a sum of 9.

Problem 23.5Prove that, given any 12 natural numbers, we can chose two of them andsuch that their difference is divisible by 11.

Problem 23.6In a group of 1500, use Example 23.2 to find the least number of people whoshare the same birthday.

Problem 23.7Suppose that every student in this class flipped a coin four times. Show tahtat least two students would have the exact same sequence of heads and tails.

Page 193: Discrete Book

Basics of Graph Theory

In this chapter we present the basic concepts related to graphs and trees suchas the degree of a vertex, connectedness, Euler and Hamiltonian circuits,isomorphisms of graphs, rooted and spanning trees.

189

Page 194: Discrete Book

190 BASICS OF GRAPH THEORY

24 Graphs and the Degree of a Vertex

An undirected graph G consists of a set VG of vertices and a set EG ofedges such that each edge e ∈ EG is associated with an unordered pair ofvertices, called its endpoints.A directed graph or digraph G consists of a set VG of vertices and a setEG of edges such that each edge e ∈ EG is associated with an ordered pairof vertices.We denote a graph by G = (VG, EG).

Example 24.1Find the vertices and edges of the directed graph shown in Figure 24.1.

Figure 24.1

Solution.The vertices are

VG = {0, 1, 2, 3, 4, 5, 6}

and the edges are

EG = {(0, 2), (0, 4), (0, 5), (1, 0), (2, 1), (2, 5), (3, 1), (3, 6), (4, 0), (4, 5), (6, 3), (6, 5)}

Two vertices are said to be adjacent if there is an edge connecting the twovertices. Two edges associated to the same vertices are called parallel. An

Page 195: Discrete Book

24 GRAPHS AND THE DEGREE OF A VERTEX 191

edge incident to a single vertex is called a loop. A vertex that is not incidenton any edge is called an isolated vertex. A graph with neither loops norparallel edges is called a simple graph.

Example 24.2Consider the following graph G = (VG, EG).

Figure 24.2

(a) Find EG and VG.(b) List the isolated vertices.(c) List the loops.(d) List the parallel edges.(e) List the vertices adjacent to {3}..(f) Find all edges incident on {8}.

Solution.(a) We have

VG = {1, 2, 3, 4, 5, 6, 7, 8}

andEG = {e1, e2, · · · , e14}.

(b) There is only one isolated vertex, {4}.(c) There are two loops {3, 7}.(d) {e2, e3, e11, e12, e13}.(e) {2, 3, 5, 8}.(f) {e2, e3, e4, e7, e9, e11, e12, e13}

Page 196: Discrete Book

192 BASICS OF GRAPH THEORY

Example 24.3Which one of the following graphs is simple?

Figure 24.3

Solution.(i) G is not simple since it has a loop and parallel edges.(ii) and (iii) are simple graphs

A complete graph on n vertices, denoted by Kn, is the simple graph thatcontains exactly one edge between each pair of distinct vertices.

Example 24.4Draw K2, K3, and K4.

Solution.K2, K4 and K5 are shown in Figure 24.4

Figure 24.4

Page 197: Discrete Book

24 GRAPHS AND THE DEGREE OF A VERTEX 193

A graph in which the vertices can be partitioned into two disjoint sets V1and V2 with every edge incident on one vertex in V1 and one vertex of V2 iscalled bipartite graph.

Example 24.5(a) Show that the graph G is bipartite.

Figure 24.5

(b) Show that K3 is not bipartite.

Solution.(a) Clear from the definition and the graph.(b) Any two sets of vertices of K3 will have one set with at least two vertices.Thus, according to the definition of bipartite graph, K3 is not bipartite

A complete bipartite graph Km,n, is the graph that has its vertex setpartitioned into two disjoint subsets of m and n vertices, respectively. More-over, there is an edge between two vertices if and only if one vertex is in thefirst set and the other vertex is in the second set.

Example 24.6Draw K2,3 and K3,3.

Solution.K2,3 and K3,3 are given in Figure 24.6

Page 198: Discrete Book

194 BASICS OF GRAPH THEORY

Figure 24.6

The degree of a vertex v in an undirected graph, in symbol deg(v), is thenumber of edges incident on it. By definition, a loop at a vertex contributestwice to the degree of that vertex. The total degree of G is the sum of thedegrees of all the vertices of G.

Example 24.7What are the degrees of the vertices in Figure 24.7.

Figure 24.7

Solution.deg(B) = 0, deg(A) = 5, deg(T ) = 3, deg(M) = 3, and deg(H) = 5

Theorem 24.1For any graph G = (VG, EG) we have

2|EG| =∑

v∈V (G)

deg(v).

Page 199: Discrete Book

24 GRAPHS AND THE DEGREE OF A VERTEX 195

Proof.Suppose that VG = {v1, v2, · · · , vn} and |EG| = m. Let e ∈ EG. If e is a loopthen it contributes 2 to the total degree of G. If e is not a loop then let vi andvj denote the endpoints of e. Then e contributes 1 to deg(vi) and contributes1 to the deg(vj). Therefore, e contributes 2 to the total degree of G. Since ewas chosen arbitrarily, this shows that each edge of G contributes 2 to thetotal degree of G. Thus,

2|EG| =∑

v∈V (G)

deg(v)

The following is easily deduced from the previous theorem.

Theorem 24.2In any graph there are an even number of vertices of odd degree.

Proof.Let G = (VG, EG) be a graph. By the previous theorem, the sum of all thedegrees of the vertices is T = 2|EG|, an even number. Let E be the sum ofthe numbers deg(v), each which is even and O the sum of numbers deg(v)each which is odd. Then T = E +O. That is, O = T −E. Since both T andE are even, O is also even. This implies that there must be an even numberof the odd degrees. Hence, there must be an even number of vertices withodd degree.

Example 24.8Find a formula for the number of edges in Kn.

Solution.Since G is complete, each vertex is adjacent to the remaining vertices. Thus,the degree of each of the n vertices is n−1, and we have the sum of the degreesof all of the vertices being n(n− 1). By Theorem 24.1, n(n− 1) = 2|EG|

Page 200: Discrete Book

196 BASICS OF GRAPH THEORY

Review Problems

Problem 24.1Find the vertices and edges of the following directed graph.

Problem 24.2Consider the following graph G

(a) Find EG and VG.(b) List the isolated vertices.(c) List the loops.(d) List the parallel edges.(e) List the vertices adjacent to v3..(f) Find all edges incident on v4.

Problem 24.3Which one of the following graphs is simple?

Page 201: Discrete Book

24 GRAPHS AND THE DEGREE OF A VERTEX 197

Problem 24.4Draw K5.

Problem 24.5Which of the following graph is bipartite?

Problem 24.6Draw the complete bipartite graph K2,4.

Problem 24.7What are the degrees of the vertices in the following graph

Page 202: Discrete Book

198 BASICS OF GRAPH THEORY

Problem 24.8The union of two graphs G1 = (V1, E1) and G2 = (V2, E2) is the graphG1 ∪G2 = (V1 ∪ V2, E1 ∪E2). The intersection of two graphs G1 = (V1, E1)and G2 = (V2, E2) is the graph G1 ∩G2 = (V1 ∩ V2, E1 ∩ E2).Find the union and the intersection of the graphs

Problem 24.9Graphs can be represented using matrices. The adjacency matrix of a graphG with n vertices is an n×n matrix AG such that each entry aij is the numberof edges connecting vi and vj. Thus, aij = 0 if there is no edge from vi to vj.(a) Draw a graph with the adjacency matrix

0 1 1 01 0 0 11 0 0 10 1 1 0

Page 203: Discrete Book

24 GRAPHS AND THE DEGREE OF A VERTEX 199

(b) Use an adjacency matrix to represent the graph

Problem 24.10A graph H = (VH , EH) is a subgraph of G = (VG, EG) if and only if VH ⊆ VGand EH ⊆ EG.Find all nonempty subgraphs of the graph

Problem 24.11When (u, v) is an edge in a directed graph G then u is called the initialvertex and v is called the terminal vertex. In a directed graph, the in-degree of a vertex v, denoted by deg−(v), is the number of edges with v astheir terminal vertex. Similarly, the out-degree of v, denoted by deg+(v),is the number of edges with v as an initial vertex. Note that deg(v) =

Page 204: Discrete Book

200 BASICS OF GRAPH THEORY

deg+(v) + deg−(v).Find the in-degree and out-degree of each of the vertices in the graph G withdirected edges.

Problem 24.12Show that for a digraph G = (VG, EG) we have

|EG| =∑

v∈V (G)

deg−(v) =∑

v∈V (G)

deg+(v).

Problem 24.13Another useful matrix representation of a graph is known as the incidencematrix. It is constructed as follows. We label the rows with the verticesand the columns with the edges. The entry for row v and column e is 1 if eis incident on v and 0 otherwise. If e is a loop at v we assign the value 2. Itis easy to see that the sum of entries of each column is 2 and that the sumof entries of a row gives the degree of the vertex corresponding to that row.Find the incidence matrix corresponding to the graph

Page 205: Discrete Book

24 GRAPHS AND THE DEGREE OF A VERTEX 201

Problem 24.14If each vertex of an undirected graph has degree k then the graph is called aregular graph of degree k.How many edges are there in a graph with 10 vertices each of degree 6?

Page 206: Discrete Book

202 BASICS OF GRAPH THEORY

25 Paths and Circuits

In an undirected graph G, a sequence of n non-repeated edges connectingtwo vertices is called a path of length n. A circuit, a cycle, or a closedpath is a path which the first and last vertices are the same. A path orcircuit is simple if no vertex is repeated. A graph that does not contain anycircuit is called acyclic.

Example 25.1In the graph below, determine whether the following sequences are paths,simple paths, circuits, or simple circuits.

Figure 25.1

(a) v0e1v1e10v5e9v2e2v1.(b) v3e5v4e8v5e10v1e3v2.(c) v1e2v2e3v1.(d) v5e9v2e4v3e5v4e6v4e8v5.

Solution.(a) a path (no repeated edge), not a simple path (repeated vertex v1), not acircuit(b) a simple path(c) a simple circuit(d) a circuit, not a simple circuit (vertex v4 is repeated)

Example 25.2Give an example of an acyclic graph with three vertices.

Page 207: Discrete Book

25 PATHS AND CIRCUITS 203

Solution.One such an example is shown in Figure 25.2

Figure 25.2

An undirected graph is called connected if there is a path between everypair of distinct vertices of the graph. A graph that is not connected is saidto be disconnected.

Example 25.3Determine which graph is connected and which one is disconnected.

Figure 25.3

Solution.(I) is connected whereas (II) and (III) are disconnected

A path that contains all edges of a graph G is called an Euler path. Ifthis path is also a circuit, it is called an Euler circuit.. Note that an Eulerpath starts and ends at different vertices whereas an Euler circuit starts andends at the same vertex.

Page 208: Discrete Book

204 BASICS OF GRAPH THEORY

Example 25.4Consider the grap in Figure 25.4.

Figure 25.4

(a) Is the circuit ABCDEFA an Euler circuit? Explain.(b) Is the circuit ABCDEFBDFA an Euler circuit? Explain.

Solution.(a) The circuit ABCDEFA is not an Euler circuit since it does not uses theedges BD,DF, FB.(b) The circuit ABCDEFBDFA is an Euler circuit since it uses all theedges of the graph

How can we tell if a graph has an Euler circuit or an Euler path? Thefollowing theorem whose proof is omitted provides criteria for the existenceof either Euler path or Euler circuit.

Theorem 25.1 (Euler’s Theorem)Let G be a connected graph.(a) If some vertex has odd degree, then G has no Euler circuit.(b) If every vertex has even degree, then G has an Euler circuit.(c) If there are exactly two vertices of odd degree, then G has an Euler paththat starts at one of these vertices and ends at the other.(d) If there are more than two vertices of odd degree, then G has no Eulerpath.

Example 25.5Show that the following graph has no Euler circuit.

Page 209: Discrete Book

25 PATHS AND CIRCUITS 205

Figure 25.5

Solution.Vertices v1 and v3 both have degree 3, which is odd. Hence, by Theorem25.1, this graph does not have an Euler circuit

Example 25.6Show that the following graph has an Euler path.

Figure 25.6

Solution.We have deg(A) = deg(B) = 3 and deg(C) = deg(D) = deg(E) = 4. Hence,by Theorem 25.1, the graph has an Euler path

A path is called a Hamiltonian path if it visits every vertex of the graphexactly once. A circuit that visits every vertex exactly once except for thelast vertex which duplicates the first one is called a Hamiltonian circuit.

Page 210: Discrete Book

206 BASICS OF GRAPH THEORY

Example 25.7Find a Hamiltonian circuit in the graph

Solution.vwxyzv

Example 25.8Show that the following graph has a Hamiltonian path but no Hamiltoniancircuit.

Solution.vwxyz is a Hamiltonian path. There is no Hamiltonian circuit since no cyclegoes through v

Page 211: Discrete Book

25 PATHS AND CIRCUITS 207

Review Problems

Problem 25.1Consider the graph shown below.

(a) Give an example of a path of length 4 connecting the vertices A andC.(b) Give an example of a simple path of length 4 connecting the vertices Aand B.(c) Give an example of a simple circuit of length 5 starting and ending at A.

Problem 25.2Give an example of an acyclic graph with four vertices.

Problem 25.3Determine which graph is connected and which one is disconnected.

Page 212: Discrete Book

208 BASICS OF GRAPH THEORY

Problem 25.4Show that the following graph has an Euler circuit.

Problem 25.5Show that the following graph has no Hamiltonian path.

Problem 25.6Which of the graphs shown below are connected?

Problem 25.7If a graph is disconnected then the various connected pieces of the grapgare called the connected components. Find the number of connected

Page 213: Discrete Book

25 PATHS AND CIRCUITS 209

components of each of the given graphs.

Problem 25.8Find the connected components of the graph shown below.

Problem 25.9Show that the graphs given below do not have an Euler circuit.

Problem 25.10Show that the graphs given below do not have an Euler path.

Page 214: Discrete Book

210 BASICS OF GRAPH THEORY

Problem 25.11Show that the graph given below has an Euler path.

Problem 25.12Does the graph shown below have a Hamiltonian circuit?

Problem 25.13Does the graph shown below have a Hamiltonian circuit?

Page 215: Discrete Book

25 PATHS AND CIRCUITS 211

Problem 25.14Does the graph shown below have a Hamiltonian circuit?

Page 216: Discrete Book

212 BASICS OF GRAPH THEORY

26 Trees

An undirected graph is called a tree if each pair of distinct vertices hasexactly one path between them. Thus, a tree has no parallel edges or loops.Trees are examples of connected acyclic graphs.

Example 26.1Which of the following graphs are trees?

Figure 26.1

Solution.(a) and (c) satisfy the definition of a tree whereas (b) does not

A vertex of degree 1 in a tree is called a leaf. A vertex of degree 2 ormore in a tree is called a branch.Next, we want to show that the number of edges in a tree is one fewer thanthe number of vertices. In order to prove this result, we establish first thefollowing lemma.

Lemma 26.1Let T be a graph with more than one vertex. If T is a tree then one vertexmust be of degree 1.

Proof.We use contrapositive to prove the lemma. Suppose that graph T has novertex of degree 1. Starting at any vertex v, follow a sequence of distinctedges until a vertex repeats; this is possible because the degree of every ver-tex is at least two, so upon arriving at a vertex for the first time it is alwayspossible to leave the vertex on another edge. When a vertex repeats for thefirst time, we have discovered a cycle. Hence, T is not a tree

Page 217: Discrete Book

26 TREES 213

The following result shows that trees have one fewer edge than they havevertices. Thus, it can be used as a criterion for showing that a graph is nota tree.

Theorem 26.1A tree with n vertices has exactly n− 1 edges. That is, if G = (VG, EG) is atree then |EG| = |VG| − 1.

Proof.The proof is by induction on n ≥ 1. Let P (n) be the property: Any tree withn vertices has n− 1 edges.Basis of induction: P (1) is valid since a tree with one vertex has zero edges.Induction hypothesis: Suppose that P (n) holds up to n ≥ 1.Induction Step: We must show that any tree with n+ 1 vertices has n edges.Indeed, let T be any tree with n + 1 vertices. Since n + 1 ≥ 2, by theprevious lemma, T has a vertex v of degree 1. Let T0 be the graph obtainedby removing v and the edge attached to v. Then T0 is a tree with n vertices.By the induction hypothesis, T0 has n− 1 edges and so T has n edges

Example 26.2Which of the following graphs are trees?

Figure 26.2

Solution.The first graph satisfies the definition of a tree. The second and third graphsdo not satisfy the conclusion of Theorem 26.1 and therefore they are nottrees

The following is the converse to Theorem 26.1.

Page 218: Discrete Book

214 BASICS OF GRAPH THEORY

Theorem 26.2Any connected graph with n vertices and n− 1 edges is a tree.

Proof.We prove the result by contradiction. Let G be a connected graph withn vertices and n − 1 edges. Suppose that G is not a tree. Then G has acircuit or a cycle. Let G1 be the connected graph obtained by removingan edge of the cycle. We continue this process until we reach a connectedgraph Gk with no cycles, where k is the number of edges removed. Thus, Gk

is a tree with n vertices and n−1−k edges. This contradicts Theorem 26.1

Rooted TreesA rooted tree is a tree in which a particular vertex is designated as theroot. The level of a vertex v is the length of the simple path from theroot to v. The height of a rooted tree is the largest level number that occurs.

Example 26.3Find the level of each vertex and the height of the following rooted tree.

Figure 26.3

Page 219: Discrete Book

26 TREES 215

Solution.v1 is the root of the given tree.

vertex levelv2 1v3 1v4 2v5 2v6 2v7 2

The height of the tree is 2

Let T be a rooted tree with root v0. Suppose (v0, v1, · · · , vn) is a simplepath in T and x, y, z are three vertices of a tree. Then(a) vn−1 is the parent of vn.(b) v0, v1, · · · , vn−1 are the ancestors of vn.(c) vn is the child of vn−1.(d) If x is an ancestor of y then y is a descendant of x.(e) If x and y are children of z then x and y are siblings.(f) If x has no children, then x is a leaf.(g) The subtree of T rooted at x is the graph with vertex set V and edgeset E, where V is x together with the descendants of x and

E = {e|e is an edge on a simple path from x to some vertex in V }.

Example 26.4Consider the rooted tree of Figure 26.4.(a) Find the parent of v6.(b) Find the ancestors of v13.(c) Find the children of v3.(d) Find the descendants of v11.(e) Find an example of a siblings.(f) Find the leaves.(g) Construct the subtree rooted at v7.

Page 220: Discrete Book

216 BASICS OF GRAPH THEORY

Figure 26.4

Solution.(a) v2.(b) v1, v3, v7.(c) v7, v8, v9.(d) None.(e) {v2, v3, v4, v5}.(f) {v4, v5, v6, v8, v9, v10, v11, v12, v13}.(g)

Figure 26.5

Binary TreesA binary tree is a rooted tree such that each vertex has at most two children.Moreover, each child is designated as either a left child or a right child. A

Page 221: Discrete Book

26 TREES 217

full binary tree is a binary tree in which each vertex has either two childrenor zero children.

Example 26.5(a) Show that the following tree is a binary tree.

Figure 26.6

(b) Find the left child and the right child of vertex v5.(c) Construct an example of a full binary tree.

Solution.(a) Follows from the definition of a binary tree.(b) The left child is v6 and the right child is v7.(c)

Figure 26.7

Page 222: Discrete Book

218 BASICS OF GRAPH THEORY

Spanning TreeLet T be a subgraph of a graph G such that T is a tree containing all of thevertices of G. Such a tree is called a spanning tree.

Example 26.6Find a spanning tree of the following graph.

Figure 26.8

Solution.

Figure 26.9

The following theorem provides an algorithm for creating a spanning tree ofany connected graph.

Theorem 26.3(a) Every connected graph G has a spanning tree.(b) Any two spanning trees of G have the same number of edges.

Page 223: Discrete Book

26 TREES 219

Proof.(a) If G is circuit-free it is a tree and hence its own spanning tree. If G hasa circuit. Remove an edge of this circuit to get a new graph G1. If G1 has acircuit remove an edge from the circuit to obtain a new graph G2. Continueuntil we reach a circuit-free graph, Gk for some k. Gk is a spanning tree forG.(b) Any spanning tree of G has |V | − 1 edges, where |V | is the number ofvertices of G

Example 26.7Find four spanning trees of tge graph shown in Figure 26.10.

Figure 26.10

Solution.The four spanning trees are shown in Figure 26.11

Figure 26.11

Page 224: Discrete Book

220 BASICS OF GRAPH THEORY

Review Problems

Problem 26.1Which of the following four graphs is a tree?

Problem 26.2Find the number of edges in a tree with 58 vertices.

Problem 26.3Can a graph with 41 vertices and 40 edges be a tree?

Problem 26.4Find the level of each vertex and the height of the following rooted tree.

Page 225: Discrete Book

26 TREES 221

Problem 26.5Consider the rooted tree

(a) Find the parent of v6.(b) Find the ancestors of v10.(c) Find the children of v4.(d) Find the descendants of v1.(e) Find all the siblings.(f) Find the leaves.(g) Construct the subtree rooted at v1.

Problem 26.6The binary tree below gives an algorithm for choosing a restaurant. Eachinternal vertex asks a question. If we begin at the root, answer each ques-tion, and follow the appropriate edge, we will eventually arrive at a terminalvertex that chooses a restaurant. Such a tree is called a decision tree.

Page 226: Discrete Book

222 BASICS OF GRAPH THEORY

Construct a decision tree that sorts three given numbers a1, a2, a3 in as-cending order.

Problem 26.7A binary search tree is a binary tree T in which data are associated withthe vertices. The data are arranged so that, for each vertex v in T, each dataitem in the left subtree of v is less than the data item in v and each data itemin the right subtree of v is greater than the data item in v. Using numericalorder, form a binary search tree for a number in the set {1, 2, · · · , 15}.

Problem 26.8Procedures for systematically visiting every vertex of a tree are called traver-sal algorithms. In the preorder traversal, the root r is listed first andthen the subtrees T1, T2, · · · , Tn are listed, from left to right, in order of theirroots. The preorder traversal begins by visiting r. It continues by travers-ing T1 in preorder, thenT2 in preorder, and so on, until Tn is traversed inpreorder. In which order does a preorder traversal visit the vertices in thefollowing rooted tree?

Page 227: Discrete Book

26 TREES 223

Problem 26.9A forest is a simple graph with no circuits. Which of the following graphsis a forest?

Problem 26.10What do spanning trees of a connected graph have in common?

Problem 26.11Find three spanning trees of the graph below.

Page 228: Discrete Book

224 BASICS OF GRAPH THEORY

Page 229: Discrete Book

Answer Key

Section 1

1.1 (a) Proposition with truth value (T). Note that 322 = 1024 and 312 =961.(b) Not a proposition since the truth or falsity of the sentence depends onthe reference for the pronoun “she.” For some values of “she” the statementis true; for others it is false.(c) A proposition with truth value (F).(d) Not a proposition.

1.2 p∧ ∼ q.

1.3 Exclusive.

1.4

p q r ∼ p ∼ r ∼ p ∨ q q∧ ∼ r ∼ (q∧ ∼ r) p ∨ (∼ p ∨ q) sT T T F F T F T T TT T F F T T T F T FT F T F F F F T T TT F F F T F F T T TF T T T F T F T T TF T F T T T T F T FF F T T F T F T T TF F F T T T F T T T

225

Page 230: Discrete Book

226 ANSWER KEY

1.5

p t p ∨ tT T TF T T

1.6

p c p ∨ cT F TF F F

1.7

p q r r ∨ p r ∨ q ∼ r ∨ (p ∧ q) s (r ∨ p) ∧ s p ∧ qT T T T T T T T TT T F T T T T T TT F T T T F F F FT F F T F T F F FF T T T T F F F FF T F F T T F F FF F T T T F F F FF F F F F T F F F

1.8 “This computer program is error free in the first ten lines and it is beingrun with complete data”

1.9 “The dollar is not at an all-time high or the stock market is not ata record low.”

1.10 x ≤ −5 or x > 0.

1.11

p q ∼ p ∼ q ∼ p ∨ (p∧ ∼ q) p ∧ q sT T F F F T TT F F T T F TF T T F T F TF F T T T F T

Page 231: Discrete Book

227

1.12p q ∼ p ∼ q p∧ ∼ q) ∼ p ∨ q sT T F F F T FT F F T T F FF T T F F T FF F T T F T F

1.13 (a) Constructing the truth table for p⊕ p we find

p p⊕ pT FF F

Hence, p⊕ p ≡ c, where c is a contradiction.Now, (p⊕ p)⊕ p ≡ c⊕ p ≡ p.(b) (p⊕ q)⊕ r ≡ p⊕ (q ⊕ r) because of the following truth table.

p q r p⊕ q (p⊕ q)⊕ r p⊕ (q ⊕ r)T T T F T TT T F F F FT F T T F FT F F T T TF T T T F FF T F T T TF F T F F TF F F F F F

(c) (p⊕ q) ∧ r 6≡ (p ∧ r)⊕ (q ∧ r) because of the following truth table.

p q r p⊕ q (p ⊕ q) ∧ r (p ∧ r)⊕ (q ∧ r)T T T F F FT T F F F 6= TT F T T T TT F F T F FF T T T T TF T F T F FF F T T T 6= FF F F F F F

Page 232: Discrete Book

228 ANSWER KEY

1.14 (a)

p t p ∧ tT T TF T F

(b)

p c p ∧ cT F FF F F

(c)

t ∼ t cT F FT F F

c ∼ c tF T TF T T

(d)

p p ∧ pT TF F

p p ∨ pT TF F

1.15 (a) Proposition with truth value (T).(b) This is a question and not a proposition.(c) This is a command and not a proposition.(d) This is not a proposition since it is true for x = 2 but false otherwise.(e) This is a proposition with true value (F).

1.16 (a) ∼ (∼ p ∧ q) ≡∼ (∼ p)∨ ∼ q ≡ p∨ ∼ q.(b) John is at work and Peter is not at the gym.

Page 233: Discrete Book

229

1.17p q p ∧ q ∼ p (p ∧ q) ∨ (∼ p)T T T F TT F F F FF T F T TF F F T T

1.18

p q p ∨ q p ∧ q ∼ (p ∧ q) (p ∨ q)∧ ∼ (p ∧ q) p⊕ pT T T T F F FT F T F T T TF T T F T T TF F F F T F F

1.19p q p ∧ q ∼ (p ∧ q) p∨ ∼ (p ∧ q)T T T F TT F F T TF T F T TF F F T T

1.20 By De Morgan’s Law, ∼ p∧(p∧q) =∼ [p∨ ∼ (p∧q)]. From the previousproblem, p∨ ∼ (p ∧ q) is a tuatology. Hence, its negation ∼ p ∧ (p ∧ q) is acontradiction.

Page 234: Discrete Book

230 ANSWER KEY

Section 2

2.1

P Q R S1 1 1 11 1 0 11 0 1 11 0 0 10 1 1 10 1 0 00 0 1 00 0 0 0

2.2

2.3 (a) (P ∧Q)∧ ∼ R.(b)

Page 235: Discrete Book

231

2.4

P Q R S1 1 1 01 0 0 10 1 0 10 0 0 0

2.5 (a) 104310 = 100000100112 (b) 011011012 = 10910.

2.6 +10410 = 01101000 and −10410 = 10011000.

2.7

A B S Z1 1 1 11 1 0 11 0 1 01 0 0 10 1 1 10 1 0 00 0 1 00 0 0 0

2.8

A B Z =∼ (A⊕B)1 1 11 0 00 1 00 0 1

2.9 A⊕ A = 0 and A⊕ (∼ A) = 1.

2.10

Page 236: Discrete Book

232 ANSWER KEY

2.11 (A ∧B) ∨ [(C ∨D) ∧ E].

2.12 S = P ∧R.

2.13 The truth table is

A B S1 1 01 0 10 1 10 0 0

which is the same as the truth table of A⊕B.

2.14

A B Q Z1 1 1 11 0 0 00 1 0 00 0 1 1

which is the same of the truth table of ∼ (A⊕B).

Page 237: Discrete Book

233

2.15

X =(∼ A) ∧ (∼ D)

Y =D ∧BZ =(∼ D) ∧ CW =(D ∧B) ∨ [(∼ D) ∧ C]

Q =[(∼ A) ∧ (∼ D)] ∨ {(D ∧B) ∨ [(∼ D) ∧ C]}.

2.16 (a) 51310 = 10000000012 (b) 11102 = 1310.

2.17Number 4-bit 8-bit 16-bit

7 0111 00000111 0000000000000111−2 1110 11111110 1111111111111110

2.18 10111000.

2.19 01010001.

2.20 −10910.

2.21 7210.

Page 238: Discrete Book

234 ANSWER KEY

Section 3

3.1

p q r ∼ p ∼ p ∨ q ∼ p ∨ q → rT T T F T TT T F F T FT F T F F TT F F F F TF T T T T TF T F T T FF F T T T TF F F T T F

3.2

p q r p→ r q → r (p→ r)↔ (q → r)T T T T T TT T F F F TT F T T T TT F F F T FF T T T T TF T F T F FF F T T T TF F F T T T

3.3 (a) 2+2 = 4 and 2 is not a prime number. (b) 1 = 0 and√

2 is irrational.

3.4 (a) If 2 is not a prime number then 2 + 2 6= 4. (b) If√

2 is irrational then1 6= 0.

3.5 (a) If 2 is a prime number then 2+2 = 4. (b) If√

2 is rational then 1 = 0.

3.6 (a) If 2 + 2 6= 4 then 2 is not a prime number. (b) If 1 6= 0 then√2 is irrational.

Page 239: Discrete Book

235

3.7p q p→ q (p→ q)→ q p ∨ qT T T T TT F F T TF T T T TF F T F F

3.8

∼ (p↔ q) ≡ ∼ [(p→ q) ∧ (q → p)]

≡[∼ (p→ q)] ∨ [∼ (q → p)]

≡(p∧ ∼ q) ∨ (∼ p ∧ q).

3.9 Since p is false, the conditional proposition p → q is vacuously true.Since q is true and p is false, the conditional proposition q → p is false.

3.10 (a) Since p ∧ q is false, p ∧ q → r is vacuously true.(b) Since p ∨ q is true and ∼ r is false, p ∨ q →∼ r is false.(c) Since q → r is vacuously true, p ∨ (q → r) is true.

3.11

(p ∨ q)→ r ≡[∼ (p ∨ q)] ∨ r≡[(∼ p) ∧ (∼ q)] ∨ r≡[(∼ p) ∨ r] ∧ [(∼ q) ∨ r]≡(p→ r) ∧ (q → r).

3.12 Let c denote a contradiction. Using Example 1.10 and Problem 1.6, Wehave

p↔ q ≡(p→ q) ∧ (q → p)

≡(∼ p ∨ q) ∧ (∼ q ∨ p)≡[∼ p ∧ (∼ q ∨ p)] ∨ [q ∧ (∼ q ∨ p)]≡[(∼ p∧ ∼ q) ∨ (∼ p ∧ p)] ∨ [(q∧ ∼ q) ∨ (q ∧ p)]≡[(∼ p∧ ∼ q) ∨ c] ∨ [c ∨ (q ∧ p)]≡(∼ p∧ ∼ q) ∨ (p ∧ q)≡(p ∧ q) ∨ (∼ p∧ ∼ q).

Page 240: Discrete Book

236 ANSWER KEY

3.13 (a) By Example 3.6, this is the case when either both p and q are trueor both are false.(b) None.

3.14

p q p↔ q ∼ (p⊕ q)T T T TT F F FF T F FF F T T

3.15 Vacuously true since the hypothesis is false.

3.16

p q ∼ p p→ q ∼ p ∧ (p→ q)T T F T FT F F F FF T T T TF F T T T

3.17

p q p→ q q → p (p→ q) ∨ (q → q)T T T T TT F F T TF T T F TF F T T T

3.18

p q r p→ q q → r (p→ q) ∧ (q → r)T T T T T TT T F T F FT F T F T FT F F F T FF T T T T TF T F T F FF F T T T TF F F T T T

Page 241: Discrete Book

237

3.19 The hypothesis here is “Me not going to town” and the conclusion is“It is raining”. Hence, the converse proposotion is “If it is raining then Iam not going to town.” The inverse is “If I am going to town then it is notraining.” The contrapositive is “If it is not raining then I am going to town.”

3.20

p q ∼ q p∧ ∼ q (p∧ ∼ q) ∨ q p ∨ q (p∧ ∼ q) ∨ q ↔ p ∨ qT T F F T T TT F T T T T TF T F F T T TF F T F F F T

Page 242: Discrete Book

238 ANSWER KEY

Section 4

4.1 The truth or falsity of each of the propositions has no bearing on thatof the others.

4.2 The premises are “Steve is a physician” and “All doctors have goneto medical school.” The conclusion is “Steve went to medical school.”

4.3 This follows from Modus Ponens.

4.4 The truth table is as follows.

p q p→ qT T TT F FF T TF F T

Because of the third row the argument is invalid.

4.5 The truth table is as follows.

p q p→ q ∼ q ∼ pT T T F FT F F T FF T T F TF F T T T

The third row shows that the argument is invalid.

4.6 The truth table is as follows.

p q p ∨ qT T TT F TF T TF F F

The first and third rows show that the argument is valid.

Page 243: Discrete Book

239

4.7 The truth table is as follows.

p q p ∧ qT T TT F FF T FF F F

The first row shows that the argument is valid.

4.8 The truth table is as follows.

p q ∼ p ∼ q p ∨ qT T F F TT F F T TF T T F TF F T T F

The third row shows that the argument is valid.

4.9 If√

2 is rational, then√

2 = ab

for some integers a and b.

It is not true that√

2 = ab

for some integers a and b.

.̇.√

2 is irrational.

4.10 If logic is easy, then I am a monkey’s uncle.I am not a monkey’s uncle..̇. Logic is not easy.

4.11 The truth table is

p q p→ q q → p p ∧ qT T T T TT F F T TF T T F TF F T T F

Because of the last row, the given argument is invalid.

Page 244: Discrete Book

240 ANSWER KEY

4.12 The truth table is

p q r p→ q ∼ q ∨ rT T T T TT T F T FT F T F TT F F F TF T T T TF T F T FF F T T TF F F T T

Because of the first row, the given argument is valid.

4.13 Letp : Tom is on team A.q : Hua is on team B.

Then the given argument is of the form

∼ p→ q

∼ q → p

.̇. ∼ p∨ ∼ q

The truth table is

p q ∼ p→ q ∼ q → p ∼ p∨ ∼ qT T T T FT F T T TF T T T TF F F F T

Because of the first row, the given argument is invalid.

4.14p : Jules solved this problem correctly.q : Jules obtained the answer 2.

Page 245: Discrete Book

241

Then the given argument is of the form

p→ q

q

.̇. p

Its truth table is

p q p→ qT T TT F FF T TF F T

From the third row we see that the argument is invalid.

4.15 Let

p : This number is larger than 2.q : The square of this number is larger than 4.

Then the given argument is of the form

p→ q

∼ p

.̇. ∼ q

Its truth table is

p q p→ q ∼ p ∼ qT T T F FT F F F TF T T T FF F T T T

From the third row we see that the argument is invalid.

Page 246: Discrete Book

242 ANSWER KEY

4.16

(1) p→ t premise∼ t premise

.̇. ∼ p by modus tollens(2) ∼ p by (1).̇. ∼ p ∨ q by disjunctive addition

(3) ∼ p ∨ q → r premise∼ p ∨ q by (2)

.̇. r by modus ponens(4) ∼ p by (1)

r by (3).̇. ∼ p ∧ r by conjunctive addition

(5) ∼ p ∧ r →∼ s premise∼ p ∧ r by (4)

.̇. ∼ s by modus ponens(6) s∨ ∼ q premise

∼ s by (5).̇. ∼ q by disjunctive syllogism

4.17

(1) ∼ w premiseu ∨ w premise

.̇. u by disjunctive syllogism(2) u→∼ p premise

u by (1).̇. ∼ p by modus ponens

(3) ∼ p→ r∧ ∼ s premise∼ p by (2)

.̇. r∧ ∼ s by modus ponens(4) r∧ ∼ s by (3).̇. ∼ s by conjunctive simplification

(5) t→ s premise∼ s by (4)

.̇. ∼ t by modus tollens(6) ∼ t by (5).̇. ∼ t ∨ w by disjunctive addition

Page 247: Discrete Book

243

4.18(1) ∼ (p ∨ q)→ r Premise

∼ r Premise.̇. p ∨ q Modus tollens

(2) ∼ p Premisep ∨ q by (1)

.̇. q Disjunctive Syllogism

4.19(1) p ∧ q Premise.̇. p Conjunctive simplification

(2) p ∧ q Premise.̇. q Conjunctive simplification

(3) p→∼ (q ∧ r) Premisep by (1)

.̇. ∼ (q ∧ r) Modus tonens(4) ∼ r∨ ∼ q DeMorgan′s

q by (2).̇. ∼ r Disjunctive syllogism

(5) s→ r Premise∼ r by (4)

.̇. ∼ s by Modus tollens

4.20 This follows from Modus tonens.

Page 248: Discrete Book

244 ANSWER KEY

Section 5

5.1 Let m = n = 1. Then m.n = 1 < m+ n = 2.

5.2 (b), (c), and (e) mean the same thing.

5.3 (a) All squares are rectangles.Every square is a rectangle.(b) There exists a set A which has 16 subsets.Some sets have 16 subsets.

5.4 (a) ∃ a problem x such that x has an answer.(b) ∃x ∈ R such that x ∈ Q.

5.5 (a) ∀x, if x is a COBOL program then x has at least 20 lines.(b) ∀x, if x is a valid argument with true premises then x has a true conclu-sion.(c) ∀ integers m and n, if m and n are even then m+n is also an even integer.(d) ∀ integers m and n, if m and n are odd then m · n is also an odd integer.

5.6 (b) and (d).

5.7 Correct negation : There exists an integer n such that n2 is even and nis not even.

5.8 (a) True (b) True (c) False. 26 is even and positive (d) True (e) False.A counterexample is 36.

5.9 ∃x ∈ R such that x(x+ 1) > 0 and −1 ≤ x ≤ 0.

5.10 “ ∃ an integer n such that n is divisible by 2 and n is not even.”

5.11 (a) n = 16 (b) n = 108 + 1 (c) n = 101010 + 1.

5.12 (a) For all real numbers x, there exists a real number y such thatx+ y = 0 (b) ∃x ∈ R,∀y ∈ R, x+ y 6= 0.

5.13 (a) There exists a real number x such that for all real numbers y we

Page 249: Discrete Book

245

have x + y = 0. (b) For every real number x there exists a real number ysuch that x+ y 6= 0.

5.14 “∃ a person x such that ∀ persons y, x is older than y.”

5.15 (a) ∃ a program x such that ∀ question y that is posed to x the pro-gram gives a correst answer. (b) ∀ programs x, ∃ a question y for which theprogram gives a wrong answer.

5.16 (a) ∃x ∈ R, ∀y ∈ R, x < y. (b) The given proposition is correct whereasthe one given in a. is false.

5.17 Contrapositive: ∀x ∈ R, if −1 ≤ x ≤ 0 then x(x+ 1) ≤ 0.Converse : ∀x ∈ R, if x > 0 or x < −1 then x(x+ 1) > 0.Inverse : ∀x ∈ R, if x(x+ 1) ≤ 0 then −1 ≤ x ≤ 0.

5.18 TP = {1}.

5.19 (i) This is false since P (2) and P (3) are false. (ii) This is true sinceP (1) is true.

5.20 (a) True. If x ∈ R, then y = 10− x ∈ R and x+ y = 10.(b) False. If y ∈ R and x = −y then x+ y = 0 6= 10.

Page 250: Discrete Book

246 ANSWER KEY

Section 6

6.1 m = 3 and n = 4.

6.2 By the rational zero test, possible rational zeros are: ±1,±2. One caneasily check that x = 1 is a solution to the given equation.

6.3 The two prime numbers are 11 and 13.

6.4 One such point is (1, 1).

6.5 Note first that r = 6.32 + 0.00152152.... = 632100

+ 0.00152152... Letx = 0.00152152... then 100000x = 152.152152... that is 100000x = 152 + xand solving for x we find x = 152

99999. Thus, r = 632

100+ 152

99999.

6.6 This follows from the Intermediate Value theorem where f(x) = x3 −3x2 + 2x − 4, a = 2, and b = 3. Note that f(2) = −4 and f(3) = 2 so thatf(2) < 0 < f(3).

6.7 If q =√

2√2

is rational then a = b =√

2, an irrational number. If

q =√

2√2

is irrational, let a =√

2√2

and b =√

2. In this case, ab =

(√

2√2)√2 =√

2√2·√2

=√

22

= 2, a rational number.

6.8 We have

n Sum4 2+26 3 + 38 3 + 510 5 + 512 7 + 514 7 + 716 11 + 518 11 + 720 13 + 7

6.9 We have

Page 251: Discrete Book

247

n 2n+ 12 53 75 11

6.10 Let p and q be two rational numbers. Then there exist integers a, b, c,and d with b 6= 0, d 6= 0 and p = a

b, q = c

d. Hence, pq = ac

bd∈ Q.

6.11 Let a = b in the previous theorem to find that a2 = a.a ∈ Q.

6.12 Indeed, one can easily check that x = r+s2

satisfies the inequalityr < x < s.

6.13 Let x = 5, y = 2, and z = −1. Then x > y but xz = −5 < −2 = yz.

6.14 1x+2

= 136= 1

x+ 1

2= 3

2.

6.15 Since n is even, there is an integer k such that n = 2k. Hence, (−1)n =(−1)2k = [(−1)2]k = 1k = 1.

6.16 Let n1 = 2k1 + 1 and n2 = 2k2 + 1 be two odd integers where k1and k2 are integers. Then

n1 · n2 = (2k1 + 1)(2k2 + 1) = 2(2k1k2 + k1 + k2) + 1 = 2k3 + 1

where k3 = 2k1k2 + k1 + k2 is an integer. Hence, n1 · n2 is odd.

6.17 Proving a general property can not be done by just showing it to betrue in some special cases.

6.18 By using the same letter k we end up having n = m which contra-dicts the statement that the two integers are distinct.

6.19 If the sum of two integers is even this does not mean the the inte-gers are necessarily even. For example, n = m = 1 are odd and n+m = 2 iseven. The author here jumps to a conclusion without justification.

6.20 We are trying to proof that x or y is divisible by 5 and this doesnot merely follow from the fact that the product is divisible by 5.

Page 252: Discrete Book

248 ANSWER KEY

Section 7

7.1 If n is a positive integer then n > 0 and −n < 0. Thus, the statementn = −n is false and by the method of vacuous proof, the given implicationis true.

7.2 Since x2 + 1 < 0 is false for all x ∈ R, the implication is vacuouslytrue.

7.3 Since x2−2x+ 5 = (x−1)2 + 4 > 0, the hypothesis is false and thereforethe implication is vacuously true.

7.4 For all x ∈ R, we have x2 + 1 > x2 ≥ 0. Hence, x2 + 1 > 0. Notethat the implication is proved without using the hypothesis x > 0.

7.5 Since the conclusion 62 = 36 is always true, the implication is triv-ially true even though the hypothesis is false.

7.6 We use the method of proof by cases.

Case 1. Suppose n is even. Then there is an integer k such that n = 2k.Thus, n(n+1) = 2k(2k+1) = 2k′ where k′ = k(2k+1) ∈ Z. That is, n(n+1)is even.Case 2. Suppose that n is odd. Then there exists an integer k such thatn = 2k+1. So, n(n+1) = 2(2k+1)(k+1) = 2k′ where k′ = (2k+1)(k+1) ∈ Z.Again, n(n+ 1) is even.

7.7 We use the method of proof by cases.

Case 1. Suppose n is even. Then there is an integer m such that n = 2m.Taking the square of n we find n2 = 4m2 = 4k where k = m2 ∈ Z.Case 2. Suppose n is odd. Then there is an integer m such that n = 2m+ 1.Taking the square of n we find n2 = 4m2 + 4m+ 1 = 4(m2 +m) + 1 = 4k+ 1where k = m2 +m ∈ Z.

7.8 We use the method of proof by cases.

Case 1. Suppose n is even. Then there is an integer m such that n = 2m This

Page 253: Discrete Book

249

implies that n(n2−1)(n+2) = 2m(4m2−1)(2m+2) = 4m(4m2−1)(m+1) =4k where k = m(4m2− 1)(m+ 1) ∈ Z. Thus, n(n2− 1)(n+ 2) is divisible by4.Case 2. Suppose n is odd. Then there is an integer m such that n = 2m+ 1.Thus, n(n2 − 1)(n + 2) = (2m + 1)(4m2 + 4m)(2m + 3) = 4k where k =(2m+ 1)(m2 +m)(2m+ 3) ∈ Z. Thus, n(n2 − 1)(n+ 2) is divisible by 4.

7.9 Theorem. bxc = x if and only if x ∈ Z.

Proof. Suppose that bxc = x. From the definition of the floor func-tion, x ∈ Z. Conversely, if x ∈ Z then x is the smallest integer such thatx ≤ x < x+ 1. That is, bxc = x.

7.10 If n is an even integer then there is an integer k such that n = 2k.In this case,

bn2c = bkc = k =

n

2.

7.11 As a counterexample, let x = 0 and y = −12. Then

bx− yc = b12c = 0

whereas

bxc − byc = −b−1

2c = −(−1) = 1.

7.12 As a counterexample, let x = y = 12. Then

dx+ ye = d1e = 1

whereasdxe+ dye = 2.

7.13 Suppose that x ∈ R and m ∈ Z. Let n = dxe. By definition of ceilfunction, n ∈ Z and

n− 1 < x ≤ n.

Add m to all sides to obtain

n+m− 1 < x+m ≤ n+m.

Page 254: Discrete Book

250 ANSWER KEY

Since n+m ∈ Z and n = dxe, we find

dx+me = n+m = dxe+m.

7.14 Let n be an odd integer. Then there is an integer k such that n = 2k−1.Hence, n

2= k − 1

2. By the previous problem

dn2e = dk − 1

2e = k + d−1

2e = k =

n+ 1

2.

7.15 (i) =⇒ (ii): Suppose that x2− 4 = 0. Then x2− 2x+ 2x− 4 = 0. Thus,x(x− 2) + 2(x− 2) = . Hence, (x− 2)(x+ 2) = 0.(ii) =⇒ (iii): If (x− 2)(x+ 2) = 0 then either x = 2 or x = −2.(iii) =⇒ (i): Suppose x = −2 or x = 2. Then x+ 2 = 0 or x− 2 = 0. Hence,(x− 2)(x+ 2) = 0. Expanding on the left, we find x2 − 4 = 0.

7.16 (a) =⇒ (b): Suppose x < y. Then 2x = x+ x < x+ y. Hence, x+y2> x.

(b) =⇒ (c): Suppose x+y2

> x. Then x < y. Hence, x + y < y + y = 2y.Dividing through by 2, we find x+y

2< y.

(c) =⇒ (a): Suppose x+y2

< y. Then x + y < 2y. Subtracting y from bothsides, we find x < y.

7.17 (a) =⇒ (b): Suppose x2− 5x+ 6 = 0. Factoring the righthabd side, wefind (x− 2)(x− 3) = 0.(b) =⇒ (c): Suppose (x − 2)(x − 3) = 0. By the zero product property,x− 2 = 0 or x− 3 = 0.(c) =⇒ (d): Suppose x− 2 = 0 or x− 3 = 2. Adding 2 to the first equationyields x = 2. Adding 3 to the second equation yields x = 3.(d) =⇒ (a): Suppose x = 2 or x = 3. Then (x − 2)(x − 3) = 0. Expandingthe righthand side to obtain x2 − 5x+ 6 = 0.

Page 255: Discrete Book

251

Section 8

8.1 Suppose the contrary. That is, suppose there is a greatest even inte-ger N. Then for any even integer n we have N ≥ n. Define the numberM := N + 2. Since the sum of two even integers is again an even integer, Mis even. Moreover, M > N. This contradicts the supposition that N is thelargest even integer. Hence, there in no greatest even integer.

8.2 Suppose the contrary. That is, suppose there exist a rational numberx and an irrational number y such that x − y is rational. By the definitionof rational numbers, there exist integers a, b, c, and d with b 6= 0 and d 6= 0such that x = a

band x− y = c

d. Thus,

a

b− y =

c

d.

Solving for y we find

y =ad− bcbd

=p

q

where p = ad−bc ∈ Z and q = bd ∈ Z−{0}. This contradicts the assumptionthat y is irrational.

8.3 We must prove by the method of contraposition the implication “if xand y are positive real numbers such that xy > 100 then either x > 10 ory > 10. Instead, we will show that if x ≤ 10 and y ≤ 10 then xy ≤ 100.Indeed, if 0 < x ≤ 10 and 0 < y ≤ 10 then the algebra of inequalities wehave xy ≤ 100.

8.4 Suppose the contrary. That is, suppose there exist a rational numberx and an irrational number y such that xy ∈ Q. By the definition of rationalnumbers there exist integers a, b 6= 0, c, and d 6= 0 such that x = a

band

xy = cd. Since x 6= 0, it has a multiplicative inverse, that is we can multiply

both side of the equality xy = cd

by ba

to obtain y = bcad

= pq

where p = bc ∈ Zand q = ad ∈ Z−{0}. This shows that y ∈ Q which contradcits the assump-tion that y is irrational.

8.5 Suppose not. That is, suppose that n is not divisible by 3. Then ei-ther n = 3k + 1 or n = 3m + 2 for some integers k and m. If n = 3k + 1then n2 = 9k2 + 6k + 1 = 3(3k2 + 2k) + 1 = 3k′ + 1. This means that

Page 256: Discrete Book

252 ANSWER KEY

n2 is not divisible by 3, a contradiction. Likewise, if n = 3m + 2 thenn2 = 9m2 + 12m + 4 = 3(3m2 + 4m + 1) + 1 = 3m′ + 1. Again, this meansthat n2 is not divisible by 3, a contradiction. We conclude that n must bedivisible by 3.

8.6 Suppose not. That is, suppose that√

3 is rational. Then there existtwo integers m and n with no common divisors such that

√3 = m

n. Squaring

both sides of this equality we find that 3n2 = m2. Thus, m2 is divisible by3. By the previous problem, m is divisible by 3. But then m = 3k for someinteger k. Taking the square we find that 3n2 = m2 = 9k2, that is n2 = 3k2.This says that n2 is divisible by 3 and so n is divisible by 3. We concludethat 3 divides both m and n and this contradicts our assumption that m andn have no common divisors. Hence,

√3 must be irrational.

8.7 Suppose that n is even and m is odd. Then n = 2k and m = 2k′ + 1.Thus, n+m = 2k+ 2k′+ 1 = 2(k+ k′) + 1 = 2k′′+ 1. That is, m+n is odd.

8.8 Suppose that n is even. Then n = 2k for some integer k. Then 3n+ 1 =6k + 1 = 2(3k) + 1 = 2k′ + 1. That is, 3n+ 1 is odd.

8.9 Suppose the contrary. Then (n − m)(n + m) = 2 implies one the twosystems of equations {

n−m = 2n+m = 1

or {n−m = 1n+m = 2.

Solving the first system we find n = 32

and m = −12, contradicting our as-

sumption that n and m are positive integers. Likewise, solving the secondsystem, we find n = 3

2and m = 1

2which again contradicts our assumption.

8.10 Suppose the contrary. Let n and m be integers such that n2− 4m = 2.This implies that n2 = 4m + 2 = 2(2m + 1) = 2k. That is, n2 is even.Hence, n is also even. Thus, n = 2b for some integer b. In this case,n2 − 4m = 4b2 − 4m = 2 and this implies 2b2 − 2m = 1. The lefthandside is even where as the righthand side is odd, a contradiction.

Page 257: Discrete Book

253

8.11 Suppose that n is a perfect square. Then n = k2 for some k ∈ Z.• If the remainder of the division of k by 3 is 0 then k = 3m1 and n = 9m2

1 =4(4m2

1) so that the remainder of the division of n by 3 is 0.• If the remainder of the division of k by 3 is 1 then k = 3m2 + 1 andn = 3(3m2

2 + 2m2) + 1 so that the remainder of the division of n by 3 is 1.• If the remainder of the division of k by 3 is 2 then k = 3m3 + 2 andn = 3(3m2

3 + 2m3 + 1) + 1 so that the remainder of the division of n by 4 is1.

8.12 Suppose that either a or b is divisible by 5. Let’s say that a is di-visible by 5. Then a = 5m for some m ∈ Z. Hence, ab = 5(mb) and mb ∈ Z.This says that a · b is divisible by 5. Likewise, if we assume that b is divisibleby 5 then b = 5n for some n ∈ Z. Hence, ab = 5(na) and na ∈ Z. This saysthat a · b is divisible by 5.

Page 258: Discrete Book

254 ANSWER KEY

Section 9

9.1 Let P (n) = 2 + 4 + 6 + · · · + 2n. We will show that P (n) = n2 + nfor all n ≥ 1 by the method of mathematical induction.(i) (Basis of induction) P (1) = 2 = 12 + 1. So P (n) holds for n = 1.(ii) (Induction hypothesis) Assume P (n) is true. That is, P (n) = n2 + n.(iii) (Induction step) We must show that P (n+1) = (n+1)2 +n+1. Indeed,

P (n+ 1) = 2 + 4 + · · ·+ 2n+ 2(n+ 1)= P (n) + 2(n+ 1)= n2 + n+ 2n+ 2= n2 + 2n+ 1 + (n+ 1)= (n+ 1)2 + (n+ 1)

9.2 Let P (n) = 1 + 2 + 22 + · · · + 2n. We will show, by induction on n ≥ 0that P (n) = 2n+1 − 1.

(i) (Basis of induction) P (0) = 1 = 20+1 − 1. That is, P (1)is true.(ii) (Induction hypothesis) Assume P (n) is true. That is, P (n) = 2n+1 − 1.(iii) (Induction step) We must show that P (n+ 1) = 2n+2 − 1. Indeed,

P (n+ 1) = 1 + 2 + · · ·+ 2n + 2n+1

= P (n) + 2n+1

= 2n+1 − 1 + 2n+1

= 2.2n+1 − 1= 2n+2 − 1

9.3 Let P (n) = 12 + 22 + · · ·+ n2. We will show by mathematical induction

that P (n) = n(n+1)(2n+1)6

for all integers n ≥ 1.

(i) (Basis of induction) P (1) = 1 = 1(1+1)(2+1)6

. That is, P (1)is true.

(ii) (Induction hypothesis) Assume P (n) is true. That is, P (n) = n(n+1)(2n+1)6

.

(iii) (Induction step) We must show that P (n+ 1) = n(n+1)(2n+1)6

. Indeed,

P (n+ 1) = 12 + 22 + · · ·+ (n+ 1)2

= P (n) + (n+ 1)2

= n(n+1)(2n+1)6

+ (n+ 1)2

= (n+1)(n+2)(2n+3)6

Page 259: Discrete Book

255

9.4 Let P (n) = 13 + 23 + · · ·+ n3. We will show by mathematical induction

that P (n) = (n(n+1)2

)2 for all integers n ≥ 1.

(i) (Basis of induction) P (1) = 1 = (1(1+1)2

)2. That is, P (1)is true.

(ii) (Induction hypothesis) Assume P (n) is true. That is, P (n) = (n(n+1)2

)2.

(iii) (Induction step) We must show that P (n+ 1) = ( (n+1)(n+2)2

)2. Indeed,

P (n+ 1) = 13 + 23 + · · ·+ (n+ 1)3

= P (n) + (n+ 1)3

= (n(n+1)2

)2 + (n+ 1)3

= ( (n+1)(n+2)2

)2

9.5 Let P (n) = 11·2 + 1

2·3 + · · ·+ 1n(n+1)

. We will show by mathematical induc-

tion that P (n) = nn+1

for all integers n ≥ 1.

(i) (Basis of induction) P (1) = 11·2 = 1

1+1. That is, P (1)is true.

(ii) (Induction hypothesis) Assume P (n) is true. That is, P (n) = nn+1

.

(iii) (Induction step) We must show that P (n+ 1) = n+1n+2

. Indeed,

P (n+ 1) = 11·2 + 1

2·3 + · · ·+ 1n(n+1)

+ 1(n+1)(n+2)

= P (n) + 1(n+1)(n+2)

= nn+1

+ 1(n+1)(n+2)

= n(n+2)+1(n+1)(n+2)

= (n+1)2

(n+1)(n+2)

= n+1n+2

9.6 500,497

9.7 2− 12n

9.8 By substitution we find S(1) = 12, S(2) = 5

6, S(3) = 23

24, S(4) = 119

120, S(5) =

719720. We claim that P (n) : S(n) = 1 − 1

(n+1)!. We will prove this formula by

induction on n ≥ 1.

(i) (Basis of induction) S(1) = 11·2 = 1− 1

(1+1)!. That is, P (1)is true.

(ii) (Induction hypothesis) Assume P (n) is true. That is, S(n) = 1− 1(n+1)!

.

Page 260: Discrete Book

256 ANSWER KEY

(iii) (Induction step) We must show that P (n + 1) : S(n + 1) = 1 − 1(n+2)!

.Indeed,

n+1∑k=1

k

k(k + 1)!=

n∑k=1

k

k(k + 1)!+

n+ 1

(n+ 2)!

=1− 1

(n+ 1)!+

n+ 1

(n+ 1)!(n+ 2)

=1− (n+ 2− n− 1

(n+ 2)(n+ 1)!

=1− 1

(n+ 2)!

9.9 (a) P (1) : 41 − 1 is divisible by 3. P (1) is true.(b) P (k) : 4k − 1 is divisible by 3.(c) P (k + 1) : 4k+1 − 1 is divisible by 3.(d) We must show that if 4k−1 is divisible by 3 then 4k+1−1 is divisible by 3.

9.10 (i) (Basis of induction) P (1) : 23 − 1. P (1)is true since 7 is divisi-ble by 7.(ii) (Induction hypothesis) Assume P (n) is true. That is, 23n − 1 is divisibleby 7.(iii) (Induction step) We must show that P (n+ 1) is true. That is, 23n+3− 1is divisible by 7. Indeed,

23n+3 − 1 = (1 + 7) · 23n − 1= 23n − 1 + 7 · 23n

Since 7|(23n − 1) and 7|7 · 23n, we find 7 divides the sum. that is, 7 divides23n+3 − 1.

9.11 Let P (n) = (n + 2)! − 2n. We use mathematical induction to showthat P (n) > 0 for all n ≥ 0.

(i) (Basis of induction) P (0) = 2!− 1 = 1 > 0 so that P (1) is true.(ii) (Induction hypothesis) Assume P (n) is true. That is, P (n) = (n+ 2)!−2n > 0.

Page 261: Discrete Book

257

(iii) (Induction step) We must show that P (n + 1) = (n + 3)! − 2n+1 > 0.Indeed,

P (n+ 1) = (n+ 3)!− 2n+1

= (n+ 3)(n+ 2)!− 2n+1

≥ 2(n+ 2)!− 2 · 2n

= 2((n+ 2)!− 2n)> 0

9.12 (a) Let P (n) = n3 − 2n − 1. We want to show that P (n) > 0 for allintegers n ≥ 2.

(i) (Basis of induction) P (2) = 23 − 4− 1 = 3 > 0 so that P (1) is true.(ii) (Induction hypothesis) Assume P (n) is true. That is, P (n) = n3 − 2n−1 > 0.(iii) (Induction step) We must show that P (n+ 1) > 0. Indeed,

P (n+ 1) = (n+ 1)3 − 2(n+ 1)− 1= n3 + 3n2 + 3n+ 1− 2n− 3= n3 − 2n− 1 + 3n2 + 3n− 1> 3n2 + 3n− 1> 0

note that for n ≥ 2, 3n2 + 3n− 1 ≥ 15

(b) Let P (n) = n! − n2. We want to show that P (n) > 0 for all integersn ≥ 4.

(i) (Basis of induction) P (4) = 4!− 42 = 8 > 0 so that P (1) is true.(ii) (Induction hypothesis) Assume P (n) is true. That is, P (n) > 0.(iii) (Induction step) We must show that P (n+ 1) > 0. Indeed,

P (n+ 1) = (n+ 1)!− (n+ 1)2

= (n+ 1)n!− n2 − 2n− 1> n!− n2 + nn!− 2n− 1> nn!− 2n− 1= n(n!− 2)− 1 > 0

note that for n ≥ 4, n! ≥ 24 so that n(n!− 2)− 1 ≥ 87.

Page 262: Discrete Book

258 ANSWER KEY

9.13 Listing the first few terms of the sequence we find, a1 = 3 = 3·71−1, a2 =3 · 72−1, etc. We will use induction on n ≥ 1 to show that the formula for anis valid for all n ≥ 1.

(i) (Basis of induction) a1 = 3 = 3 · 71−1 so that the formula is true forn = 1.(ii) (Induction hypothesis) Assume that an = 3 · 7n−1

(iii) (Induction step) We must show that an+1 = 3 · 7n.

an+1 = 7an= 7 · 3 · 7n−1

= 3 · 7n.

Page 263: Discrete Book

259

Section 1010.1 Let R+ denote the set of all positive real numbers. If x ∈ R+ then x

2∈

R+ and x2< x. Thus, R+ does not have a least element.

10.2 Let S be the set of positive integers n ≥ 8 that cannot be writtenas sums of 3’s and 5’s. We must show that S is the empty set. Suppose thecontrary. Then by Theorem 10.1, S has a smallest element m ∈ S. Since8 = 3 + 5, 9 = 3 + 3 + 3 and 10 = 5 + 5, we must have m ≥ 11. This impliesthat m − 3 ≥ 8. If m − 3 can be written as sums of 3’s and 5’s so does mwhich is not possible since m ∈ S. Hence, m − 3 ∈ S and m − 3 < m. Thiscontradicts the definition of m. Hence, we must have S = ∅.

10.3 Since a ∈ S, S is non-empty subset of N and so by Theorem 10.1,S has a smallest element m. In this case, m

b=√

2. Hence, m2 = 2b2 so thatm2 is even and hence m is even. Thus, m = 2k with k ∈ N. Hence, 2b2 = 4k2

or b2 = 2k2. It follows that kb

=√

2 and hence k ∈ S with k < m. Thiscontradicts the definition of m. We conclude that S is empty.

10.4 Since n|m, we have m = nq for some q ∈ Z. Thus, −m = (n)(−q)and hence n|(−m).

10.5 Suppose that n|a and n|b. Then a = nq and b = nq′ for some q, q′ ∈ Z.Thus, a± b = n(q ± q′). Hence, n|(a± b).

10.6 Suppose that n|m and m|p. Then m = nq and p = mq′ for someq, q′ ∈ Z. Thus, p = n(qq′). Since qq′ ∈ Z then n|p.

10.7 If n|m and m|n then m = nq and n = mq′ for some q, q′ ∈ Z. Thus,m = mqq′ or (1 − qq′)m = 0. Since m 6= 0, qq′ = 1. This only true if eitherq = q′ = 1 or q = q′ = −1. That is, n = m or n = −m.

10.8 The numbers are : 2, 3, 5, 7, 11, 13, 17.

10.9 The prime numbers less that or equal to√

227 are: 2, 3, 5, 7, 11,13. Since none of them divides 227, by Theorem 10.5, we conclude that 227is prime.

10.10 42 = 2× 3× 7 and 105 = 3× 5× 7.

Page 264: Discrete Book

260 ANSWER KEY

10.11 21.

10.12 Yes.

10.13 (a) Since 6m+ 8n = 2(3m+ 4n) and 3m+ 4n ∈ Z, we have 6m+ 8nis even (b) Since 6m + 4n2 + 3 = 2(3m + 2n2 + 2) + 1 = 2k + 1 wherek = 3m+ 2n2 + 2, we have 6m+ 4n2 + 3 is odd.

10.14 If a|b then there is an integer k such that b = ka. Multiply bothsides of this equality by c to obtain bc = (kc)a. Thus, a|bc.

10.15 If m = 2 and n = 1 then m2 − n2 = 3 which is prime. Depend-ing on m and n the number m2 − n2 can be either prime or composite.

Page 265: Discrete Book

261

Section 11

11.1 (i) 20, (ii) gcd(17,21)=1.

11.2 If a ≡ b(mod n) and c ≡ d(mod n) then there exist integers k1 andk2 such that a − b = k1n and c − d = k2n. Adding these equations we find(a+ c)− (b+ d) = kn where k = k1 + k2 ∈ Z. Hence, a+ c ≡ (b+ d)(mod n).

11.3 Using the notation of the previous problem, we he ac − bc = k1cnand cb − bd = k2bn. Adding these equations we find ac − bd = kn wherek = k1c+ k2b ∈ Z. Hence, ac ≡ bd(mod n).

11.4 Since a− a = n(0), we have a ≡ a(mod n).

11.5 From a ≡ b(mod n), we have a − b = nq for some q ∈ Z. Multiplythrough by −1 to obtain b− a = n(−q) = nq′. That is, b ≡ a(mod n).

11.6 From a ≡ b(mod n) and b ≡ c(mod n), we obtain a − b = nq1and b − c = nq2 where q1, q2 ∈ Z. Adding these two equations, we finda− c = n(q1 + q2) = nq. Hence, a ≡ c(mod n).

11.7 By previous problem, we have 9x ≡ 12(mod 7). Since 7x ≡ 0(mod 7),using Problem 11.2, we have 2x ≡ 12(mod 7). This, the given equation, andProblem 11.2 lead to x ≡ −8(mod 7). Hence, x = 7k − 8 where k ∈ Z.

11.8 From −11 ≡ −11(mod 3) and −4 ≡ 2(mod 3) we can write 2x ≡2(mod 3). Multiply by 2 to obtain 4x ≡ 4(mod 3). Thus, 4x ≡ 1(mod 3).From −3x ≡ 0(mod 3) we obtain x ≡ 1(mod 3).

11.9 20.

11.10 2.

11.11 1.

11.12 3.

Page 266: Discrete Book

262 ANSWER KEY

Section 12

12.1 (a) A = ∅.(b) A = {2, 3, 5, 7, 11}.(c) A = {−1}.

12.2 (a) A = {x ∈ N|x ≤ 5}.(b) A = {x ∈ N|x is composite ≤ 10}.

12.3 {a, b, c, d} = {d, b, a, c} and {d, e, a, c} = {a, a, d, e, c, e}.

12.4 (a) B is not a subset of A since j ∈ B but j 6∈ A.(b) C ⊆ A.(c) C ⊆ C.(d) C is a proper subset of A since C ⊆ A and c ∈ A but c 6∈ C.

12.5 (a) 3 ∈ {1, 2, 3}.(b) 1 6⊆ {1}.(c) {2} ⊆ {1, 2}(d) {3} ∈ {1, {2}, {3}}(e) 1 ∈ {1}(f) {2} ∈ {1, {2}, {3}}(g) {1} ⊆ {1, 2}(h) 1 6∈ {{1}, 2}(i) {1} ⊆ {1, {2}}(j) {1} ⊆ {1}.

12.6 (a) A ∪B = {a, b, c, d, f, g}.(b) A ∩B = {b, c}.(c) A−B = {d, f, g}.(d) B − A = {a}.

12.7 (a) True (b) False (c) False (d) False (e) True (f) False (g) True (h)True.

12.8 (a) A×B = {(x, a), (x, b), (y, a), (y, b), (z, a), (z, b), (w, a), (w, b)}.(b) B × A = {(a, x), (b, x), (a, y), (b, y), (a, z), (b, z), (a, w), (b, w)}.

Page 267: Discrete Book

263

(c)

A× A ={(x, x), (x, y), (x, z), (x,w), (y, x), (y, y), (y, z), (y, w),

(z, x), (z, y), (z, z), (z, w), (w, x), (w, y), (w, z), (w,w)}.

(d) B ×B = {(a, a), (a, b), (b, a), (b, b)}.

12.9 (a) {∅, {a}, {b}, {c}, {a, b}, {a, c}, {b, c}, A}.(b) There are 7 proper subsets, namely,

{∅, {a}, {b}, {c}, {a, b}, {a, c}, {b, c}}.

12.10

(a) 55 sandwiches with tomatoes or onions.(b) There are 40 sandwiches with onions.(c) There are 10 sandwiches with onions but not tomatoes.

12.11 (a) Students speaking English and spanish, but not french, are inthe brown region; there are 20 of these.(b) Students speaking none of the three languages are outside the three cir-cles; there are 5 of these.(c) Students speaking french, but neither english nor spanish, are in the blueregion; there are 11 of these.(d) Students speaking english, but not spanish, are in the yellow and grayregions; there are 25 + 17 = 42 of these. (e) Students speaking only one ofthe three languages are in the yellow, green, and blue regions; there are 25+ 10 + 11 = 46 of these.(f) Students speaking two of the three languages are in the pink, gray, andbrown regions; there are 9 + 17 + 20 = 46 of these.

Page 268: Discrete Book

264 ANSWER KEY

12.12 First, writing the tabular form of each set, we find A = {4, 6, 8, 9, 10}and B = {1, 2, 3, 4, 5, 6, 7, 8}. Thus, A−B = {4, 6, 8} and B = {1, 2, 3, 5, 7}.

12.13 We have U c = ∅ and ∅c = U.

12.14 Note that A = ∅ so that A ∪B = B and A ∩B = ∅.

12.15 We have A ∪B = {2, 4, 5, 6, 7, 8} and A ∩B = {6}.

12.16

Page 269: Discrete Book

265

Section 13

13.1 Let x ∈ A ∩ C. Then x ∈ A and x ∈ C. Since A ⊆ B, we havex ∈ B. Thus, x ∈ B and x ∈ C so that x ∈ B ∩ C.

13.2 Let A = {1}, B = {2}, and C = {3}. Then A ∩ C = B ∩ C = ∅and A 6= B.

13.3 Let A be a proper subset of a set B and let C = ∅. Then ∅ = A ∩ C ⊆B ∩ C = ∅ and A = A ∪ C ⊆ B ∪ C. However, A 6= B.

13.4 Let x ∈ Bc. If x 6∈ Ac then x ∈ A. Since A ⊆ B, we have x ∈ B.That is, x 6∈ Bc. This contradicts our assumption that x ∈ Bc. Hence, x ∈ Ac.

13.5 Let x ∈ A ∪ B. Then either x ∈ A or x ∈ B. In either case we havex ∈ C since both A and B are subsets of C. This shows that A ∪B ⊆ C.

13.6 We first show that A × (B ∪ C) ⊆ (A × B) ∪ (A × C). Let (x, y) ∈A×(B∪C). Then x ∈ A and y ∈ B∪C. So either y ∈ B or y ∈ C. Hence, wehave either (x, y) ∈ A×B or (x, y) ∈ A×C. Thus, (x, y) ∈ (A×B)∪(A×C).Next we show that (A×B)∪ (A×C) ⊆ A× (B ∪C). Let (x, y) ∈ (A×B)∪(A × C). Then either (x, y) ∈ A × B or (x, y) ∈ A × C. If (x, y) ∈ A × Bthen x ∈ A and y ∈ B. This implies that x ∈ A and y ∈ B ∪ C and so(x, y) ∈ A× (B ∪ C). Similar argument if (x, y) ∈ A× C.

13.7 Let (x, y) ∈ A × (B ∩ C). Then x ∈ A, y ∈ B, and y ∈ C. Thus,(x, y) ∈ A×B and (x, y) ∈ A×C. It follows that (x, y) ∈ (A×B)∩ (A×C).This shows that A× (B ∩ C) ⊆ (A×B) ∩ (A× C).Conversely, let (x, y) ∈ (A × B) ∩ (A × C). Then (x, y) ∈ A × B and(x, y) ∈ A×C. Thus, x ∈ A, y ∈ B and y ∈ C. That is, (x, y) ∈ A× (B ∩C).

13.8 (a) The empty set contains no elements. Thus 0 6∈ ∅.(b) The set {∅} has one element, namely ∅, whereas ∅ contains no elements.Thus ∅ 6= {∅}.(c) The set {∅} has one element, namely ∅. That is, ∅ ∈ {∅}.

13.9 Suppose not. Then there is an x ∈ (A−B)∩ (A∩B). This implies thatx ∈ A−B, x ∈ A, and x ∈ B. But x ∈ A−B implies that x ∈ A and x 6∈ B.

Page 270: Discrete Book

266 ANSWER KEY

So we have x ∈ B and x 6∈ B a contradiction. Hence, (A−B)∩ (A∩B) = ∅.

13.10 Suppose that A ∩ Bc 6= ∅. Then there is an x ∈ A ∩ Bc. This im-plies that x ∈ A and x ∈ Bc. Thus, x ∈ A and x 6∈ B. Since A ⊆ B andx ∈ A, we have x ∈ B. It follows that x ∈ B and x 6∈ B which is impossible.

13.11 Suppose the contrary. That is, A ∩ C 6= ∅. Then there is an ele-ment x ∈ A and x ∈ C. Since A ⊆ B, we have x ∈ B. So we have thatx ∈ B and x ∈ C. This yields that x ∈ B∩C which contradicts the fact thatB ∩ C = ∅.

13.12 Let A = {1} and B = {2}. Then A∩B = ∅ and A×B = {(1, 2)} 6= ∅.

13.13 (a) Since A ∩B = {2}, we have P(A) = {∅, {2}}.(b) P(A) = {∅, {1}, {2}, A}.(c) Since A ∪B = {1, 2, 3}, we have

P(A ∪ B) = {∅, {1}, {2}, {3}, {1, 2}, {1, 3}, {2, 3}, {1, 2, 3}}.

(d) Since A×B = {(1, 2), (1, 3), (2, 2), (2, 3)}, we have

P(A× B) = {∅, {(1, 2)}, {(1, 3)}, {(2, 2)}, {(2, 3)}, {(1, 2), (1, 3)},{(1, 2), (2, 2)}, {(1, 2), (2, 3)}, {(1, 3), (2, 2)}, {(1, 3), (2, 3)},{(2, 2), (2, 3)}, {(1, 2), (1, 3), (2, 2)}, {(1, 2), (1, 3), (2, 3)},{(1, 2), (1, 3), (2, 3)}, {(1, 3), (2, 2), (2, 3)}, A×B}

13.14 (a) P(∅) = {∅}.(b) P(P(∅)) = {∅, {∅}}.(c) P(P(P(∅))) = {∅, {∅}, {{∅}}, {∅, {∅}}}.

13.15 (a) Let A = {1, 2} and B = {1, 3}. Then A ∪B = {1, 2, 3} andP(A ∪ B) = {∅, {1}, {2}, {3}, {1, 2}, {1, 3}, {2, 3}, {1, 2, 3}} whereasP(A) ∪ P(B) = {∅, {1}, {2}, {1, 2}, {3}, {1, 3}}.(b) Let X ∈ P(A ∩ B). Then X ⊆ A ∩ B and therefore X ⊆ A and X ⊆ B.This implies that X ∈ P(A) and X ∈ P(B) and hence X is in the intersec-tion. This completes a proof of P(A ∩ B) ⊆ P(A) ∩ P(B).Conversely, let X ∈ P(A) ∩ P(B). Then X ∈ P(A) and X ∈ P(B). Hence,X ⊆ A and X ⊆ B. It follows that X ⊆ A∩B and therefore X ∈ P(A ∩ B).

Page 271: Discrete Book

267

This ends a proof for P(A) ∩ P(B) ⊆ P(A ∩ B).(c) Let X ∈ P(A) ∪ P(B). Then X ⊆ A and X ⊆ B. Thus, X ⊆ A ∪B andthis implies that X ∈ P(A ∪ B).(d) Let A = {1} and B = {2}. Then A×B = {(1, 2)}. So we have

P(A× B) = {∅, {(1, 2)}}

whereas

P(A)× P(B) = {(∅, ∅), (∅, {2}), ({1}, ∅), ({1}, {2})}

Page 272: Discrete Book

268 ANSWER KEY

Section 14

14.1 We have

x� x =(x� x)⊕ 0 = (x� x)⊕ (x� x)

=x� (x⊕ x)

=x� 1 = x.

14.2

x� 0 =x� (x� x)

=(x� x)� x=x� x = 0.

14.3

(x⊕ y)� x =(x� x)⊕ (x� y)

=x⊕ (x� y)

=x.

14.4

x⊕ (x� y) =(x⊕ x)� (x⊕ y)

=1� (x⊕ y)

=x⊕ y.

14.5 Since x � v = x for all x ∈ S, we can choose x = 1 and obtain1 = 1� v = v.

14.6

(x⊕ y)� (x� y) =[x� (x� y)]⊕ [y � (x� y)]

=[(x� x)� y]⊕ [x� (y � y)]

=(0� y)⊕ (0� x)

=0⊕ 0 = 0

Page 273: Discrete Book

269

and

(x⊕ y)⊕ (x� y) =[(x⊕ y)⊕ x]� [(x⊕ y)⊕ y]

=[(x⊕ x)⊕ y]� [x⊕ (y ⊕ y)]

=1⊕ y � x⊕ 1

=1.

Hence, by Example 14.6, we have x⊕ y = x� y.

14.7 Let y = 0 and x = 1 in Example 14.6 an notice that 1 ⊕ 0 = 1and 1� 0 = 0 to conclude that 1 = 0. Similar argument for 0 = 1.

14.8 We have x� (x⊕ y) == x� x⊕ x� y = 0⊕ x� y = x� y.

Page 274: Discrete Book

270 ANSWER KEY

Section 15

15.1 8.

15.2 A× A = {(1, 1), (1, 2), (1, 3), (2, 1), (2, 2), (2, 3), (3, 1), (3, 2), (3, 3)}.

15.3 x = 2 and y = −1.

15.4 Dom(R) = {−12,−6, 5, 6} and Range(R) = {−6, 4, 6}.

15.5 (a) f is a function from A to B since every element in A maps toexactly one element in B. The range of f is the set {a, d}.(b) f is not a function since the element 2 is related to two members of B.

15.6

15.7 R−1 = {(1, a), (5, b), (2, c), (1, d)}. R−1 is not a function since 1 is re-lated to a and d.

15.8

R ∪ S ={(1, 1), (1, 2), (1, 3), (1, 4), (2, 2), (2, 3), (3, 4), (4, 1), (4, 2)}R ∩ S ={(1, 2), (1, 3)}R− S ={(1, 4), (2, 2), (3, 4), (4, 1), (4, 2)}S −R ={(1, 1), (2, 3)}.

15.9 S ◦R = {(a, a), (a, b), (a, g), (b, b), (b, g), (c, a)}.

Page 275: Discrete Book

271

15.10 (a) {a, b}R{b, c} since |{a, b}| = |{b, c}| = 2.(b) Since 1 = |{a}| 6= 2 = |{a, b}|, we have {a} 6 R{a, b}.(c) Since |{c}| = |{b}|, we have {c}R{b}.

15.11

(a)(b) R is not a function since (4, y) 6∈ R for any y ∈ B. S defines a functionsince it satisfies the definition of a function. Finally, T is not a function since(6, 5) ∈ R and (6, 7) ∈ R but 5 6= 7.

15.12R = {(3, 4), (3, 5), (3, 6), (4, 5), (4, 6), (5, 6)}R−1 = {(4, 3), (5, 3), (6, 3), (5, 4), (6, 4), (6, 5)}.

15.13

A×B = {(2, 6), (2, 8), (2, 10), (4, 6), (4, 8), (4, 10)}

R = {(2, 6), (2, 8), (2, 10), (4, 8)}

S = {(2, 6), (4, 8)}

Page 276: Discrete Book

272 ANSWER KEY

R ∪ S = R

R ∩ S = S.

15.14 Since x ≥ x for any number x ∈ R, R is reflexive. R is not symmetricsince 2 ≥ 1 but 1 6> 2. Finally, R is transitive because if x ≥ y and y ≥ zthen x ≥ z.

15.15 Since xx ≥ 0 for all x ∈ R, R is reflexive. If x, y ∈ R are suchthat xy ≥ 0 then yx ≥ 0 since multiplication of real numbers is commuta-tive. Thus, R is symmetric. Finally, R is transitive for if xy ≥ 0 then x andy have the same sign. Similarly, if yz ≥ 0 then y and z have the same sign.It follows that all three numbers x, y, and z have the same sign.

15.16 Since every set is a subset of itself, R is reflexive. However, R isnot symmetric since it is possible to find two subsets X and Y of P(A) suchthat X ⊂ Y. Finally, for A ⊆ B and B ⊆ C then A ⊆ C. That is, R istransitive.

15.17 R is reflexive: For all m ∈ Z, m ≡ m (mod 2) since m−m = 2 · 0.R is symmetric: If m,n ∈ Z are such that m ≡ n(mod 2) then there is aninteger k such that m − n = 2k. Multiplying this equation by −1 to obtainn−m = 2(−k) so that n ≡ m (mod 2). That is, n R m.R is transitive: Let m,n, p ∈ Z be such that m ≡ n (mod 2) and n ≡p (mod 2). Then there exist integers k1 and k2 such that m − n = 2k1 andn− p = 2k2. Add these equalities to obtain m− p = 2(k1 + k2) which meansthat m ≡ p (mod 2).If x ∈ [a] then x = 2k+a so that a is the remainder of the division of x by 2.Thus the only possible values of a are 0 and 1 and so the equivalence classesare [0] and [1].

15.18 R is reflexive: For all a ∈ R we have a I a since a− a = 0 ∈ Z.R is symmetric: If a, b ∈ R are such that a I b then a − b ∈ Z. But thenb− a = −(a− b) ∈ Z. That is, b I a.R is transitive: If a, b, c ∈ R are such that a I b and b I c then a− b ∈ Z andb− c ∈ Z. Adding to obtain a− c ∈ Z that is a I c.For a ∈ R, [a] = {b ∈ R|a = b+ n, for some n ∈ Z}.

Page 277: Discrete Book

273

15.19 || is reflexive: Any line is parallel to itself.|| is symmetric: If l1 is parallel to l2 then l2 is parallel to l1.|| is transitive: If l1 is parallel to l2 and l2 is parallel to l3 then l1 is parallelto l3.If l ∈ A then [l] = {l′ ∈ A|l′||l}.

15.20 (a) R is reflexive since a+ a = a+ a, i.e. (a, a) R (a, a).(b) R is symmetric: If (a, b) R (c, d) then a + d = b + c which implies thatc+ b = d+ a. That is, (c, d) R (a, b).(c) R is transitive: Suppose (a, b) R (c, d) and (c, d) R (e, f). Then a+d = b+cand c + f = d + e. Multiply the first equation by -1 and add to the secondto obtain a+ f = d+ e. That is, (a, b) R (e, f).(d) (1, 1), (2, 2), (3, 3), (4, 4), (5, 5) ∈ [(1, 1)].(e) (3, 1), (4, 2), (5, 3), (6, 4), (7, 5) ∈ [(3, 1)].(f) (0, 1), (1, 2), (2, 3), (3, 4), (4, 5) ∈ [(1, 2)].(g) [(a, b)] = {(c, d)|d− c = b− a}.

15.21 We must show that x R x for all x ∈ A. Let x ∈ A. Then thereis a y ∈ A such that x R y. Since R is symmetric, y R x. The facts thatx R y, y R x and R transitive imply that x R x.

Page 278: Discrete Book

274 ANSWER KEY

Section 16

16.1 For all m ∈ Z we have that m + m is even and so m R m. Thisshows that R is reflexive. R is not antisymmetric sine 2 R 4 and 4 R 2 but2 6= 4. Hence, R is not a partial order relation.

16.2 R is not antisymmetric since 2 R (−2) and (−2) R 2 but 2 6= −2.Hence, R is not a partial order relation.

16.3

16.4 a. Reflexivity: for all 0 ≤ i ≤ n, we have 2i|2i.Antisymmetry: Suppose that 2i|2j and 2j|2i. Then i ≤ j and j ≤ i. Since therelation ≤ is antisymmetric on N we conclude that i = j. That is 2i = 2j.Transitivity: Suppose that 2i|2j and 2j|2k. Then i ≤ j and j ≤ k. Since ≤ istransitive on N we conclude that i ≤ k. Hence, 2i|2k.Now, if 2i and 2j are element of A then either i ≤ j or j ≤ i. That is, either2i|2j or 2j|2i. So A is a totally ordered set.b.

Page 279: Discrete Book

275

16.5 R−1 is reflexive: Since aRa for all a ∈ A, we have aR−1a.R−1 is antisymmetric: Suppose that aR−1b and bR−1a. Then aRb and bRa.Since R is antisymmetric, we have a = b.R−1 is reflexive: Suppose that aR−1b and bR−1c. Then cRb and bRa. SinceR is transitive, we must have cRa. Hence, aR−1c.

16.6 The set R is given by

R = {(a, a), (b, b), (c, c), (d, d), (e, e), (a, b), (a, c), (c, d), (b, e), (b, d), (a, e), (a, d)}.

16.7 (1) R is reflexive since |A| ≤ |A| for all A ∈ F .(2) R is not anti-symmetric: Letting A = {1, 2} and B = {3, 4} we see thatA R B and B R A but A 6= B.(3) R is not symmetric: Letting A = {1, 2} and B = {3, 4, 5} we see thatA R B but B not related to A.(4) R is transitive: Suppose A R B and B R C. Then |A| ≤ |B| and |B| ≤ |C|.Thus, |A| ≤ |C|. That is, A R C.

16.8 (1) Reflexive: For any (a, a) ∈ A, we have (a, a)R(a, a) because a ≤ aand a ≥ a.(2) Anti-symmetric: Suppose (a, b)R(c, d) and (c, d)R(a, b). Then a ≤ c, b ≥d, c ≤ a, and d ≥ b. Hence, a = c and b = d. That is (a, b) = (c, d).(3) Transitive: Suppose (a, b)R(c, d) and (c, d)R(e, f). Then a ≤ c, b ≥ d, c ≤e, and d ≥ f. Hence, a ≤ e and b ≥ f. That is, (a, b)R(e, f).

16.9 {a} is the least element of S. S does not have a greatest element.

Page 280: Discrete Book

276 ANSWER KEY

16.10 The set of negative integers is a non-empty subset of Z with no least-element. Hence, the relation ≤ on Z is not well-ordered.

16.11 Let a, b ∈ A. Then the set {a, b} is a non-empty subset of A so thatit has a least element. That is, either a ≤ b or b ≤ a. But this shows that aand b are comparable. Hence, ≤ is a total order on A.

16.12 Suppose that a and b are least elements of S. Then we have a ≤ b andb ≤ a. Since ≤ is anti-symmetric, we must have a = b.

Page 281: Discrete Book

277

Section 17

17.1 (a) (i) Suppose that g(n1) = g(n2). Then 3n1 − 2 = 3n2 − 2. Thisimplies that n1 = n2 so that g is one-to-one.(ii) g is not onto because there is no integer n such that g(n) = 2.(b) Let y ∈ R such that G(x) = y. That is, 3x− 2 = y. Solving for x we findx = y+2

3∈ R. Moreover, G(y+2

3) = y. So G is onto.

17.2 Suppose that f(x1) = f(x2). Then x1+1x1

= x2+1x2

. Cross multiply toobtain x1x2 + x1 = x1x2 + x2. That is, x1 = x2 so that f is one-to-one.

17.3 Suppose that f(x1) = f(x2). Then x1

x21+1

= x2

x22+1

. Cross multiply to

obtain x1x22 + x1 = x21x2 + x2. That is, (x1 − x2)(x1x2 − 1) = 0. This is

satisfied for example for x1 = 2 and x1 = 12. Thus, f(2) = f(1

2) with 2 6= 1

2.

Thus, f is not one-to-one.

17.4 (a) Since f(0.2) = f(0.3) = 0 with 0.2 6= 0.3, f is not one-to-one.(b) f is onto since for any x ∈ Z, x ∈ R and f(x) = x.

17.5 The answer is no. Let f(x) = x − 1 and g(x) = −x + 1. Then bothfunctions are one-to-one but f + g = 0 is not one-to-one.

17.6 (a) Since F ({a}) = F ({b}) = 1 and {a} 6= {b}, F is not one-to-one.(b) Since Range(F ) = {0, 1, 2, 3} 6= N, F is not onto.

17.7 The answer is no. The functions f, g : Z→ Z defined by f(n) = n + 2and g(n) = n + 3 are both onto but (f + g)(n) = 2n + 5 is not onto sincethere is no integer n such that (f + g)(n) = 2.

17.8 We have (F ◦ F−1)(y) = F (F−1(y)) = F (y−23

) = 3(y−23

) + 2 = y.This shows that F ◦ F−1 = IR. Similarly, (F−1 ◦ F )(x) = F−1(F (x)) =F−1(3x+ 2) = 3x+2−2

3= x. Hence, F−1 ◦ F = IR.

17.9 Let f : {a, b} → {1, 2, 3} be the function given by f(a) = 2, f(b) = 3.Then f is one-to-one. Let g : {1, 2, 3} → {0, 1} be the function given byg(1) = g(2) = 0 and g(3) = 1. Then g is not one-to-one. However, thecomposition function g ◦ f : {a, b} → {0, 1} given by (g ◦ f)(a) = 0 and

Page 282: Discrete Book

278 ANSWER KEY

(g ◦ f)(b) = 1 is one-to-one.

17.10 Let f : {a, b} → {1, 2, 3} be the function given by f(a) = 2, f(b) = 3.Then f is not onto. Let g : {1, 2, 3} → {0, 1} be the function given byg(1) = g(2) = 0 and g(3) = 1. Then g is onto. However, the compositionfunction g ◦ f : {a, b} → {0, 1} given by (g ◦ f)(a) = 0 and (g ◦ f)(b) = 1 isonto.

17.11 Let x1, x2 ∈ X such that f(x1) = f(x2). Then g(f(x1)) = g(f(x2)).That is, (g ◦ f)(x1) = (g ◦ f)(x2). Since g ◦ f is one-to-one, we have x1 = x2.This argument shows that f must be one-to-one.

17.12 Let z ∈ Z. Since g◦f is onto, there is an x ∈ X such that (g◦f)(x) = z.That is, there is a y = f(x) ∈ Y such that g(y) = z. This shows that g is onto.

17.13 Let w ∈ W. Then (h ◦ (g ◦ f))(w) = h((g ◦ f)(w)) = h(g(f(w))).Similarly, ((h ◦ g) ◦ f)(w) = (h ◦ g)(f(w)) = h(g(f(w))). It follows thath ◦ (g ◦ f) = (h ◦ g) ◦ f.

17.14 By Examples 17.4 and 17.7, g ◦ f is bijective. Hence, by Theorem17.1 (g ◦ f)−1 exists. Since (f−1 ◦ g−1) ◦ (g ◦ f) = f−1 ◦ (g−1 ◦ g) ◦ f =f−1 ◦ (IY ◦ f) = f−1 ◦ f = IX and similalrly (g ◦ f) ◦ (f−1 ◦ g−1) = IY , wefind (g ◦ f)−1 = f−1 ◦ g−1 by the uniqueness of the inverse function.

Page 283: Discrete Book

279

Section 18

18.1 The first four terms are:

v1 = 1v2 = 2v3 = 4v4 = 7

18.2 a. We have

F 2k − F 2

k−1 = (Fk+1 − Fk−1)2 − F 2

k−1= F 2

k+1 − 2Fk+1Fk−1= Fk+1(Fk+1 − 2Fk−1)= Fk+1(Fk + Fk−1 − 2Fk−1)= Fk+1(Fk − Fk−1)= Fk+1Fk − Fk+1Fk−1

b.

F 2k+1 − F 2

k − F 2k−1 = (Fk + Fk−1)

2 − F 2k − F 2

k−1= 2FkFk−1

c.

F 2k+1 − F 2

k = (Fk+1 − Fk)(Fk+1 + Fk)= (Fk + Fk−1 − Fk)Fk+2

= Fk−1Fk+2

d. The proof is by induction on n ≥ 0.

Basis of induction: F2F0 − F 21 = (2)(1)− 1 = 1 = (−1)0.

Induction hypothesis: Suppose that Fn+2Fn − F 2n+1 = (−1)n.

Induction step: We must show that Fn+3Fn+1 − F 2n+2 = (−1)n+1. Indeed,

Fn+3Fn+1 − F 2n+2 = (Fn+2 + Fn+1)Fn+1 − F 2

n+2

= Fn+2Fn+1 + F 2n+1 − F 2

n+2

= Fn+2(Fn+1 − Fn+2) + F 2n+1

= −Fn+2Fn + F 2n+1

= −(−1)n = (−1)n+1

Page 284: Discrete Book

280 ANSWER KEY

18.3 Suppose that limn→∞Fn+1

Fn= L. Since Fn+1

Fn> 1 then L ≥ 1.

On the other hand, we have

L = limn→∞Fn+Fn−1

Fn

= limn→∞(Fn−1

Fn+ 1)

= limn→∞( 1Fn

Fn−1

+ 1)

= 1L

+ 1

Multiply both sides by L to obtain

L2 − L− 1 = 0.

Solving we find L = 1−√5

2< 1 and L = 1+

√5

2> 1.

18.4 Suppose that L = limn→∞ xn. Then L =√

2 + L. Squaring both sideswe obtain L2 = L+2. Solving this equation for L we find L = −1 and L = 2.Since L ≥ 0 we see that L = 2.

18.5 a. 1 + 2 + · · ·+ (n− 1) = n(n−1)2

.b. 3 + 2 + 4 + 6 + 8 + · · ·+ 2n = 3 + 2(1 + 2 + · · ·n) = 3 + n(n+ 1).

c. 3 · 1 + 3 · 2 + 3 · 3 + · · ·+ 3 · n = 3n(n+1)2

.

18.6 a. 1 + 2 + 22 + · · ·+ 2n−1 = 2n−12−1 = 2n − 1.

b. 3n−1 + 3n−2 + · · ·+ 32 + 3 + 1 = 3n−12.

c. 2n + 3 · 2n−2 + 3 · 2n−3 + · · ·+ 3 · 22 + 3 · 2 + 3 = 2n+1 − 3.d. 2n − 2n−1 + 2n−2 − 2n−3 + · · · + (−1)n−1 · 2 + (−1)n = 2n

∑nk=0(−

12)k =

2n+1

3(1− (−1

2)n+1).

18.7 Listing the first five terms of the sequence we find:

c1 = 1c2 = 1 + 31

c3 = 1 + 31 + 32

c4 = 1 + 31 + 32 + 33

c5 = 1 + 31 + 32 + 33 + 34

Our guess is cn = 1 + 3 + 32 + · · ·+ 3n−1 = 3n−12. Next, we prove by induction

the validity of this formula.

Page 285: Discrete Book

281

Basis of induction: c1 = 1 = 31−12.

Induction hypothesis: Suppose that cn = 3n−12.

Induction step: We must show that cn+1 = 3n+1−12

. Indeed,

cn+1 = 3cn + 1

= 3(3n−1)2

+ 1

= 3n+1−32

+ 1

= 3n+1−12

.

18.8 Listing the first five terms of the sequence we find:

w0 = 1w1 = 2− 1w2 = 22 − 2 + 1w3 = 23 − 22 + 2− 1w4 = 24 − 23 + 22 − 2 + 1

Our guess is wn =∑n

k=0(−1)k2n−k =2n+1(1−(− 1

2)n+1)

3. Next, we prove the

validity of this formula by mathematical induction.

Basis of induction: w0 = 1 =20+1(1−(− 1

2)0+1)

3.

Induction hypothesis: Suppose that wn =2n+1(1−(− 1

2)n+1)

3.

Induction step: We must show that wn+1 =2n+2(1−(− 1

2)n+2)

3. Indeed,

wn+1 = 2n+1 − wn

= 2n+1 − 2n+1(1−(− 12)n+1)

3

=2n+2(1−(− 1

2)n+2)

3.

18.9 If the given formula is the correcto one then it must satisfy the mathe-matical induction argument.Basis of induction: a1 = 0(1− 1)2.Induction hypothesis: Suppose that an = (n− 1)2.Induction step: We must then have an+1 = n2. This implies 2an +n− 1 = n2

and by the induction hypothesis we have 2(n− 1)2 +n− 1 = n2. Simplifyingto obtain n2 − 3n+ 1 = 0 and this equation has no integer solutions.

Page 286: Discrete Book

282 ANSWER KEY

18.10 a. Yes. A = 2 and B = −5.b. No. A = n depends on n.c. No. Degree of cn−2 is 2.d. Yes. A = 3, B = 1.e. No because of the -2.f. Yes. A = 0, B = 10.

18.11 a. We have the following system{C +D = 12C +D = 3

Solving this system we find C = 2 and D = −1. Hence, an = 2n+1 − 1. Inparticular, a2 = 7.

b. We have the following system{C +D = 02C +D = 2

Solving this system we find C = 2 and D = −2. Hence, an = 2n+1 − 2. Inparticular, a2 = 6.

18.12 Indeed,

3an−1 − 2an−2 = 3(C · 2n−1 +D)− 2(C · 2n−2 +D)= 3C · 2n−1 − C · 2n−1 +D= 2C · 2n−1 +D= C · 2n +D = an.

18.13 The roots of the characteristic equation

t2 − 2t− 3 = 0

are t = −1 and t = 3. Thus,

an = C(−1)n +D3n

is a solution toan = 2an−1 + 3an−2.

Page 287: Discrete Book

283

Using the values of a0 and a1 we obtain the system{C +D = 1−C + 3D = 2.

Solving this system to obtain C = 14

and D = 34. Hence,

an =(−1)n

4+

3n+1

4.

18.14 The single root of the characteristic equation

t2 − 2t+ 1 = 0

is t = 1. Thus,

an = C +Dn

is a solution to

an = 2an−1 − an−2.

Using the values of a0 and a1 we obtain the system{C = 1

C +D = 4.

Solving this system to obtain C = 1 and D = 3. Hence,

an = 3n+ 1.

18.15 The proof is by mathematical induction.

Basis of induction: The formula holds for n = 1. Suppose that a1 is a realnumber. Then

|1∑

k=1

ak| ≤1∑

k=1

|ak|.

Induction hypothesis: Suppose that the formula holds up to n, i.e.

|n∑

k=1

ak| ≤n∑

k=1

|ak|.

Page 288: Discrete Book

284 ANSWER KEY

Induction step: We must show that

|n+1∑k=1

ak| ≤n+1∑k=1

|ak||.

Indeed,|∑n+1

k=1 ak| = |(∑n

k=1 ak) + an+1|≤ |

∑nk=1 ak|+ |an+1|

≤∑n

k=1 |ak|+ |an|=

∑n+1k=1 |ak|.

18.16 The proof is by mathematical induction.

Basis of induction: The property holds for n = 1. Let A and B1 be sets. Bythe recursive definition of the union, ∪1k=1Bk = B1 and ∪1k=1(A∩Bk) = A∩B1.Hence,

A ∩ (∪1k=1Bk) = ∪nk=1(A ∩Bk).

Induction hypothesis: Suppose the property holds up to n, i.e.

A ∩ (∪nk=1Bk) = ∪nk=1(A ∩Bk).

Induction step: We must show that

A ∩ (∪n+1k=1Bk) = ∪n+1

k=1(A ∩Bk).

Indeed,A ∩ (∪nk=1Bk) = A ∩ [(∪nk=1Bk) ∪ An+1]

= (A ∩ (∪nk=1Bk) ∪ (A ∩ An+1

= (∪nk=1(A ∩Bk)) ∪ (A ∩Bn+1)

= ∪n+1k=1(A ∩Bk).

18.17 The proof is by mathematical induction.

Basis of induction. The formula holds for n = 1 since

(∩1k=1Ak)c = (A1)c = ∪1k=1A

ck.

Induction hypothesis: Suppose that the formula holds up to n. That is

(∩nk=1Ak)c = ∪nk=1Ack.

Page 289: Discrete Book

285

Induction step: We must show that

(∩n+1k=1Ak)c = ∪n+1

k=1Ack.

Indeed,(∩n+1

k=1Ak)c = ((∩nk=1Ak) ∩ An+1)c

= (∩nk=1Ak)c ∪ Acn+1

= (∪nk=1Ack) ∪ Ac

n+1

= ∪n+1k=1A

ck.

18.18 If we assume that F is a function then the number F (3) exists. ButF (3) = 1 + F (6) = 1 + F (3). This implies that 1 = 0 which is impossible.Hence, F does not define a function.

Page 290: Discrete Book

286 ANSWER KEY

Section 19

19.1 (a) This is a decision with two steps. For the first step we have 10choices and for the second we have also 10 choices. By the FundamentalPrinciple of Counting there are 10 · 10 = 100 two-digit code numbers.(b) This is a decision with three steps. For the first step we have 9 choices,for the second we have 10 choices, and for the third we have 10 choices. Bythe Fundamental Principle of Counting there are 9 · 10 · 10 = 900 three-digitidentification card numbers.(c) This is a decision with four steps. For the first step we have 10 choices,for the second 9 choices, for the third 8 choices, and for fourth 7 choices. Bythe Fundamental Principle of Counting there are 10 ·9 ·8 ·7 = 5040 four-digitcode numbers.(d) This is a decision with five steps. For the first step we have 9 choices, forthe second 10 choices, for the third 10 choices, for the fourth 10 choices, andfor the fifth 10 choices. By the Fundamental Principle of Counting there are9 · 10 · 10 · 10 · 10 = 90, 000 five-digit zip codes.

19.2 (a) This is a decision with three steps. For the first finisher we have8 choices, 7 choices for the second and 6 choices for the third. By the Fun-damental Principle of Counting there are 8 · 7 · 6 = 336 different first threefinishers.(b) This is a decision with three steps. For the first finisher we have 3 choices,2 choices for the second and 1 choice for the third. By the Fundamental Prin-ciple of Counting there are 3 · 2 · 1 = 6 different arrangements.

19.3 This is a decision with two steps. For the choice of shirts there are2 choices and for the choice of pants there are 3 choices. By the Fundamen-tal Principle of Counting there are 3 · 2 = 6 different choices of outfits.

19.4 This is a decision with two steps. For the choice of president we have10 possible choices, for the choice of the vice-president we have 9 choices. Bythe Fundamental Principle of Counting there are 10 · 9 = 90 different ways.

19.5

Page 291: Discrete Book

287

19.6 48.

19.7 For the first prize we have 20 choices and for the second we have 19choices. By the Fundamental Principle of Counting we have 20 · 19 = 380possible choices.

19.8 For the president we have 24 choices, for the vice-president we have 23choices, for the secretary we have 22 choices, and for the treasurer we have 21choices. By the Fundamental Principle of Counting there are 24 ·23 ·22 ·21 =255, 024 possible choices.

19.9 For the first ranking we have 10 choices, for the second we have 9choices, for the third 8 choices, and for the fourth 7 choices. By the Funda-mental Principle of Counting there are 10 · 9 · 8 · 7 = 5040 possible ways.

19.10 We can look at this problem as a task with 8 seats numbered 1 -8. For the first seat, we have 8 possibilities. For the second seat, we haveonly 1 possibility since each person must sit next to his/her spouse. For thethird seat, there are 6 possibilities. For the fourth seat, 1 possibility, the fifthseat, 4 possibilities, the sixth seat 1 possibility, the seventh seat 2 possibili-ties, and the eight seat 1 possibility. By the multiplication rule of counting,there are 8 · 6 · 4 · 2 = 384 ways.

Page 292: Discrete Book

288 ANSWER KEY

Section 20

20.1 If m!(m−n)! = 9!

6!then m = 9 and n = 3.

20.2 (a) 26 · 26 · 26 · 26 = 456976(b) 26P4 = 26!

(26−4)! = 26 · 25 · 24 · 23 = 358800.

20.3 (a) 26P3 · 10 · 10 · 10 = 15600000(b) 26P3 ·10 P3 = 15600 · 720 = 11232000.

20.4 (a) 40 · 40 · 40 = 64000(b) 40P3 = 40!

(40−3)! = 40!37!

= 59280.

20.5 (a) 12P12 = 12! = 479001600(b) 7P7 ·5 P5 = 7! · 5! = 5040 · 120 = 604800.

20.6 (a) 5(b) 5P2 = 20(c) 5P3 = 60(d) 5P4 = 120.

20.7 There are 5P3 = 60 different ways.

20.8 26P3 = 15600.

Page 293: Discrete Book

289

Section 21

21.1 If mCn = m!n!(m−n)! = 13 then we can choose m = 13 and n = 1 or n = 12.

21.2 Since order is not of importance here, the number of different choicesis 42C3 = 11480

21.3 Since the handshake between persons A and B is the same as thatbetween B and A, this is a problem of choosing combinations of 25 peopletwo at a time. There are 25C2 = 300 different handshakes.

21.4 Since order is irrelevant, the different two-person committe is 5C2 = 10.

21.5 There are 8C2 = 28 different ways.

21.6 There are 30C3 = 4060 different ways.

21.7 Recall that mPn = m!(m−n)! = n!mCn. Since n! ≥ 1, we can multiply

both sides by mCn to obtain mPn = n!mCn ≥ mCn.

21.8 (a) The order of how you select the toppings is irrelevant. Thus, thisproblem is a combination problem.(b) The order of the winners is important. Thus, the problem is a permuta-tion problem.

21.9 We have

(a+ b)7 = a7 + 7a6b+ 21a5b2 + 35a4b3 + 35a3b4 + 21a2b5 + 7ab6 + b7.

21.10 The 5th term in the binomial theorem corresponds to k = 4. Thus, theanswer is 7C4(2a)3(−3b)4 = 22, 680a3b4.

Page 294: Discrete Book

290 ANSWER KEY

Section 22

22.1 The probability is 1352

= 14.

22.2 (a) 26

= 13.

(b) 36

= 12.

22.3 782920

= 391460.

22.4 (a) The events are not mutually exclusive since a person can get aticket for both speeding and going through a red light.(b) The events are mutually exclusive since the president of the United Statescannot be a foreign born person.

22.5 (a) P (Ac) = 1− P (A) = 1− .22 = .78 = 78%(b) P (A ∪B) = P (A) + P (B) = .22 + .35 = .57 = 57%(c) P (A ∩B) = 0.

22.6 P(failing grade in Stat or in English) = P(failing grade in Stat) +P(failing grade in English) - P(failing grade in stat and English) = .20 +.15− .03 = .32 = 32%

22.7 Let A = research project well planned, B = research project executed.Then

P (B|A) =P (A ∩B)

P (A)=.54

.60= .9 = 9%

22.8 Indeed,

P (A|B) =P (A ∩B)

P (B)=.15

.30= .5 = P (A).

So A and B are independent.

22.9 P (f = 0) = P ({(T, T, T, T )} = 116,

P (f = 1) = P ({(H,T, T, T ), (T,H, T, T ), (T, T,H, T ), (T, T, T,H)} = 416,

P (f = 2) = P ({(H,H, T, T ), (H,T,H, T ), (H,T, T,H), (T,H,H, T ), (T,H, T,H), (T, T,H,H)} =616,

P (f = 3) = P ({(H,H,H, T ), (H,T,H,H), (T,H,H,H), (H,H, T,H)} = 416,

P (f = 4) = P ({(H,H,H,H)} = 116.

Page 295: Discrete Book

291

Section 23

23.1 The pigeons are the 10 children and the pigeongoles are the days ofthe week.

23.2 The 11 people are the pigeons and the 10 floors are the pigeonholes.By the Pigeonhole principle, at least two people will exist the elevator on thesame floor.

23.3 The pigeons are the 11 students. The smallest number of errors is0 and the largest number of errors is 9. The Pigeonholes are the number oferrors of each student with one of them reserved to Gearoge Perry. By thePigeonhole principle, there are at least two students that made equal numberof errors.

23.4 The pigeons are the five selected integers and the holes are the subsets{1, 8}, {2, 7}, {3, 6}, {4, 5}. By the Pigeonhole principle, there are at least twomust be from the same subset. But then the sum of these two integers is 9.

23.5 The remainder of the division of a natural number by 11 are: 0,1,2,3,4,5,6,7,8,9,10.If we count these as the pigeongoles and the 12 numbers as the pigeons thenby the Pigeonhole principle there must be at least two numbers with thesame remainder. But in this case, the difference of these two numbers is amultiple of 11, i.e., the difference is divisible by 11.

23.6 Let S by the number of people and T the number of birthdays. Then|S| = 1500 and |T | = 366. Note that |S| > 4|T |. Let f : S → T be the func-tion f(x) = x′s birthday. Thus, for a given birthday, Example 23.2 assertsthat at least five people share the same birthday.

23.7 There are 16 combinations of heads and tails. Since there are morethan 16 students in this class, at least two of them had to have the exactsame sequence. In this case, the “pigeons” are the students, and the “holes”into which they are being placed are the possible sequences of heads andtails.

Page 296: Discrete Book

292 ANSWER KEY

Section 24

24.1 VG = {a, b, c, d}, EG = {(a, c), (a, d), (b, d), (b, c), (c, d)}.

24.2 (a) EG = {e1, e2, e3, e4, e5, e6}, VG = {v1, v2, v3, v4, v5, v6, v7} (b) v5 (c)e5 (d) {e2, e3} (e) {v2, v4} (f) {e1, e4, e5}.

24.3 (a) G is not simple since it has a loop and parallel edges. (b) G issimple.

24.4

24.5 (a) Bipartite. (b) Any two sets of vertices of K3 will have one setwith at least two vertices. Thus, according to the definition of bipartitegraph, K3 is not bipartite.

24.6

24.7 The graph G has deg(u) = 2, deg(v) = 3, deg(w) = 4 and deg(z)= 1.

Page 297: Discrete Book

293

24.8

24.9 (a)

(b) 0 2 0 22 0 2 10 2 0 12 1 1 0

Page 298: Discrete Book

294 ANSWER KEY

24.10

24.11vertex in− degree out− degreev1 3 3v2 2 4v3 2 0v4 2 2v5 1 1

24.12 Each edge contributes 1 to the indegree of one vertex, so contributes1 to the sum

∑v∈VG

deg−(v). Thus, the sum∑

v∈VGdeg−(v) is just the total

number of edges. Similarly for the outdegrees.

24.13

e1 e2 e3 e4 e5 e6v1 1 0 0 0 0 0v2 0 1 1 0 0 0v3 0 1 1 1 0 0v4 1 0 0 1 2 0v5 0 0 0 0 0 0v6 0 0 0 0 0 1v7 0 0 0 0 0 1

24.14 Since 2|EG| = 60, |EG| = 30. So the graph has 30 edges.

Page 299: Discrete Book

295

Section 25

25.1 (a) Ae1Be3De7Ee6C. (b) Ae2De7Ee6Ce4B. (c) Ae2De7Ee6Ce4Be1A.

25.2

25.3 (I) is disconnected (II) is connected.

25.4 Each vertex has an even degree. Hence, by Theorem 25.1, the graphhas an Euler circuit.

25.5 There is no path that visits every vertex of the graph exactly once.

25.6 Both graphs are disconnected.

25.7 (a) 3 connected components (b) 2 connected components.

25.8

25.9 (a) Vertices A and D have odd degrees. (b) Vertices B and D have odddegrees.

Page 300: Discrete Book

296 ANSWER KEY

25.10 (a) deg(A) = 1, deg(B) = deg(D) = 3. (b) deg(A) = deg(D) =1, deg(B) = 3.

25.11 Only A and B are of odd degree.

25.12 Any circuit will have to cross D twice.

25.13 No.

25.14 Yes.

Page 301: Discrete Book

297

Section 26

26.1 Graphs A and C satisfy the definition of a tree. Graph B has a circuitand Graph D is disconnected.

26.2 57.

26.3 Yes provided the graph is connected.

26.4 The root is a.

vertex levelb 1c 2d 2e 3f 4g 3h 1i 2j 2k 2

The height of the tree is 4.

26.5 (a) v3. (b) {v0, v3, v5}. (c) {v7, v8, v9}. (d) {v4, v7, v8, v9}. (e) {v1, v2, v3}, {v5, v6}, {v7, v8, v9}.(f) {v2, v6, v7, v8, v9, v10. (g)

26.6

Page 302: Discrete Book

298 ANSWER KEY

26.7

Given a number between 1 and 15, it is first compared with 8; if it is lessthan 8, proceed to key 4; if it greater than 8, proceed to key 12; if it is equalto 8 the search is over. One proceeds down the tree in this manner until thenumber is found.

Page 303: Discrete Book

299

26.8 Let Ts, Tv, and Tw be the trees rooted at s, v, and w respectively. Ap-plying the preorder traversal algorithm we obtain the following listings:Tw : w, x, y, z.Tv : v, u, Tw or v, u, w, x, y, z.Ts : s, p, q.T : r, Tv, Ts or r, v, u, w, x, y, z, s, p, q.

26.9 The first graph is a forest whereas the second is not.

26.10 They have the same number of edges.

26.11

Page 304: Discrete Book

300 ANSWER KEY

Page 305: Discrete Book

Index

Absolute complement, 97Acyclic, 202Adjacency matrix, 198Adjacent vertices, 190Ancestors, 215Antisymmetric, 123Argument, 33Arithmetic sequence, 144Axiom, 54Axiomatic set theory, 94

Barber puzzle, 94Bernoulli’s inequality, 77Biconditional proposition, 29Bijective function, 139Binary relation, 119Binary search tree, 222Binary tree, 216Binomial coefficient, 170Binomial Theorem, 170Bipartite graph, 193Boolean algebra, 111Boolean expression, 18Boolean variable, 18Branch, 212

Cardinality, 107Cartesian product, 99, 118Ceiling function, 63Characteristic equation, 146Child, 215

Circuit, 202Circular proof, 64Classical probability, 174Closed path, 202Combination, 168Comparable, 133Complete bipartite graph, 193Complete graph, 192Composite number, 84Composition of relations, 122Compound propositions, 5Conclusion, 27, 33Conditional connective, 27Conditional proposition, 27Congruent modulo n, 91Conjunction, 5Conjunctive addition, 36Conjunctive simplification, 37Connected components, 208Connected graph, 203Constructive proof, 54Continuous random variable, 180Contradiction, 11Contrapositive, 28Converse, 28Corollary, 54Counterexample, 45, 58Cycle, 202

De Morgan’s Laws, 105De Morgan’s laws, 8

301

Page 306: Discrete Book

302 INDEX

Decision tree, 221Definition, 54Degree of a vertex, 194Dependent events, 178Descendant, 215Descriptive form, 94Digraph, 120, 190Direct method of proof, 56Directed edge, 120Directed graph, 190Disconnected graph, 203Discrete random variable, 180Disjoint sets, 98Disjunction, 5Disjunctive addition, 36Disjunctive syllogism, 37Divisible, 84Division Algorithm, 82Domain, 119Domain of discourse, 44

Edges, 190Empty set, 95Equal sets, 96Equally likely, 174Equivalence classes, 125Equivalence relation, 124Equivalent circuits, 18Equivalent propositions, 8Estimated probability, 175Euclidean Algorithm, 89Euler circuit, 203Euler path, 203Event, 174Exclusive or, 6Existential quantifier, 46Expected value, 181Experiment, 174

Factorial, 163Fibonacci, 143Finite set, 107Floor function, 63Forest, 223Free variable, 44Full binary tree, 217Function, 120Fundamental Theorem of Arithmetic,

84

Generating rule, 143Geometric progression, 74Geometric sequence, 144Greatest common divisor, 85

Hash Function, 137Hasse diagram, 132Height, 214Histogram, 180Hupothetical syllogism, 38Hypothesis, 27

In-degree, 199Incidence matrix, 200Independent events, 178Indirect proof method, 68Inference, 33Infinite set, 107Initial condition, 143Injective, 137Intermediate Value Theorem, 55Intersection of sets, 98Invalid argument, 34Inverse, 28Inverse relation, 121Inverter, 15Isolated vertex, 191Iteration, 143

Page 307: Discrete Book

INDEX 303

Leaf, 212, 215Least element, 134Left child, 216Lemma, 54Level of a vertex, 214Logic, 54Logic gate, 15Loop, 120, 191

Mathematical induction, 73Mathematical system, 54Method of exhaustion, 55Modus ponens, 35Modus Tollens, 35Multiplication rule of counting, 158Mutually exclusive, 176

Naive set theory, 94Natural numbers, 44Negation, 7Nonconstructive proof, 55

One’s complement, 20One-to-one, 137One-to-one correspondence, 139Onto function, 138Ordered pair, 118Out-degree, 199

Paradox, 94Parallel edges, 190Parent, 215Partial order, 131Partition of sets, 106Pascal’s identity, 169Pascal’s triangle, 171Path of length n, 202Permutation, 163Pierce arrow, 15

Pigeonhole principle, 186Poset, 131Power set, 107Predicate, 44Premises, 33Preorder traversal, 222Prime number, 84Probability distribution, 180Projection function, 138Proof, 54Proof by cases, 62Proof by contradiction, 68Proof by contrapositive, 69Proper subset, 96Proposition, 4Propositional functions, 5Propositional variables, 5

Quantifier, 45

Range, 119Recurrence, 143Reflexive, 122Regular graph, 201Relative complement, 97Relatively prime, 86Right child, 216Rooted tree, 214Rule of contradiction, 38Russell’s Paradox, 94

Sample space, 174Scheffer stroke, 15Set, 93, 94Set-builder form, 95Siblings, 215Simple graph, 191Simple path, 202Spanning tree, 218

Page 308: Discrete Book

304 INDEX

Standard deviation, 182Subgraph, 199Subset, 95Subtree, 215Surjective, 138Symbolic connectives, 5Symmetric, 123Symmetric difference, 99

Tabular form, 94Tautology, 8Theorem, 54Total degree, 194Total order, 133Transitive, 123Tree, 212Tree diagram, 158Trichotomy Law, 131Trivial proof, 62Truth set, 44Truth table, 6Truth Value, 4two’s complement, 20

Undirected graph, 190Unioin of sets, 97Unique Factorization Theorem, 84universal conditional proposition, 46Universal quantifier, 45Universal set, 97

Vacuous proof, 62Vacuously true, 27Valid argument, 34Variance, 182Venn diagrams, 96Vertex, 120Vertices of a graph, 190

Well order, 134Well-Ordering Principle, 82